Case Closed! Neuroanatomy 9781138381773, 1138381772

763 125 29MB

English Pages [353] Year 2018

Report DMCA / Copyright

DOWNLOAD FILE

Polecaj historie

Case Closed! Neuroanatomy
 9781138381773, 1138381772

Table of contents :
Content: Part I. General Anatomy Overview. Peripheral Nervous System. Spinal Cord. Brainstem and Cranial Nerves. Basal Ganglia and Cortex. Part II: The Cases. Peripheral Lesions. Spinal Cord. Brainstem. Cranial Nerve. Lacunar. Cortex.

Citation preview

CASE CLOSED!

NEUROANATOMY

K25928_C000.indd 1

10/01/2017 10:48

CASE CLOSED!

NEUROANATOMY

WARREN BERGER BSc BESc MD JOHN BERGER BA

K25928_C000.indd 3

10/01/2017 10:48

CRC Press Taylor & Francis Group 6000 Broken Sound Parkway NW, Suite 300 Boca Raton, FL 33487-2742 © 2017 by Taylor & Francis Group, LLC CRC Press is an imprint of Taylor & Francis Group, an Informa business No claim to original U.S. Government works Printed on acid-free paper International Standard Book Number-13: 978-1-4987-2852-2 (Paperback) This book contains information obtained from authentic and highly regarded sources. While all reasonable efforts have been made to publish reliable data and information, neither the author[s] nor the publisher can accept any legal responsibility or liability for any errors or omissions that may be made. The publishers wish to make clear that any views or opinions expressed in this book by individual editors, authors or contributors are personal to them and do not necessarily reflect the views/opinions of the publishers. The information or guidance contained in this book is intended for use by medical, scientific or health-care professionals and is provided strictly as a supplement to the medical or other professional’s own judgement, their knowledge of the patient’s medical history, relevant manufacturer’s instructions and the appropriate best practice guidelines. Because of the rapid advances in medical science, any information or advice on dosages, procedures or diagnoses should be independently verified. The reader is strongly urged to consult the relevant national drug formulary and the drug companies’ and device or material manufacturers’ printed instructions, and their websites, before administering or utilizing any of the drugs, devices or materials mentioned in this book. This book does not indicate whether a particular treatment is appropriate or suitable for a particular individual. Ultimately it is the sole responsibility of the medical professional to make his or her own professional judgements, so as to advise and treat patients appropriately. The authors and publishers have also attempted to trace the copyright holders of all material reproduced in this publication and apologize to copyright holders if permission to publish in this form has not been obtained. If any copyright material has not been acknowledged please write and let us know so we may rectify in any future reprint. Except as permitted under U.S. Copyright Law, no part of this book may be reprinted, reproduced, transmitted, or utilized in any form by any electronic, mechanical, or other means, now known or hereafter invented, including photocopying, microfilming, and recording, or in any information storage or retrieval system, without written permission from the publishers. For permission to photocopy or use material electronically from this work, please access www.copyright.com (http://www.copyright.com/) or contact the Copyright Clearance Center, Inc. (CCC), 222 Rosewood Drive, Danvers, MA 01923, 978-750-8400. CCC is a not-for-profit organization that provides licenses and registration for a variety of users. For organizations that have been granted a photocopy license by the CCC, a separate system of payment has been arranged. Trademark Notice: Product or corporate names may be trademarks or registered trademarks, and are used only for identification and explanation without intent to infringe. Visit the Taylor & Francis Web site at http://www.taylorandfrancis.com and the CRC Press Web site at http://www.crcpress.com

K25928_C000.indd 4

10/01/2017 10:48

v

Contents Preface

vi

Acknowledgements

vii

Abbreviations

viii

Part One 1 Introduction to the Nervous System

3

2 The Cortex and Associated Structures

23

3 The Cranial Nerves and the Brainstem

55

4 The Spinal Cord

97

5 The Peripheral Nervous System

119

6 Localization Primer

153

Part Two 7 The Cases

177

Index

329

K25928_C000.indd 5

10/01/2017 10:48

vi

Preface Neuroanatomy texts typically fall into two distinct groups. The more traditional text is usually in excess of 1000 pages and covers every esoteric detail of this already challenging subject. At the other end of the spectrum are short texts, typically less than 150 pages, that succinctly summarize material and may be useful for review studies but don’t actually teach neuroanatomy. Neither of these is particularly useful for the beginning student. Indeed, a straightforward text designed exclusively for novice studies in neuroanatomy has remained elusive. Appropriately frustrated, many students have shunned the subject. This phenomenon has been coined “Neurophobia” in three international studies of medical students. It doesn’t need to be this way. That’s where Case Closed! Neuroanatomy comes in. We know students can’t afford the inefficiency of sifting through pages and pages of a tome of a text, trying to determine whether what they are reading is or isn’t relevant. Conversely, the blunt memorization encouraged by short review texts leads to a poor understanding of the art of localization; localization is the skill that neuroanatomy texts try to impart and involves determining where in the nervous system the dysfunction is located. In Chapters 1–5 we present the core fundamentals of neuroanatomy as we work through the nervous system at all its levels: the cortex, brainstem, spinal cord and finally the peripheral nervous system. An uncluttered, concise figure is worth more than a thousand words, so we accompany each figure with an entire page of explanatory text. Key points are then summarized in our Take Home Message. In Chapter 6 we introduce how to localize, and also present a Localization Key. This is designed to serve as a quick reference for all the common localizations you should be familiar with. Finally, in Chapter 7 we present 25 cases for you to practice and master the art of localization. Each case is complemented with a detailed step-by-step solution and a Clinical Pearl, which supplements the case with practical clinical points. Case Closed! Neuroanatomy is everything you need and nothing you don’t. “Neurophobia” exists because of a lack of restraint from instructors, not a lack of enthusiasm from students. We hope that this text remedies that. Warren Berger John Berger February 2016

K25928_C000.indd 6

10/01/2017 10:48

vii

Acknowledgements When undertaking an enterprise of this size, one finds that the list of people to thank nearly becomes the size of the original text itself. The following is by no means exhaustive and we are indebted to the many students who have contributed their thoughts and desires as to what a neuroanatomy text should be. Firstly, we’d like to thank our parents for teaching us the value of divergent thinking and instilling in us the tenacity to see an idea through to its conclusion. To our wives, Anna and Allison, we’d like to thank you for your love and support during this challenging time. You have had to do more than your fair share; thank you for cancelling countless weekend plans to allow us to work on this project. Warren Berger would like to thank the faculty and residents of the Department of Clinical Neurological Sciences at the University of Western Ontario for their inspiration and instruction over the years. In addition, he would like to thank Drs. Jorge Burneo, Alex Fraser and Shannon Venance for (patiently) teaching him how to localize. Deep appreciation also goes to Drs. Ian Taylor and Mike Wiley of the University of Toronto for introducing him a passion for teaching anatomy. He would also like to thank Drs. Nicholas Cothros, Nevena Markovic and Miljan Tripic for joining him on this residency adventure. It has been said that your friends are the ones you consider your equals, but your closest friends are the ones you consider your betters. The latter is certainly the case here. A finer group of residents could not be found. Finally, we would like to thank our editor Dr. Joanna Koster and everyone at CRC Press. Without your guidance and gentle redirections on this adventure we would have veered far off course! Warren Berger John Berger February 2016

K25928_C000.indd 7

10/01/2017 10:48

viii

Abbreviations Abd

abduct

EF

elbow flexor

ABP

abductor pollicis

EHB

extensor hallucis brevis

ACA

anterior cerebral artery

EHL

extensor hallucis longus

Ach

acetylcholine

EMG

electromyogram

ACOM anterior communicating artery

EWN

Edinger–Westphal nucleus

Add

adductors

EP

extensor pollicis

ADF

ankle dorsiflexion

EV

eversion (ankle)

ADP

adductor pollicis

FE

finger extensors

AF

arcuate fasciculus

FEF

frontal eye fields

AHC

anterior horn cell

FF

finger flexors

AICA

anterior inferior cerebellar artery

FP

flexor pollicis

AIDP

acute inflammatory demyelinating polyneuropathy

GBS

Guillain–Barre syndrome

HE

hip extensors

ALS

amyotrophic lateral sclerosis

HF

hip flexors

AP

anterior – posterior

HIV

human immunodeficiency virus

APF

ankle plantar flexion

IAM

internal acoustic meatus

B

Broca’s area

ICA

internal carotid arteries

BIPAP bilevel positive airway pressure

ICP

intracranial pressure

C

conception tract

ICU

intensive care unit

CN

cranial nerve

INO

internuclear ophthalmoplegia

CNS

central nervous system

INV

inversion (ankle)

CSF

cerebrospinal fluid

IO

inferior oblique muscle

CT

computed tomography

IR

inferior rectus muscle

CTS

carpal tunnel syndrome

IV

intravenous

DC

dorsal column(s)

KE

knee extensors

DI

dorsal interossei

KF

knee flexors

DRG

dorsal root ganglion

LGN

lateral geniculate body

EC

extensor carpi

LHH

ECR

extensor carpi radialis

left sided homonymous hemianopsia

ECU

extensor carpi ulnaris

LMN

lower motor neuron

K25928_C000.indd 8

10/01/2017 10:48

Abbreviations

LOC

level of consciousness

RAS

reticular activating system

MCA

middle cerebral artery

RAPD

relative afferent pupillary defect

MCAT

medical college admissions test

RHH

MGN

medial geniculate nucleus

right sided homonymous hemianopsia

ML

medial lemniscus, also medial – lateral (plane)

SACD

subacute combined degeneration

MLF

medial longitudinal fasciculus

SCA

superior cerebellar artery

MMA

middle meningeal artery

SCM

sternocleidomastoid

MRC

Medical Research Council

SO

superior oblique muscle

MRI

magnetic resonance imaging

SR

superior rectus muscle

MS

multiple sclerosis

ST

spinothalamic tract

NE

norepinephrine

SUP

supinator

NMJ

neuromuscular junction

TCM

transcortical motor (aphasia)

OP

opponens (short for opponens pollicis)

TCS

transcortical sensory (aphasia)

TIA

transient ischemic attack

posterior cerebral arteries

UMN

upper motor neuron

PCOM posterior communicating arteries

VA

ventral anterior nucleus

VLN

ventral lateral nucleus

PICA

posterior inferior cerebellar artery

VPL

ventral posterior lateral nucleus

PNS

peripheral nervous system

VPM

ventral posteromedial nucleus

PPRF

paramedian pontine reticular formation

W

Wernicke’s area

WE

wrist extensor

PRO

pronator (short for pronator teres)

WF

wrist flexor

PCA

K25928_C000.indd 9

ix

10/01/2017 10:48

Part One

K25928_C001.indd 1

10/01/2017 10:54

Chapter

1

Introduction to the Nervous System

K25928_C001.indd 3

10/01/2017 10:54

4

Chapter 1 Introduction to the Nervous System

Figure 1.1 Divisions of the Nervous System

K25928_C001.indd 4

10/01/2017 10:54

Divisions of the Nervous System

5

Let’s begin by examining the divisions of the nervous system. The human nervous system is first divided into the central nervous system (CNS) and the peripheral nervous system (PNS). The CNS is made up of the brain and spinal cord. The PNS is made up of cranial nerves (CN), which innervate the head, and spinal nerves, which innervate the body. The function of the PNS is to transmit information to and from the CNS. Since many nerves feed into the spinal cord, it is sometimes called the “bridge” between the PNS and CNS. The PNS can be further subdivided into the autonomic nervous system, and the somatic nervous system. The autonomic nervous system is not under conscious control; it automatically controls all of the functions needed to sustain life (i.e., blood pressure, respiratory rate, digestion and excretion). The autonomic nervous system can be further subdivided into the sympathetic nervous system, which readies you for an impeding conflict, and the parasympathetic nervous system, which conserves energy and regulates growth. The somatic nervous system, by contrast, is under conscious control. It is responsible for conscious sensation (such as feeling something is cold, or that something is sharp) as opposed to the unconscious sensation of the autonomic nervous system (such as your carotid bulbs sensing decreased blood perfusion to the brain). It is also responsible for the voluntary control of muscles. The autonomic and somatic systems act through the CN in the head as well as the spinal nerves in the body. The autonomic system and the somatic system should be thought of as distinct entities. However, they can both travel together in the same nerve. For example, CN III is considered to have a somatic and autonomic component. However, this is usually for just a short part of the nerve pathway, and the components soon separate. We will examine this further in Chapter 3.

Take Home Messages The CNS is made up of the brain and spinal cord. The PNS is composed of the somatic nervous system, under conscious control, and the autonomic nervous system, which is not under conscious control. The somatic system and the autonomic system act both through cranial nerves and spinal nerves.

K25928_C001.indd 5

10/01/2017 10:54

6

Chapter 1 Introduction to the Nervous System

Figure 1.2 Anatomy of a Neuron

K25928_C001.indd 6

10/01/2017 10:54

Anatomy of a Neuron

7

Before we continue, it is useful to go over the anatomy of a neuron. A neuron is a single cell composed of three main parts – dendrites, a single cell body and a single axon. Dendrites transmit information signals toward the cell body of the neuron. Axons transmit information away from the cell body, in the form of an electrical impulse called an action potential. Action potentials are caused by transient changes in the voltage measured between a neuron’s cell membrane and the exterior environment. The cell body contains the nucleus and all the cellular machinery necessary to sustain the neuron. An axon is covered in myelin, which is produced in the PNS by the Schwann cell. In the CNS, myelin is produced by another kind of cell, the oligodendrocyte. Nodes of Ranvier are small areas along the axon that do not contain myelin. Myelin allows for much more rapid transmission of action potentials, as they can jump between Nodes of Ranvier as shown in Fig. 1.2. Many dendrites can feed into one cell body, but each cell body has only one axon. Even though most books draw the dendrites as being quite short, they can be very long, even longer than the axon. It is important to note that one neuron spans the entire length of any individual nerve. Thus, the neurons of the sciatic nerve are over 1 meter (over 3 feet) long! The nomenclature of neuron cell bodies is important and can be a cause of confusion. In the CNS a collection of cell bodies is called a nucleus. In the PNS a collection of cell bodies is called a ganglion (pl. ganglia). This difference is illustrated nicely by the autonomic nervous system, as we will see in a few pages.

Take Home Messages Dendrites transmit information towards the cell body and axons transmit information away from the cell body. Axons are covered in myelin, which is made by Schwann cells in the PNS and oligodendrocytes in the CNS. A collection of cell bodies is called a nucleus in the CNS and a ganglion in the PNS.

K25928_C001.indd 7

10/01/2017 10:54

8

Chapter 1 Introduction to the Nervous System

Figure 1.3 Overview of the Central Nervous System

K25928_C001.indd 8

10/01/2017 10:54

Overview of the Central Nervous System

9

As we saw earlier, the CNS is composed of the brain and spinal cord. The brain itself is made up of the cerebrum, brainstem and cerebellum. The cerebrum is what most people picture when they think of the brain. It is divided into two identical hemispheres: one on the left side and one on the right side. The outer layer of the cerebrum is called the cortex and made up of folded layers of tissue called gray matter. Gray matter houses neuronal cell bodies and initiates neuronal impulses. The other type of tissue in the CNS is white matter, which contains the axons that transmit the impulses initiated from the gray matter. If you examine the cerebrum shown in Fig. 1.3, you will see it is composed of many ridges and valleys. A ridge is called a gyrus and a valley is called a sulcus. The central sulcus is important because the gyrus anterior to it is the primary motor cortex, which initiates all motor functions. The gyrus posterior to the central sulcus is the primary sensory cortex, which receives all sensory input. The cerebrum is divided into lobes by various anatomical landmarks. The frontal lobe extends back to the central sulcus, and inferiorly until the Sylvian fissure. The frontal lobe is responsible for thinking, planning and all motor control. The parietal lobe is posterior to the central sulcus but doesn’t have a specific defining landmark between it and the temporal or occipital lobes. The parietal lobe receives and integrates all sensory information. The temporal lobe sits inferior to the Sylvian fissure, but doesn’t have a defined posterior border; it creates and stores new memory and is involved in emotional regulation. It also contributes to the understanding of language. Finally, the occipital lobe serves as the visual processing center. The brainstem sits just inferior to the cerebrum, and is divided into three sections called the midbrain, pons and medulla. The brainstem regulates all life preserving functions and controls consciousness. It also provides all innervation to the head via the CN. Just inferior to it sits the cerebellum, which is responsible for coordinating smooth, voluntary movement. Just as the brainstem provides all innervation to the head via the CN, the spinal cord provides all innervation to the body via the spinal nerves.

Take Home Messages The brain is made up of the cerebrum, brainstem and cerebellum. The cerebrum is divided into four lobes: frontal, parietal, temporal and occipital. The brainstem is divided into the midbrain, pons and medulla. The brainstem provides all innervation to the head, and the spinal cord provides all innervation to the body.

K25928_C001.indd 9

10/01/2017 10:54

10

Chapter 1 Introduction to the Nervous System

Figure 1.4 Overview of the Peripheral Nervous System

K25928_C001.indd 10

10/01/2017 10:54

Overview of the Peripheral Nervous System

11

The PNS consists of the CN and the spinal nerves. We will discuss the CN in detail in Chapter 3. In order to understand the spinal nerves, we need to briefly discuss their relation to the spinal cord. A total of 31 paired spinal nerves exit the spinal cord. Because of morphological differences that occur in the spinal cord, the spinal cord and its associated nerves are divided into four different sections: cervical, thoracic, lumbar and sacral. There are 8 cervical nerves, 12 thoracic nerves, 5 lumbar nerves and 5 sacral nerves (in addition there is one coccygeal nerve, but it serves no clinically relevant purpose so it is usually ignored). Nerves are named by stating which group they belong to, and the relative position of the nerve in the group; for example the 6th nerve of the cervical group is called Cervical 6 (abbreviated C6). If we take a closer look at the spinal cord we will see that a total of four nerve roots exit the spinal cord (two on each side). There is an anterior nerve root, which carries motor signals to muscle, and a posterior nerve root, which carries sensory information to the brain. The nucleus for the motor neuron lies in a special area of the gray matter, called the anterior horn cell (AHC). The cell body for the sensory neuron lies outside the cord in what is called the dorsal root ganglion (DRG). Just distal to the DRG, the nerve roots merge to form a spinal nerve. The spinal nerves destined for the arm or the leg travel a short distance and then enter a mesh-like network called a plexus. In the plexus, the spinal nerves mix together and ultimately form individual peripheral nerves. The peripheral nerves travel throughout the body to their end target, which is a muscle or a sensory organ (usually skin). The plexus in the upper limb is called the brachial plexus and the plexus in the lower limb is called the lumbosacral plexus.

Take Home Messages 31 paired spinal nerves exit the spinal cord: 8 cervical, 12 thoracic, 5 lumbar, 5 sacral and 1 coccygeal (often ignored). The nucleus for the motor neuron lies in the AHC of the spinal cord. The cell body of the sensory neuron lies outside the cord, in the DRG. The anterior nerve root carries motor signals and the posterior nerve root carries sensory signals. Spinal nerves mix together in the plexus to form peripheral nerves.

K25928_C001.indd 11

10/01/2017 10:54

12

Chapter 1 Introduction to the Nervous System

Figure 1.5 Long Tracts of the Central Nervous System

LMN: lower motor neuron; UMN: upper motor neuron.

K25928_C001.indd 12

10/01/2017 10:54

Long Tracts of the Central Nervous System

13

There are three different pathways, or tracts, that the CNS uses to transmit and receive information, as shown in Fig. 1.5. Collectively, these three pathways are often referred to as the long tracts because they run longitudinally throughout the entirety of the CNS. Let’s consider each of these. The motor pathway is made up of two neurons; an upper motor neuron (UMN) and a lower motor neuron (LMN). The UMN travels in the corticospinal tract, which runs from the motor cortex to the AHC in the spinal cord. As the corticospinal tract passes through the medulla it crosses over, or decussates, to the contralateral side. The UMN continues downward and eventually exits the corticospinal tract to synapse with the LMN in the AHC. The LMN exits the spinal cord, and travels as part of the peripheral nerve until it reaches its end muscle. As we will examine further later, lesions of the UMN produce very different symptoms than lesions of the LMN. The sensory pathway is made up of three neurons called the 1st order, 2nd order and 3rd order neurons. The CNS has two different sensory tracts and they carry different types of information. The spinothalamic tract (ST) carries pain and temperature information. The dorsal columns (DC) carry vibration and proprioception. Proprioception is the innate knowledge of where your body is in space (i.e., if your eyes are closed and you put your hand in front of your face, you are still are aware that your hand is in front of your face). Unfortunately the DC undergo a needless name change once it synapses with its 2nd order neuron, and is afterwards known as the medial lemniscus (ML). The 1st order neuron for both paths begin in the PNS and travel into the spinal cord; as they do, they pass their cell body, which is housed in the DRG. If the neuron carries vibration/proprioceptive it enters the DC. It continues up the DC into the medulla where it synapses with its 2nd order neuron, decussates and is now known as the ML. The 2nd order neuron travels into the thalamus where it synapses with the 3rd order neuron and then terminates in the sensory cortex. If the 1st order neuron carries pain/temperature signals, it synapses with its 2nd order neuron upon entering the spinal cord and decussates to enter the contralateral ST. It then continues up the ST, through the brainstem and synapses with its 3rd order neuron in the thalamus before finally terminating in the sensory cortex.

Take Home Messages Motor information is transmitted by the UMN (motor cortex to AHC) and the LMN (AHC to muscle). The UMN travels in the corticospinal tract and decussates at the medulla. The ST decussates upon entering the cord. The DC decussate at the level of the medulla.

K25928_C001.indd 13

10/01/2017 10:54

14

Chapter 1 Introduction to the Nervous System

Figure 1.6 Somatotopy of the Central Nervous System

K25928_C001.indd 14

10/01/2017 10:54

Somatotopy of the Central Nervous System

15

Unlike the PNS, the CNS displays an important property called somatotopy. This is a fundamental, defining characteristic of the CNS and its importance cannot be overstated. Somatotopy is the point mapping of body part to a precise neuroanatomical area. Figure 1.6 shows a distorted person, called a homunculus, who appears to be lying on top of the cortex. The homunculus is a pictorial representation that shows which neuroanatomical area controls what part of the body. For example, we see that the lateral part of the motor cortex innervates the face and mouth, whereas the medial part innervates the leg. Note that somatotopic representation is proportional; as the more precise control needed over a part of the body, the more neuroanatomical area is required. For example, the hand, with its fine, highly evolved, intricate movements, requires more neuroanatomical area than the foot, with its relatively primitive, easy to perform movements. This is also true between individuals. Anatomical studies show, for example, that musicians, with the need for extremely precise dexterity, will have a larger section of the brain devoted to finger and hand control then their nonmusician equivalents. Figure  1.6 shows that somatotopy continues as we move down the CNS. The internal capsule is an important area of white matter; once again parts of the body are mapped to precise, defined, neuroanatomical areas. The same is true of the spinal cord. Another way to think of it is that the CNS is analogous to a computer. Each specific motor or sensory function (equivalent to a computer program) is stored in a specific, defined neuroanatomical area (equivalent to an area of memory). Damage to one area of memory will cause the corresponding program to be unable to run. Neuroanatomical damage will cause symptoms in the corresponding somatotopic area of the body. This is why understanding the somatotopy of the CNS at each level is so important. As mentioned, somatotopy does not exist in the PNS. If one were to examine a cross section of a nerve, there would be no 1:1 mapping of body function to neuroanatomical area. We will revisit the concept of somatotopy as we explore each part of the CNS.

Take Home Message The CNS exhibits somatotopy. All parts of the body are mapped 1:1 to a distinct and precise neuroanatomical area.

K25928_C001.indd 15

10/01/2017 10:54

16

Chapter 1 Introduction to the Nervous System

Figure 1.7 Arterial Supply to the Central Nervous System

ACA: anterior cerebral artery; ACOM: anterior communicating artery; AICA: anterior inferior cerebellar artery; ICA: internal carotid arteries; MCA: middle cerebral artery; PCA: posterior cerebral arteries; PCOM: posterior communicating arteries; artery; SCA: superior cerebellar artery.

K25928_C001.indd 16

10/01/2017 10:54

Arterial Supply to the Central Nervous System

17

Two distinct circulations, the anterior circulation and the posterior circulation, supply blood to the brain. The anterior circulation is ultimately derived from the internal carotid arteries (ICA), and supplies the anterior two-thirds of the brain. The posterior circulation is ultimately derived from the vertebral arteries, and supplies the posterior one-third of the brain, including the brainstem and cerebellum. The carotid arteries branch from the aorta itself, whereas the vertebral arteries come off the subclavian artery. The common carotid artery divides into the external carotid artery, which supplies the face, and the ICA. The ICA travels into the skull and, once it reaches the base of the brain, bifurcates into the anterior cerebral artery (ACA), and the middle cerebral artery (MCA). The ACA supplies the medial aspect of the cerebral hemispheres while the MCA supplies the lateral aspect. The vertebral arteries travel up into the head through the foramen of the vertebral bones. Once they have passed through the skull they unite to form the basilar artery. Before they do so, however, they each give off a branch called the posterior inferior cerebellar artery (PICA). The basilar artery gives rise to two other cerebellar arteries: the superior cerebellar artery (SCA), and the anterior inferior cerebellar artery (AICA). The cerebellar arteries supply the brainstem, and cerebellum. Finally, the basilar bifurcates into two posterior cerebral arteries (PCA). The PCA supplies the occipital lobe and a small part of the midbrain. If we had the arterial system just described, strokes and other vascular events would be much more common since each part of the brain has its own, sole, independent vascular supply. One single artery (left ICA, right ICA, or basilar) would supply one single part of the brain. However, we have evolved “backup collaterals” that tie the blood supply of the left hemisphere and right hemisphere to each other, as well as tying the anterior and posterior circulations together. This is achieved through “communicating arteries,” and include the anterior communicating artery (ACOM), which link the two ACA together, thus joining the left and right circulation together, and the posterior communicating arteries (PCOM), which join the MCA to the PCA, effectively linking the anterior and posterior circulation together. The communicating arteries help form a continuous circle of circulation that supplies the entire brain, called the Circle of Willis after the physician that first described it.

Take Home Messages The common carotid artery becomes the ICA which bifurcates into the ACA and MCA. Two vertebral arteries unite to become the basilar artery. The basilar artery gives off AICA and SCA branches and bifurcates to become two PCA arteries. The ACOM joins the left and right circulation together, and the PCOM joins the anterior and posterior circulation together to form the Circle of Willis.

K25928_C001.indd 17

10/01/2017 10:54

18

Chapter 1 Introduction to the Nervous System

Figure 1.8 The Meninges

CSF: cerebrospinal fluid. Top Figure: Adapted from Blumenfeld H. Neuroanatomy Through Clinical Cases. Sinauer Associates Inc, Sunderland, 2002. Bottom Figure: Redrawn with permission from Waxman SG. Clinical Neuroanatomy. Lange, New York, 2013.

K25928_C001.indd 18

10/01/2017 10:54

The Meninges

19

The CNS is surrounded by several layers of tissue, collectively called the meninges, that separate it from the skull bone. The deepest layer of tissue, the pia mater (Latin for “tender mother”), directly covers the brain and spinal cord. The next layer is called the arachnoid mater, for its spider-like appearance. The area in between the arachnoid mater and the pia mater is called the subarachnoid space. The blood vessels of the Circle of Willis lie in the subarachnoid space. Cerebrospinal fluid (CSF), which acts as a shock absorber for the brain, also fills the subarachnoid space. CSF is produced by the choroid plexus in the ventricles and is eventually absorbed by the arachnoid granulations in the subarachnoid space. The outermost layer of tissue is called the dura mater (Latin for “tough mother”) and lies on top of the arachnoid mater and directly below the bone of the skull. It is actually composed of two layers on top of each other; meningeal dura mater, which lies directly on top of the arachnoid mater, and the periosteal dura mater, which abuts the skull bone. Two potential spaces exist: the epidural space, which is the area between the periosteal dura and the skull, and the subdural space, between the meningeal dura and the arachnoid mater. They are called potential spaces because they normally should not exist but, as we will see, can be filled with pathologic material, such as blood or pus, and become realized. Note also that at times the periosteal dura is not always in direct contact with the meningeal dura, which creates a channel. These channels are referred to as venous sinuses and help to drain venous blood from the brain. We will discuss them further in Chapter 2.

Take Home Messages Working our way in from the skin, the layers encountered on the way to the brain are: • Skin; • Skull bone; • Epidural space (potential space); • Periosteal dura mater; • Meningeal dura mater; • Subdural space (potential space); • Arachnoid mater; • Subarachnoid space (containing blood vessels bathed in CSF); • Pia mater; • Brain tissue.

K25928_C001.indd 19

10/01/2017 10:54

20

Chapter 1 Introduction to the Nervous System

Figure 1.9 Overview of the Autonomic Nervous System

CN: cranial nerve. Adapted from Freeman S, et al. Biological Science. Pearson, Boston, 2013.

K25928_C001.indd 20

10/01/2017 10:54

Overview of the Autonomic Nervous System

21

Figure 1.9 displays an overview of the autonomic nervous system, which you’ll recall is divided into the sympathetic nervous system and parasympathetic nervous system. Both systems use two neurons. The first neuron lies in the CNS and is called the preganglionic neuron. It synapses in various ganglia (which, remember, are neurons whose cell bodies are located in the PNS) with the postganglionic neuron. We’ll begin with the parasympathetic system, which, remember, acts to conserve energy and anabolically build tissue. The nuclei for preganglionic neurons arise from two locations: the brainstem (specifically, CN III, CN VII, CN IX and CN X) and the sacral part of the spinal cord (specifically, spinal levels S2–S4). These neurons travel into the periphery where they synapse with their ganglia, and the postganglionic neurons continue into their effector organs. It’s important to note that the parasympathetic ganglia are located very close to the organs they innervate, in contrast to the sympathetic system, as we’ll see now. The sympathetic nervous system acts to mobilize energy for survival purposes. The nuclei for preganglionic neurons of the sympathetic system are housed in spinal cord levels T2–L3. The preganglionic neuron has two sets of ganglia it can synapse with. The first is located in the sympathetic chain, which is a bundle of neural tissue that lies just adjacent to the spinal cord. The second set is located at various points throughout the body. Each preganglionic neuron synapses with only one postganglionic neuron, whether it is located in the sympathetic chain or elsewhere.

Take Home Messages The autonomic nervous system employs two neurons: the preganglionic neuron and the postganglionic neuron. The parasympathetic system conserves energy for growth. The nuclei of the preganglionic neurons originate in the brainstem or sacral spinal cord. The postganglionic neurons are near their effector organs. The sympathetic system uses energy for survival. The nuclei of the preganglionic neurons originate in the spinal cord from levels T2 to L3. Its postganglionic neurons begin either in the sympathetic trunk or at other various ganglia.

K25928_C001.indd 21

10/01/2017 10:54

Chapter

2

The Cortex and Associated Structures

Immediate Localization – Higher Functions As we will see, the patterns of motor and sensory dysfunction produced by lesions of the cortex are quite diverse, but typically involve one side of the body (face, arm, and leg). However, problems of any higher cognitive functions can only be produced by lesions of the cortex. These Higher Functions include: • Aphasia – the inability to perceive and use language • Agnosia – the inability to interpret sensory information to recognize objects • Apraxia – the inability to perform learned tasks

K25928_C002.indd 23

10/01/2017 10:53

24

Chapter 2 The Cortex and Associated Structures

Figure 2.1 Specialized Areas of the Cortex

K25928_C002.indd 24

10/01/2017 10:53

Specialized Areas of the Cortex

25

Earlier we saw that the cerebrum can be divided into four lobes: frontal, parietal, temporal and occipital. We also saw that the motor cortex is located in the precentral gyrus, and the sensory cortex is located in the postcentral gyrus. There are several other important functional areas of cortex and we will cover them now. Just anterior to the motor cortex lies the premotor cortex and the supplementary motor area. These structures are involved in the planning of complex movements, such writing, or playing the piano, before they are initiated by the motor cortex. Anterior to the supplementary motor area is the frontal eye field (FEF). The FEF controls voluntary eye movement in the horizontal plane. Activation of the FEF in one hemisphere drives the eyes in the contralateral direction; the left FEF will drive the eyes to the right and vice versa. On the other side of the central sulcus lies the sensory association cortex. This area of the brain integrates various sensory information to make sense of the object providing the stimulus. For example, it determines that the small, round, cold object that you are holding in your hand likely represents a coin. Near the anterior portion of the frontal lobe lies Broca’s area. This part of the brain produces language; written or verbal. Note that Broca’s area lies right beside the part of the motor cortex that controls lips, tongue and the larynx. Wernicke’s area resides in the temporal lobe and is the part of the brain responsible for language comprehension. Note its proximity to the primary auditory cortex, which receives and processes auditory information. On the medial side of the hemispheres lies the cingulate gyrus, which plays an important role in memory and behavior. Recall from Chapter 1 that the motor and sensory cortices are somatotopically organized according the homunculi shown in Fig. 2.1. The homunculi extend to the medial aspect of the hemispheres, which is a fact of great importance, as we will soon see.

Take Home Messages The premotor cortex is located adjacent to the motor cortex and plans complex movement. The FEF drive the eyes to the contralateral side. The sensory association cortex makes 3D sense of sensory input from the sensory cortex. Wernicke’s area is responsible for the understanding of language, and Broca’s area is responsible for generating language.

K25928_C002.indd 25

10/01/2017 10:53

26

Chapter 2 The Cortex and Associated Structures

Figure 2.2 The Basal Ganglia and Associated Structures

K25928_C002.indd 26

10/01/2017 10:53

The Basal Ganglia and Associated Structures

27

The basal ganglia are a group of gray matter structures located near the base of the cerebrum*. They can be thought of as gatekeepers, modulating motor and sensory signals sent back and forth between the rest of the cerebrum and the brainstem and spinal cord. The main components of the basal ganglia are the caudate, putamen, globus pallidus, and substantia nigra. Knowing the functions of each component is neither practical nor needed at this stage in learning. Dysfunction of the basal ganglia as a whole can give rise to a variety of movement disorders, by far the most common being Parkinson’s disease. The egg shaped structure that lies posterolateral to the basal ganglia is the thalamus, which we explore shortly. Several white matter tracts separate the basal ganglia from each other. Framing the basal ganglia at the anterior and posterior poles are the genu and splenium of the corpus callosum, respectively. These tracts serve to connect the left hemisphere and right hemisphere. In between the basal ganglia lies the internal capsule, which is an important white matter tract that carries both descending motor neurons from the cortex, and ascending sensory neurons from the brainstem and spinal cord. The insula lies just lateral to the putamen, and is a part of the cortex that is buried at the base of the Sylvian fissure. The functions of the insula are not completely understood, but it is believed to play a role in autonomic function and emotional regulation. * Note: If you’ve been paying attention, the opening sentence of this page will give you pause; ganglia are a group of neuron cell bodies in the peripheral nervous system (PNS). A group of nerve cell bodies in the central nervous system (CNS) are called nuclei. Well right you are, and the basal ganglia should really be referred to as the basal nuclei, but for historical reasons the name basal ganglia sticks.

Take Home Messages The basal ganglia are involved in helping make sure that motion is smooth and regulated. The internal capsule is a white matter tract that conveys all motor neurons from motor cortex as well as ascending sensory neurons from the brainstem and spinal cord.

K25928_C002.indd 27

10/01/2017 10:53

28

Chapter 2 The Cortex and Associated Structures

Figure 2.3 Nuclei of the Thalamus

LGN: lateral geniculate body; MGN: medial geniculate nucleus; VA: ventral anterior nucleus; VLN: ventral lateral nucleus; VPL: ventral posterior lateral nucleus; VPM: ventral posteromedial nucleus.

K25928_C002.indd 28

10/01/2017 10:53

Nuclei of the Thalamus

29

The thalamus can be thought of as a large relay center made up of many different nuclei. Not all of these nuclei are completely understood. We have listed some of the most relevant and well understood ones in Fig. 2.3, but a complete discussion of the thalamus is beyond this text. There are many different ways to classify the nuclei of the thalamus and this diversity is a frequent source of frustration for students. One of the simplest methods is to group them according to anatomical region. Examining Fig. 2.3 you will see that a “Y-shaped” white matter tract called the internal medullary lamina separates the thalamus into three anatomical regions; anterior, medial and lateral. There are also nuclei, called the intralaminar nuclei, embedded in the internal medullary lamina, which is sometimes referred to as the fourth anatomical region. Functionally, the nuclei of the thalamus can be classified into two groups: specific nuclei, which project to one single, precise neuroanatomical area and nonspecific nuclei, which have projections to many different areas. All of the specific nuclei are located in the lateral section of the thalamus. (These nuclei tend to pop up on exams and should be committed to memory.) Nuclei in the anterior and medial section are involved in processing both emotion and memory. The intralaminar nuclei are responsible for regulating the sleep– wake cycle and plays a critical role in consciousness. Bilateral lesions of the thalamus involving the intralaminar nuclei often result in coma. Two very important nuclei include the ventral posterior lateral nucleus (VPL nucleus) and the ventral posteromedial nucleus (VPM nucleus). These nuclei receive neurons from the spinothalamic tract (ST) and dorsal columns (DC). The VPL is the relay site for sensory neurons from the body (arms and legs) and the VPM is the relay site for sensory neurons from the face.

Take Home Messages Anatomically, the thalamus is divided into four regions: anterior, medial, lateral, and intralaminar. A white matter tract called the internal medullary lamina divides the thalamus into the above regions. Functionally, the thalamic nuclei can be classified into specific and nonspecific. The intralaminar nuclei play a critical role in consciousness.

K25928_C002.indd 29

10/01/2017 10:53

30

Chapter 2 The Cortex and Associated Structures

Figure 2.4 The Internal Capsule

LMN: lower motor neuron; UMN: upper motor neuron. Redrawn with permission from Waxman SG. Clinical Neuroanatomy. Lange, New York, 2013.

K25928_C002.indd 30

10/01/2017 10:53

The Internal Capsule

31

As we mentioned, the internal capsule is a white matter tract that cuts through the center of the basal ganglia. Descending neurons from the motor cortex and ascending sensory neurons from the brainstem and spinal cord both pass through this small area. Given the extreme density of the neuroanatomical real estate found here, small lesions, such as strokes, can have devastatingly large effects. The internal capsule is divided into three sections. The anterior limb sits between the putamen and caudate. It carries fibers running from the thalamus to cortex, the thalamocortical fibers, as well as fibers back from the cortex to the thalamus, the corticothalamic fibers. These fibers carry various types of information. The posterior limb of the internal capsule lies between the putamen and thalamus. The first part of the posterior limb carries the arm and leg neurons of the corticospinal tract. The second part of the posterior limb carries sensory neurons from the VPM and VPL nuclei of the thalamus, en route to the sensory cortex. Note that the somatotopy of the sensory information is identical to its motor counterpart, repeating the same “face, arm, leg” pattern. In between the anterior and posterior limb lies the genu, which represents the “bend,” or “knee” of the internal capsule. The motor neurons destined for the face travel here and continue on in the corticobulbar tract. The corticobulbar tract is a white matter tract that carries neurons from the motor cortex to the CN that innervate the face; it is the facial equivalent of the corticospinal tract.

Take Home Messages The components of the internal capsule are as follows: • Anterior limb: thalamocortical fibers and corticothalamic fibers. • Genu: motor neurons to the face (travelling in the corticobulbar tract). • Posterior limb motor neurons to the arm and leg (travelling in the corticospinal tract), followed by sensory neurons from face, arm and leg.

K25928_C002.indd 31

10/01/2017 10:53

32

Chapter 2 The Cortex and Associated Structures

Figure 2.5 The Motor Pathways

K25928_C002.indd 32

10/01/2017 10:53

The Motor Pathways

33

We are now in a position to trace out the motor and sensory pathways in the CNS, with an emphasis on the somatotopic arrangement at several keys levels. In doing so, we will continue to employ our “face”, “arm” and “leg” nomenclature, which refers to either a) motor neurons travelling to these areas, or b) sensory neurons travelling from these areas. We will begin with the motor system. As we just learned, the motor system is subdivided into a corticobulbar tract, which innervates the muscles of the face via the brainstem and a corticospinal tract, which innervates the muscles of the rest of the body via the spinal cord. The tracts begin in the motor cortex and descend through the white matter. Initially, the neurons were oriented in the medial–lateral (M – L) plane, with face neurons lateral and leg neurons medial. As the tracts enter the internal capsule, however they twist 90° to run in the anterior–posterior (A – P) direction. Now face neurons are anterior and leg neurons are posterior, as we saw in Fig. 2.4. They continue to twist another 90° as they enter the midbrain such that they again run in the M – L direction, however now the face neurons are the most medial and leg neurons are the most lateral. As we descend through the pons and medulla, the upper motor neurons (UMN) of the corticobulbar tract exit and synapse with the lower motor neuron (LMN) nuclei in the brainstem. At the level of the medulla the corticospinal tract both decussates and twists 180°, so that the somatotopic arrangement is maintained. Prior to decussation leg neurons were lateral and arm neurons were medial; by both decussating and twisting, the leg remains lateral, and the arm remains medial, but on the opposing side. The UMNs of the corticospinal tract continue down the lateral part of the spinal cord. At the level of the cervical spine, arm neurons synapse with their LMN in the anterior horn cell (AHC). Leg neurons continue down the cord and eventually synapse with their LMNs at the level of the lumbar spine.

Take Home Messages The motor system is made up of two tracts: the corticobulbar tract and the corticospinal tract. Just before entering the internal capsule, motor neurons twist 90° to run in the A – P direction. They then twist another 90° just before entering the midbrain, to run in the M – L direction. Just before entering the cervical cord, motor neurons decussate and twist 180°.

K25928_C002.indd 33

10/01/2017 10:53

34

Chapter 2 The Cortex and Associated Structures

Figure 2.6 The Sensory Pathways

K25928_C002.indd 34

10/01/2017 10:54

The Sensory Pathways

35

We’ll now turn our attention to the ST, which carries pain and temperature, and the DC, which carries proprioception and vibration. Remember, once the DC synapses with its 2nd order neuron, it is renamed the medial lemniscus. Let’s begin with the ST tract. Neurons from the leg enter the lumbar cord. They decussate immediately and enter the contralateral ST, which ascends up the cord. At the cervical level it is joined by neurons from the arm, which travel medial to neurons from the leg. In the pons, neurons from the face join the ST tract and now occupy the most medial spot. The neurons travel up into the VPM (facial neurons) and VPL (arm and leg neurons) nuclei of the thalamus, where they synapse with their 3rd order neurons. We’ll now move onto the DC pathway. Neurons from the leg enter the lumbar cord and travel up the ipsilateral DC. At the cervical level it is joined with neurons from the arm, which travel lateral to neurons from the leg, as shown in Fig. 2.6. At the level of the medulla, the neurons decussate without twisting, making the leg neurons lateral and arm neurons medial. As they travel up through the pons, they are joined by neurons from the face, which now occupy the most medial spot. They continue into the VPM and VPL of the thalamus, where they synapse with their 3rd order neurons. Note that as the medial lemniscus ascends up the brainstem it moves laterally until it merges with the spinothalamic tract at the level of the midbrain. Beyond this point, both tracts share the same somatotopy; face neurons are medial and leg neurons are lateral. The two tracts continue into the thalamus, where they have neighboring synapses. The paths of the 3rd order neurons for both tracts are identical. They leave the thalamus, twist 90° to run in the A – P direction and travel in the posterior limb of the internal capsule, as shown in Fig. 2.4. They then twist another 90° to run in the M – L direction, but now face neurons are lateral and leg neurons are medial. The tracts then synapse in the sensory cortex.

Take Home Messages The ST tract decussates upon entering the cord; neurons are added medially as the tract travels into the brain. The DC initially have the opposite somatotopy; neurons are added laterally. However, once the DC becomes the medial lemniscus, the neurons are added medially, as in the ST. 3rd order neurons for both tracts share the same path and somatotopy.

K25928_C002.indd 35

10/01/2017 10:54

36

Chapter 2 The Cortex and Associated Structures

Figure 2.7 The Language Circuit

AF: arcuate fasciculus B: Broca’s area; C: conception tract; W: Wernicke’s area.

K25928_C002.indd 36

10/01/2017 10:54

The Language Circuit

37

Having reviewed the motor and sensory systems, we will now study the function of several key areas of the cerebrum. We will begin with one of the most fundamental, the language circuit. By definition, the language circuit of the brain is located in the dominant hemisphere. There are many different models for language, but the best one is the Wernicke– Lichtheim model, shown in Fig. 2.7. Broca’s area, represented as B, is responsible for language production, and Wernicke’s area, W, is responsible for the reception and understanding of language. They are connected directly together by another tract, called the arcuate fasciculus (AF); this tract allows for repetition. Wernicke’s area understands the command to repeat, and sends the signal through the AF to Broca’s area. The anatomical location of Broca’s area, Wernicke’s area and the AF are known and shown in the second part of Fig. 2.7. The AF is a direct connection between Wernicke’s area and Broca’s area. As such, it does not allow for any thinking about or processing of language received by Wernicke’s area, nor does it allow for the creation of new language. Another tract must exist. The exact anatomical location of this “conception tract,” called C, is unknown. It is theoretical and used by the Wernicke–Lichtheim model to illustrate the different types of language problems, which we will discuss in a moment. Aphasia is defined as the inability to comprehend and/or express language. It can sometimes be confused with dysarthria, which is the mispronunciation of words, usually due to facial or mouth weakness. The distinction between the two can be remembered as aphasia is saying incorrect words (language problem), whereas dysarthria is saying words incorrectly (pronunciation problem).

Take Home Messages Wernicke’s area is connected to Broca’s area by the AF, which allows for repetition. Wernicke’s area is connected to Broca’s area by an alternative connection, the conception tract, which allows for processing of received language and the formation of new language. Aphasia is saying incorrect words; dysarthria is saying words incorrectly.

K25928_C002.indd 37

10/01/2017 10:54

38

Chapter 2 The Cortex and Associated Structures

Figure 2.8 The Aphasias

AF: arcuate fasciculus; B: Broca’s area; C: conception tract; TCM: transcortical motor; TCS: transcortical sensory; W: Wernicke’s area.

K25928_C002.indd 38

10/01/2017 10:54

The Aphasias

39

The examination for aphasia evaluates three cardinal abilities. They are: • the ability to understand a vocal or written command • the ability to repeat a sentence • observation of the fluency of the patient’s spontaneous speech. Fluency is notoriously difficult to define, and is best thought of as both the rate and syntax of speech. The lesions shown in Fig.  2.8 produce five distinct aphasias. A Broca’s aphasia (lesion 1) results in a patient with decreased fluency; they struggle even to say a few words, meaningful or not. Language understanding however, is largely intact. They understand the command to repeat a sentence but cannot do so. A Wernicke’s aphasia (lesion 2) results in a patient that does not comprehend language including the command to repeat a sentence. They also do not understand language of their own creation; we think in words and phrases, and patients with Wernicke’s aphasia lose the ability to do this. This extremely frustrating condition results in a speech that is fluent, but is random and nonsensical. A conduction aphasia is the result of a lesion to the AF (lesion 3). The patient can understand language and produce language correctly, but cannot repeat a sentence. A lesion to the conception tract (lesion 4 and 5), results in a transcortical motor (TCM) aphasia or transcortical sensory (TCS) aphasia, respectively. A TCM aphasia impairs fluency, as the patient cannot think about what to say, but the patient is still able to understand and repeat. A TCS aphasia results in a patient who is fluent and can repeat, but is unable to think about the speech they have heard. Note that a TCM aphasia is identical to a Broca’s aphasia except that the TCM aphasia patient retains the ability to repeat. Similarly, a TCS aphasia is identical to a Wernicke’s aphasia except that the TCS aphasia patient retains the ability to repeat. A large insult to the brain can knock out the entire language circuit, resulting in a global aphasia, where the patient cannot understand, repeat and is not fluent.

Take Home Messages Broca’s aphasia patients retain the understanding of language. Wernicke’s aphasia patients retain the fluency of language. A conduction aphasia results in impaired repetition. A TCM aphasia results in impaired fluency but intact comprehension and repetition. A TCS aphasia results in impaired understanding but intact fluency and repetition.

K25928_C002.indd 39

10/01/2017 10:54

40

Chapter 2 The Cortex and Associated Structures

Figure 2.9 Dominant Higher Functions

K25928_C002.indd 40

10/01/2017 10:54

Dominant Higher Functions

41

If you study the cortex closely you will note that each primary sensory modality (auditory cortex, visual cortex, or primary sensory cortex) has beside it an association cortex, as depicted in Fig. 2.9. The association cortex is involved in pattern recognition and allows us to perceive objects, based on the sensory information received by the corresponding primary cortex. For example, the primary sensory association cortex recognizes that the small, cold, irregularly shaped object in your pocket is a key. Difficulties with this perceptive ability is called agnosia. Interestingly, as one studies progressively higher order mammals the association areas take up an increasingly large amount of brain tissue, with humans having the largest area. Tactile agnosia is a failure to recognize objects despite having intact tactile sensation. It is the most common type of agnosia evaluated at the bedside. It is examined by asking the patient to close their eyes and placing an object (usually a coin, paperclip or key) in their hand. They then must identify the object by manipulating it with the fingers alone. The inability to do this is specifically referred to as astereognosis, as seen in Fig.  2.9. Closely related is agraphesthesia, which is the inability to recognize symbols traced on one’s skin (typically a number or a letter traced out on the hand). Visual agnosia is the inability to recognize visual objects, despite having intact primary vision; for example, despite normal vision, a patient cannot recognize simple objects (such as a cup or a pen). An extreme of visual agnosia is prosopagnosia, which is the inability to recognize faces, as was famously popularized in the book by Oliver Sacks, “The Man that Mistook His Wife for a Hat.” Auditory agnosia is the inability to recognize the meaning of sound despite having intact hearing; this can either involve language (hearing words, but not knowing what they mean, called pure word deafness) or nonlanguage (interpretation of simple sound, for example, recognizing the ‘clicking’ of high heeled shoes, or the bang of a gunshot). Another type of agnosia is anosognosia, which is the inability to recognize that one is ill. Despite demonstration of significant neurological deficit, such as having hemi-paresis, the patient states she is perfectly well. Interestingly, depending on the type of agnosia, the lesion can be located in either the dominant or nondominant lobe.

Take Home Messages Lesions to an association area will produce agnosia, which is the inability to integrate sensory information to recognize objects. This can manifest as tactile, visual, or auditory agnosia.

K25928_C002.indd 41

10/01/2017 10:54

42

Chapter 2 The Cortex and Associated Structures

Figure 2.10 Nondominant Higher Functions

K25928_C002.indd 42

10/01/2017 10:54

Nondominant Higher Functions

43

Apraxia can be thought of as the motor equivalent of agnosia. It is the inability to perform learned motor tasks, despite having intact motor power and coordination. Just as with agnosia, there are many different types of apraxia. The most common is ideomotor apraxia, which is difficulty using the limbs for complex tasks. This is tested at the bedside with simple commands, such as “show me how you would salute a soldier”, or “ show me how you would brush your teeth”. While patients retain the motor ability to perform the task, they have lost the cognitive ability to plan out and execute it. Reflexively, they are still able to perform the task; for example, if a soldier were to walk into the exam room, the patient would reflexively salute. This is known as voluntaryautomatic dissociation. Other types of apraxia include geographic apraxia (inability to plan a route to a known destination), buccofacial apraxia (the inability to carry out movements of the face, such as whistling or blowing out a match), and constructional apraxia (inability to draw complex patterns, such as intersecting pentagons). Apraxia is often seen in lesions to the nondominant hemisphere, and has many potential localizations, including premotor cortex, supplementary motor area, corpus callosum and parietal lobe. Another phenomenon that localizes to the nondominant hemisphere is hemi-neglect, which is a defect in the registration of, and attention to, space. Patients have no awareness of anything occurring in that half of space, including themselves. This was demonstrated in an episode of Seinfeld when the character Kramer, after receiving a head injury, did not shave or dress one side, and was unaware that he had failed to do so. Neglect is not due to any sort of primary sensory defect such as vision or proprioception. Patients simply do not process information from that side of the space, do not move limbs on the affected side and may even deny that their body exists on that side. This can be tested for clinically by asking patients to perform the “line bisection task.” Patients are given a full sheet of paper, as in Fig. 2.10, and asked to draw dashes through all the lines they see. Patients with neglect will often demonstrate a definitive and marked inability to fill in the lines beyond midline.

Take Home Messages Apraxia and hemi-neglect are due to lesions of the nondominant hemisphere. The most common type of apraxia is ideomotor, which is a defect in using limbs for complex motor tasks. Hemi-neglect is the inability to process information from one half of the world.

K25928_C002.indd 43

10/01/2017 10:54

44

Chapter 2 The Cortex and Associated Structures

Figure 2.11 The Cerebellum and Ataxia

K25928_C002.indd 44

10/01/2017 10:54

The Cerebellum and Ataxia

45

The cerebellum lies just posterior to the brainstem and is responsible for coordinating all voluntary movement initiated by the motor cortex. Lesions of the cerebellum result in irregular, uncoordinated movements called ataxia. The cerebellum can be divided into three parts. Two hemispheres surround a central structure, called the vermis. As we’ve seen, the CNS is organized somatotopically, and the cerebellum is no exception. Its somatotopy is extremely simple, which is a true rarity for neuroanatomy. The central structure, the vermis, is responsible for coordination of the central part of the body, the trunk. A lesion to the vermis causes truncal ataxia; the patient would involuntarily rock their body back and forth as they constantly try to stabilize themselves about their center of gravity. A lesion to a cerebellar hemisphere results in ataxia of the ipsilateral limb: the cerebellar tracts do not decussate. Testing for limb ataxia is often carried out by having a patient trace out a path. This is depicted in Fig. 2.11; here the patient is asked to run his heel down the contralateral shin (often abbreviated clinically as “heel to shin” testing). Consider for a moment what happens as the patient traces the path. His hip is flexed and externally rotated while his knee is flexed. As he traces out the path he simultaneously extends and internally rotates his hip and extends his knee; the result is smooth, coordinated movement that stays on the target path. It is this simultaneous coordination of muscle movement that is impaired in cerebellar ataxia. A patient with ataxia can only do one muscle movement at once. If observed carefully, you would note the patient extending the hip, then internally rotating it and then finally extending the knee. The patient then must do a series of correction movements in order to realign the foot with its target path and destination. Contrast this with tremor, which is often confused with ataxia. A tremor is a rhythmic oscillation about a central point. Unlike ataxia, a tremor is symmetric about its midpoint. The amplitude of the oscillation can change over the length of the movement, but it remains symmetric. Patients with ataxia of the lower limbs have an unsteady gait. Due to the inability to coordinate muscle movements, when they walk their center of gravity shifts significantly, causing falls to the same side as the lesion. To compensate for this, patients will lower their center of gravity by moving their feet further apart creating a wide based gait.

Take Home Messages The cerebellum coordinates voluntary movement and is made up of a central vermis and two lateral hemispheres. The vermis coordinates the trunk, and each hemisphere coordinates the ipsilateral limbs.

K25928_C002.indd 45

10/01/2017 10:54

46

Chapter 2 The Cortex and Associated Structures

Figure 2.12 The Anterior Circulation

ACA: anterior cerebral artery; ACOM: anterior communicating artery; ICA: internal carotid arteries; MCA: middle cerebral artery; PCA: posterior cerebral arteries; PCOM: posterior communicating arteries. Bottom figure redrawn with permission from Blumenfeld H, Neuroanatomy Through Clinical Cases. Sinauer Associates, Inc, Sunderland, 2002.

K25928_C002.indd 46

10/01/2017 10:54

The Anterior Circulation

47

As we saw in Chapter 1, the internal carotid artery (ICA) supplies the anterior two-thirds of the brain. We will now review the ICA’s path and branches in more detail. The ICA travels up the neck and enters the petrous bone of the skull through the carotid canal. It then travels through a structure called the cavernous sinus. This sinus is actually filled with venous blood and contains several important CN: we will revisit it in Chapter 3. Just before the ICA bifurcates into the anterior cerebral artery (ACA) and the middle cerebral artery (MCA), it gives off two important branches. The first is the ophthalmic artery, which supplies many structures of the eye, including the retina. The second is the anterior choroidal artery, which travels posteriorly and supplies part of the basal ganglia. The ACA continues anteriorly between the cerebral hemispheres and bifurcates into the callosomarginal artery and the pericallosal artery. These arteries continue posteriorly and supply roughly two-thirds of the medial aspect of the hemisphere. Remember, the two individual ACA are connected to each other through the anterior communicating artery (ACOM), which provides emergency collateral flow. The MCA continues laterally to the Sylvian fissure. Along its way it gives off many little arteries that supply the basal ganglia and internal capsule: these are collectively called the lenticulostriate arteries. At the Sylvian fissure the MCA bifurcates into a superior branch and an inferior branch. Overall, the MCA supplies most of the temporal lobe, and also the lateral parts of the parietal and frontal lobes.

Take Home Messages After the ICA passes through the cavernous sinus, it gives off two important branches, the ophthalmologic artery and the anterior choroidal artery, before bifurcating into the ACA and MCA. The ACA travels in between the hemispheres. The MCA travels laterally and gives off tiny perforating branches called the lenticulostriates arteries, before bifurcating at the Sylvian fissure.

K25928_C002.indd 47

10/01/2017 10:54

48

Chapter 2 The Cortex and Associated Structures

Figure 2.13 The Posterior Circulation

AICA: anterior inferior cerebellar artery; CN: cranial nerve; PCA: posterior cerebral arteries; PICA: posterior inferior cerebellar artery; SCA: superior cerebellar artery. Bottom figure redrawn with permission from Blumenfeld H, Neuroanatomy Through Clinical Cases. Sinauer Associates, Inc, Sunderland, 2002. (Figure 14.17b).

K25928_C002.indd 48

10/01/2017 10:54

The Posterior Circulation

49

The posterior circulation of the brain originates from the vertebral arteries. They travel up to the medulla and once they reach the pons, unite to form the basilar artery. However, just before doing so, they each give off a branch that merges to become the anterior spinal artery, which travels back down the body to supply the anterior portion of the spinal cord, as we will see in Chapter 4. As the basilar artery travels up the belly of the pons, it gives off many small arteries, simply called the basilar perforators. These small arteries are analogous to the lenticulostriates of the MCA. Despite their name, the superior cerebellar artery (SCA), the anterior inferior cerebellar artery (AICA) and the posterior inferior cerebellar artery (PICA) supply both the cerebellum as well as part of the brainstem. The SCA and AICA come off the basilar artery. The PICA comes off the basilar in about 50% of individuals; in the other half it comes off the vertebral artery. Finally, just over the midbrain, the basilar artery terminates and branches into the posterior cerebral artery (PCA). The PCA helps supply circulation to the entirety of the occipital lobe and the inferior part of the temporal lobe. It also supplies most of the midbrain. The PCA is joined to the anterior circulation by the posterior communicating arteries (PCOM), completing the Circle of Willis. Note the close proximity between the PCA and CN III. This is important because the first sign of a ruptured aneurysm of the PCOM is usually dysfunction of CN III, as we will examine again in the next chapter.

Take Home Messages At the brainstem, the vertebral arteries unite to form the basilar artery, which eventually bifurcates to become the PCAs. Important branches from the basilar include the basilar perforators, and the SCA, AICA and PICA (in 50% of individuals the PICA comes off the vertebral arteries). The posterior circulation is joined to the anterior circulation by the PCOM.

K25928_C002.indd 49

10/01/2017 10:54

50

Chapter 2 The Cortex and Associated Structures

Figure 2.14 Vascular Territories of the Brain

ACA: anterior cerebral artery; MCA: middle cerebral artery; PCA: posterior cerebral arteries.

K25928_C002.indd 50

10/01/2017 10:54

Vascular Territories of the Brain

51

Knowledge of the vascular supply of the brain is critical to predicting and understanding cerebral strokes and hemorrhages. We’ll now examine Fig. 2.14 in detail. Beginning with the ACA, we can see it supplies the medial two-thirds of each hemisphere. Somatotopically, this includes the leg, bowel and bladder area of both the motor and sensory cortices. Thus, a stroke involving the ACA would only produce weakness and numbness of leg. The ACA also supplies the genu of the corpus callosum, which connects the two hemispheres. The MCA supplies a tremendous amount, including a large part of the motor and sensory cortices. What would happen if the MCA were to be occluded? Now you might be tempted to examine the homunculus and see that the face and arm area of the cortex would be affected. That is true. But note also that the MCA supplies the subcortical white matter tracts that funnel into the internal capsule, including those from the area of cortex supplied by the ACA. Thus, an MCA stroke will produce whole body weakness on one side, but through two different mechanisms. Face and arm weakness is produced by infarction of the gray matter, and leg weakness is caused by infarction of the subcortical white matter. White matter is more robust than gray matter so strokes affecting the white matter are typically less severe and carry a better prognosis. Thus, in MCA strokes, leg weakness is typically less severe than face/arm weakness, and recovers better. The PCA supplies part of the temporal lobe and the majority of the occipital lobe. As we will see in Chapter 3, PCA strokes will cause characteristic patterns of visual loss. While the ACA, MCA and PCA all contribute branches to supply the components of the basal ganglia, the most important branch is the anterior choroidal artery because it supplies the posterior limb of the internal capsule, which, you’ll recall from Fig. 2.4, acts as a funnel for all motor and sensory neurons. We will examine the brainstem vascular territories in the next chapter once we get a better sense of the CN and their nuclei.

Take Home Messages ACA territory: medial aspect of each hemisphere, including the leg, bowel and bladder areas of the motor and sensory cortex. MCA territory: arm and face area of the motor and sensory cortex, as well as virtually all the subcortical white matter. PCA territory: visual cortex. Anterior Choroidal territory: posterior limb of the internal capsule, which is a narrow funnel for all motor and sensory neurons.

K25928_C002.indd 51

10/01/2017 10:54

52

Chapter 2 The Cortex and Associated Structures

Figure 2.15 Venous Drainage of the Head

K25928_C002.indd 52

10/01/2017 10:54

Venous Drainage of the Head

53

The venous system of the brain is actually composed of more than just veins. In addition to regular veins, the brain also contains venous sinuses. As we saw in Fig.  1.8, these channels are created when the periosteal dura separates from the meningeal dura and are filled with venous blood. The size and placement of cortical veins differs greatly between individuals but the sinus formation is usually quite consistent. Typically, cortical veins are very small and superficial, and usually drain into the larger sinuses before ultimately leaving the skull via the internal jugular vein. The superior sagittal sinus runs along the entire length of the skull, and drains the veins that lie in the superficial aspect of the cortex. Inferior to this lies the inferior sagittal sinus and the Great vein of Galen. Both work together to drain the basal ganglia and midbrain. The Great vein of Galen and the inferior sagittal sinus join together to form the straight sinus. The superior sagittal sinus, straight sinus, and the two transverse sinuses (which run along the base of the skull) make up the confluence of sinuses. The internal jugular veins are the ultimate destination of all venous blood in the head. They are composed of venous blood from the transverse sinus and also the cavernous sinus. Remember, the cavernous sinus is unique as it contains not only venous blood, but also a small portion of the ICA, as well as several CN, which we’ll re-examine in Chapter 3.

Take Home Message Venous drainage of the brain is carried out by both veins and sinuses (which are channels created by the separation of the layers of dura).

Suggestions for further reading: Blumenfeld H. Neuroanatomy Through Clinical Cases. Sinauer Associates, Inc, Sunderland, 2002. Lindsay, KW, Bone I, and Fuller G. Neurology and Neurosurgery Illustrated. Churchill Livingstone: Elsevier, Edinburgh, 2010.

K25928_C002.indd 53

10/01/2017 10:54

Chapter

3

The Cranial Nerves and the Brainstem Immediate Localization Only a lesion in the brainstem can produce the pattern of long tract dysfunction known as Crossed Signs. Crossed Signs – refers to a loss of long tract function on the ipsilateral side of the face, and the contralateral side of the body. This occurs in two different patterns. If the lesion involves the lateral part of the brainstem, the patient has ipsilateral loss of pain and temperature in the face and contralateral loss in the body. If the lesion involves the medial brainstem, the patient will have ipsilateral loss of motor function in the face, and contralateral loss of motor function and vibration/proprioception in the body. The important point is that patients experience ipsilateral loss in the face, and contralateral loss in the body.

K25928_C003.indd 55

12/01/2017 11:41

56 Chapter 3 The Cranial Nerves and the Brainstem

Figure 3.1 Overview of the Cranial Nerves

K25928_C003.indd 56

CN: cranial nerve. 12/01/2017 11:41

Overview of the Cranial Nerves

57

The human head is innervated by 12 cranial nerves* (CN) as shown in Fig. 3.1. In the same way as spinal nerves insert into the spinal cord, CN insert into the brainstem. Cranial nerves can have motor function, sensory function and parasympathetic function, and can thus be composed of motor neurons, sensory neurons and parasympathetic neurons. CN I is responsible for smelling, also called olfaction, and is virtually never tested clinically. CN II transmits visual information to the occipital lobe, where it is processed and interpreted. Together, CN III, CN IV and CN VI control eye movements. CN III also helps to keep the eye open and plays a role in determining pupillary size. CN V provides sensory innervation of the face, and controls muscles involved in chewing, also called mastication. CN VII controls all the other muscles in the face, receives taste from anterior two-thirds of the tongue and innervates many glands in the head. CN VIII is involved in both hearing and balance. CN IX and X both provide sensory and motor innervation to the throat. In addition, CN X provides much of the parasympathetic innervation to the body. CN XI controls turning of the head and shrugging the shoulders. Finally, CN XII controls the tongue. Like their spinal nerve equivalents, a CN is actually composed of thousands of individual neurons. The nuclei for these neurons lie in the brainstem. If a CN is made up of more than one neuron type (motor, sensory or parasympathetic), then it will have more than one nucleus in the brainstem; there is one nucleus for every neuron type that makes up the CN. For example, CN VII has a motor, sensory, and parasympathetic component so there is a motor nucleus for CN VII, which is distinct from the sensory nucleus for CN VII, which is distinct from the parasympathetic nucleus for CN VII. The opposite is not true; a single nucleus can be shared between nerves. For example, CN VII and CN IX both have a sensory component innervating taste buds in the tongue. The nucleus responsible for this in CN VII is the nucleus solitarius. The nucleus responsible for this in CN IX is also the nucleus solitarius. In this way, one nucleus can have input from multiple CN. * Note: CN I and CN II are not actually nerves at all. They are direct extensions of the central nervous system (CNS) itself, and so would more accurately be called tracts. For religious reasons, it was important for the total number of CN to be twelve and historically CN I and CN II have been referred to as such, though it makes little practical difference.

Take Home Messages There are 12 CNs, and the nuclei for each nerve lies in the brainstem. There is one nucleus for each component (motor, sensory or parasympathetic) that the CN carries.

K25928_C003.indd 57

12/01/2017 11:41

58

Chapter 3 The Cranial Nerves and the Brainstem

Figure 3.2 The Exit of the Cranial Nerves from the Brainstem

CN: cranial nerve.

K25928_C003.indd 58

12/01/2017 11:41

The Exit of the Cranial Nerves from the Brainstem

59

Before we get into the details of the CN it is important to have a working knowledge of the brainstem. As we saw in Chapter 1, the brainstem can be divided into three main sections: the midbrain, pons and medulla. Figure 3.2 shows two important white matter connections found in the brainstem; both are referred to as peduncles. The cerebral peduncle connects the brain to the spinal cord, and runs throughout the entire brainstem. They can be viewed directly at the level of the midbrain. The cerebellar peduncle (sometimes divided into superior, middle and inferior sections) connects the spinal cord and brainstem to the cerebellum. One of the many tracts that run through the cerebellar peduncle is the spinocerebellar pathway. It is important to know where each CN leaves the brainstem. All the CN come off the anterior part of the brainstem, with the exception of CN IV. It leaves the posterior midbrain and quickly wraps around the brainstem to become anterior. If you study Fig. 3.2 closely, you will note two very important patterns. First, the CN tend to spread out equally between the midbrain, pons and medulla. The midbrain contains CN I to CN IV, the pons has CN V to CN VIII, and the medulla contains CN IX to CN XII. Second, CN II, CN III, CN VI and CN XII all insert into the brainstem quite medially, whereas all other CN are quite lateral. Remembering these patterns will become very important when we examine the anatomy of the brainstem in cross section.

Take Home Messages The brainstem consists of the midbrain, which houses CN I to CN IV; the pons, which houses CN V to CN VIII; and the medulla, which houses CN IX to CN XII. All CN exit the brainstem anteriorly, except for CN IV. CN II, CN III, CN VI and CN XII exit the brainstem medially. All other CN exit laterally.

K25928_C003.indd 59

12/01/2017 11:41

60

Chapter 3 The Cranial Nerves and the Brainstem

Figure 3.3 Parasympathetic Component of the Cranial Nerves

K25928_C003.indd 60

12/01/2017 11:41

Parasympathetic Component of the Cranial Nerves

61

Before we examine the individual CN, we should take a moment and familiarize ourselves with the parasympathetic innervation they provide. As we have discussed in Chapter 1, all sympathetic innervation originates from the thoracic and lumbar spinal cord. The analogous parasympathetic innervation is transmitted by select CN. As we just learned, the parasympathetic component of these CN will have their own specific nucleus. The following CN have a parasympathetic component: CN III: responsible for pupillary constriction. The associated nucleus is the Edinger– Westphal nucleus in the midbrain. CN VII: responsible for innervating salivary glands as well as lacrimal (tear) glands. The associated nucleus is the superior salivatory nucleus in the pons. CN IX: responsible for innervating the parotid gland, which is the largest salivary gland. The associated nucleus is the inferior salivatory nucleus in the medulla. CN X: responsible for parasympathetic innervation to internal organs of the trunk, including the heart, lungs and digestive tract. The associated nucleus is the unfortunately named dorsal motor nucleus in the medulla. Recall from Chapter 1 that the autonomic system employs two neurons, a preganglionic neuron and a postganglionic neuron. Unlike the sympathetics whose ganglia lie very close to the spinal cord, the parasympathetic ganglia lie very close to the organs they innervate.

Take Home Messages Parasympathetic innervation is transmitted by CN III, CN VII, CN IX and CN X. Each parasympathetic component has its own nucleus, which lies in the brainstem.

K25928_C003.indd 61

12/01/2017 11:41

62

Chapter 3 The Cranial Nerves and the Brainstem

Figure 3.4 The Visual Pathway

K25928_C003.indd 62

12/01/2017 11:41

The Visual Pathway

63

In practice, CN I is rarely tested, so we will not concern ourselves with it. The entire area seen by a pair of stationary eyes is termed the visual field. Each eye independently views the visual field. Contrary to popular belief, the left eye does not receive input solely from the left hemifield, and the right eye does not receive input solely from the right hemifield. Rather, as shown in Fig. 3.4, each eye receives input from both the left and right hemifields. Thus, in Fig. 3.4, EYELeft receives input from the left hemifield (L1 and L2) and from the right side, R1. Similarly, EYERight receives input from R1, R2 and L2. Note that the fields are not equal; EYERight receives information from R2, which EYELeft does not and EYELeft receives information from L1, which EYERight does not. The important point is that each eye receives information from both the left and right hemifield, though the amounts are unequal. Light from the visual field passes through the cornea, then the lens, and finally hits the retina. Retinal cells are activated by light, and transmit an electrical impulse to CN II, the optic nerve. The optic nerve travels posteriorly until it reaches the optic chiasm. A very important decussation of information occurs at the optic chiasm; the fields representing peripheral vision (L1 and R2) cross over at the chiasm, so that the right brain will interpret the entire left visual field (L1 and L2) and the left brain will interpret the entire right visual field (R1 and R2). Beyond the optic chiasm the optic nerve is referred to as the optic tract. The optic tract continues until it reaches the lateral geniculate body (LGN) of the thalamus, which, remember, is a large relay center for sensory information. The fibers terminate, but axons from the LGN continue as the optic radiations, and travel back through brain cortex until they reach the occipital lobe, which is the ultimate destination of visual information.

Take Home Messages Each eye receives input from the left hemifield and the right hemifield. At the optic chiasm, the visual fields representing peripheral vision (L1 and R2) decussate, so that one side of the brain interprets one entire field (i.e., the left interprets L1 and L2). The visual information pathway: È È È È

K25928_C003.indd 63

retina optic nerve optic chiasm optic tract

È È È

lateral geniculate body of thalamus optic radiation occipital lobe.

12/01/2017 11:41

64

Chapter 3 The Cranial Nerves and the Brainstem

Figure 3.5 Visual Field Defects

K25928_C003.indd 64

12/01/2017 11:41

Visual Field Defects

65

Lesions in the visual pathway will give rise to characteristic visual field defects, in which the patient will report a complete loss of vision from the affected aspect. In naming the following defects, many authors utilize “nasal/temporal” nomenclature. They divide each hemifield into a nasal side representing central vision (in our diagram L2 and R1) and a temporal side, named after the temporal bone, representing peripheral vision (in our diagram L1 and R2). This means that information from the temporal aspects decussate in the optic chiasm. The lesion in 1) would cause complete vision loss in a single eye, also called monocular vision loss. Again, note that the patient would still have vision from both the left and right hemifields. A lesion at 2) would interrupt the decussation of the temporal fibers, and would result in a profound loss of peripheral vision. This lesion results in a bitemporal hemianopsia; “bitemporal” as both temporal aspects are involved, “hemi” for half of the field in each eye and “anopsia” for defect in a visual field. A lesion at either 3) or 6) results in a complete loss of vision of either the left or right hemifields, depending on which side of the brain is affected. This field defect is called a homonymous hemianopsia; it is called “homonymous” because the loss in each eye is the same. Because the loss is of an entire hemifield patients often incorrectly report that they have lost vision in an entire eye. The somatotopic arrangement of the visual fields continues in the optic radiations. The more superior optic radiations carry information from the inferior visual field, and vice versa. Thus a lesion at 4) results in a homonymous superior quadrantanopsia and a lesion at 5) results in a homonymous inferior quadrantanopsia.

Take Home Messages The temporal aspects of the visual field decussate in the optic chiasm. A bitemporal hemianopsia results in a profound loss of peripheral vision. A homonymous hemianopsia results in a complete loss of either the left or right hemifield. A homonymous quadrantanopsia results in a partial loss of either the left or right hemifield.

K25928_C003.indd 65

12/01/2017 11:41

66

Chapter 3 The Cranial Nerves and the Brainstem

Figure 3.6 Sympathetic and Parasympathetic Control of Pupillary Size

LGN: lateral geniculate body.

K25928_C003.indd 66

12/01/2017 11:41

Sympathetic and Parasympathetic Control Pupillary Size

67

The autonomic system controls pupillary size. As with everything in the autonomic system, the final outcome is determined by the relative contributions of the sympathetic system, responsible for pupil dilation (easily remembered by reminding yourself that in “fight or flight” situation, you would want the most visual information possible), and the parasympathetic system, responsible for pupillary constriction. Let’s examine the pathways of the two systems, as shown in Fig. 3.6. The sympathetics originate in the hypothalamus and travel down the lateral part of the brainstem before synapsing in the spinal cord and exiting at T1. They then travel up the sympathethetic chain, before synapsing again at the superior cervical ganglion. At this point the sympathetics embed themselves in the outer walls of the internal carotid artery (ICA). They continue up the ICA, through the base of the skull and eventually branch off to innervate the pupillary dilator muscle, as well as Muller’s muscle, which is responsible for eye opening (not shown). In contrast to the rest of neuroanatomy, there is no decussation anywhere along the sympathetic pathway. The pupillary light reflex activates the parasympathetic pathway. Light enters the eye and follows the typical pathway to the LGN as in Fig.  3.4. Just before synapsing at the LGN, a small number of neurons branch off and travel to the pretectal nucleus in the midbrain. The pretectal nucleus sends signals to the Edinger–Westphal nucleus (EWN) of CN III. The signal continues from the EWN to the ciliary ganglion and eventually into the pupillary constrictor muscles. Note that one pretectal nucleus innervates both the ipsilateral and contralateral EWN, explaining why shining light in one eye results in constriction of both pupils.

Take Home Messages Final pupil size is determined by the relative, often unequal, contributions of the sympathetic pathway (pupillary dilatation) and parasympathetic pathway (pupillary constriction). Sympathetic pathway: È È È È È È

K25928_C003.indd 67

hypothalamus brainstem sympathetic chain superior cervical ganglion internal carotid pupillary dilator muscle.

Parasympathetic pathway: È È È È

pretectal nuclei Edinger–Westphal nuclei ciliary ganglion pupillary constrictor muscle.

12/01/2017 11:41

68

Chapter 3 The Cranial Nerves and the Brainstem

Figure 3.7 The Extraocular Muscles

IO: inferior oblique muscle; IR: inferior rectus muscle; SO: superior oblique muscle; SR: superior rectus muscle.

K25928_C003.indd 68

12/01/2017 11:41

The Extraocular Muscles

69

Six extraocular muscles control eye movement. The muscles can be divided into two different groups: the rectus muscles, which insert into eye in a straight path, and the oblique muscles, which insert into the eye on an angled path. Two different muscles control horizontal eye movement. CN VI innervates the lateral rectus, which abducts the eye away from the nose. CN III innervates the medial rectus, which adducts the eye toward the nose. Four different muscles are responsible for vertical eye movement. CN IV innervates the superior oblique muscle and CN III innervates the superior rectus, inferior rectus, and inferior oblique muscles. Vertical eye movements are complicated by two factors. First, different vertical movement muscles will be activated depending on what horizontal position the eye is in (full adduction, full abduction or somewhere in between). We will consider this further in a moment. Second, the actions of the oblique muscles are counterintuitive because they insert behind the plane of the eye. We will consider that now. The eye can be approximated as a sphere rotating about an axis. The plane of the eye (represented by the dashed lines in Fig. 3.7) divides it into two halves. The rectus system inserts in front of the plane of the eye, and its movements are straightforward. Pulling on the muscle, we see that if we pull on the upward portion it moves the eye upward, just as we would expect. Thus the superior rectus moves the eye upward, and the inferior rectus moves the eye downward. However, consider what would happen if the muscle inserted behind the plane of the eye, as in the oblique system. Now, pulling on the upward portion will result in the eye moving downward. Thus, the actions of the oblique group are the opposite of what you would expect; the superior oblique moves the eye downward, and the inferior oblique moves the eye upward. We’ll now turn our attention to understanding which system, rectus or oblique, is activated in different horizontal eye positions.

Take Home Messages

K25928_C003.indd 69

CN III innervates:

CN IV innervates:

Medial rectus – adduction of eye Superior rectus – upward movement Inferior rectus – downward movement Inferior oblique – upward movement

Superior oblique – downward movement CN VI innervates: Lateral rectus – abduction of eye

12/01/2017 11:41

70

Chapter 3 The Cranial Nerves and the Brainstem

Figure 3.8 Movement of the Eyes

K25928_C003.indd 70

12/01/2017 11:41

Movement of the Eyes

71

As mentioned, vertical movements are more complicated because different muscles will be activated depending on the horizontal position of the eye (full adduction, full abduction or somewhere in between). There are two different vertical movement systems: the rectus system, which is activated when the eye is abducted, and the oblique system, which is activated when the eye is adducted. Anything less than full abduction or adduction will result in a partial, but unequal, activation of both systems. Let’s begin with a set of eyes looking to the right. The right eye is abducted and so has its lateral rectus activated (CN VI). The left eye is adducted so has its medial rectus activated (CN III). We will examine upgaze first; if the eye is abducted, the superior rectus will pull the eye upwards, however if the eye is adducted, the inferior oblique muscle will pull the eye upwards. Both of these muscles are innervated by CN III. Now we’ll turn our attention to downgaze; if the eye is abducted, the inferior rectus will pull the eye downwards. If the eye is adducted, the superior oblique will pull it downwards. The inferior oblique is innervated by CN III and the superior oblique is innervated by CN IV. Note that the oblique muscles pull the eye in the opposite direction compared to the their rectus equivalents i.e., the superior rectus pulls the eye upwards, whereas the superior oblique pulls the eye downwards. This is because the oblique muscles wrap around the eye and insert behind the eye’s plane of section compared to the rectus system, which inserts in front of the eye’s plane of section.

Take Home Messages The muscles responsible for vertical movement of the eyes are: • Superior rectus – upward movement in full abduction. • Inferior rectus – downward movement in full abduction. • Inferior oblique – upward movement in full adduction. • Superior oblique – downward movement in full adduction.

K25928_C003.indd 71

12/01/2017 11:41

72

Chapter 3 The Cranial Nerves and the Brainstem

Figure 3.9 The Medial Longitudinal Fasciculus

PPRF: paramedian pontine reticular formation. Redrawn with permission from Greenberg D et al. Clinical Neurology. McGraw– Hill Education, New York, 2015.

K25928_C003.indd 72

12/01/2017 11:41

The Medial Longitudinal Fasciculus

73

Certain pairs of extraocular muscles are yoked together. This means that activation of an extraocular muscle in one eye results in the simultaneous activation of another extraocular muscle in the opposing eye i.e., to look to the right, the lateral rectus must move the right eye, whereas the medial rectus must simultaneously move the left eye. Failure to do so would result in diplopia (double vision), from eyes that are not in perfect alignment. Such eyes are said to be in dysconjugate gaze. The pathway that allows for conjugate eye movement is called the medial longitudinal fasciculus (MLF) and lies in the central part of the midbrain and pons. The MLF is a white matter tract that allows CN nuclei to be connected to each other. It is responsible for both conjugate vertical and conjugate horizontal movements, but the vertical path is quite complex so we will only consider the horizontal one. It begins with the nucleus of CN VI. This nucleus actually does much more than just control CN VI, and is better referred to as the horizontal gaze center because it is responsible for the initiation of horizontal gaze in both eyes. In addition to directly innervating CN VI, it also innervates the contralateral CN III nucleus via the MLF. Thus, activation of the CN VI nucleus will cause simultaneous activation of the ipsilateral lateral rectus and the contralateral medial rectus, resulting in perfectly aligned eye movement and conjugate gaze. As we saw earlier, voluntary control of horizontal eye movement is initiated by the frontal eye field (FEF). How does the FEF, way up in the cerebrum, connect to this pathway in the brainstem? It turns out that the FEF synapses with a contralateral structure in the brainstem called the paramedian pontine reticular formation (PPRF). The PPRF connects the FEF to the CN VI nucleus and in doing so connects voluntary eye movements to the automatic movements initiated by the brainstem.

Take Home Messages Yoked extraocular muscles result in perfectly aligned eye movement and subsequent conjugate gaze. The MLF is a white matter tract in the brainstem that connects the CN VI nucleus to the contralateral CN III nucleus. Activation of the CN VI nucleus will cause contraction of the ipsilateral lateral rectus and, via the MLF, the contralateral medial rectus.

K25928_C003.indd 73

12/01/2017 11:41

74

Chapter 3 The Cranial Nerves and the Brainstem

Figure 3.10 Facial Sensation and the Muscles of Mastication

K25928_C003.indd 74

12/01/2017 11:41

Facial Sensation and the Muscles of Mastication

75

Now that we understand eye movements, we should return to CN V, which we skipped. CN V has both a sensory and motor component. CN V is called the trigeminal nerve because it has three major divisions. This includes the ophthalmic or V1 division, the maxillary or V2 division and the mandibular or V3 division. Their sensory innervation is shown in Fig. 3.10. V1 and V2 are purely sensory divisions, but V3 has a sensory and motor component. CN V leaves the pons and travels into the trigeminal ganglion, located in an area called Meckel’s cave at the base of the skull. The nerve then trifurcates and passes through several openings in the base of the skull. Specifically, the V1 division passes through the superior orbital fissure, the V2 division passes through the foramen rotundum, and the V3 division passes through the foramen ovale. Because CN V has a motor and sensory component, we expect one nucleus for the motor component and another nucleus for the sensory component. The motor nucleus of CN V is plainly named the trigeminal motor nucleus. This nucleus innervates the muscles of mastication (chewing), which include the masseter, temporalis, and the medial and lateral pterygoid muscles. CN V actually has a set of three nuclei for sensory information, and they are located throughout the brainstem. Together they are referred to as the trigeminal sensory nuclei and are each associated with different sensory modalities. They include: • The mesencephalic trigeminal nucleus, located in midbrain. It encodes proprioceptive information from the jaw. • The chief sensory nucleus, located in the central pons. It encodes fine touch and vibration information from the face. • The spinal trigeminal nucleus, located in the medulla. It encodes pain and temperature information from the face.

Take Home Messages CN V has three branches; the V1 (ophthalmic) division, the V2 (maxillary) division and the V3 (mandibular) division. CN V has three sensory nuclei; the spinal trigeminal nucleus, the chief sensory nucleus and the mesencephalic trigeminal nucleus. CN V’s motor component controls the masseter, temporalis, medial pterygoid, and lateral pterygoid, which are the muscles of mastication.

K25928_C003.indd 75

12/01/2017 11:41

76

Chapter 3 The Cranial Nerves and the Brainstem

Figure 3.11 The Cavernous Sinus

K25928_C003.indd 76

12/01/2017 11:41

The Cavernous Sinus

77

Before we continue, we should review the pathway taken by CN II to CN VI, as many of these nerves pass through an area called the cavernous sinus, just before leaving the base of the skull. As we saw in Chapters 1 and 2, a sinus is a large venous channel, created by separation of the periosteal layer of dura mater from the meningeal layer (Fig. 1.8). Most sinuses simply contain venous blood, however the cavernous sinus is a notable exception as it contains not only venous blood, but also arteries and nerves. The cavernous sinus contains the longitudinally running ICA as it ascends to the cortex. Remember at this point that the sympathetics are still attached to the walls of the ICA (Fig. 3.6). In contrast to the longitudinally running ICA, several CN pass through the cavernous sinus transversely, on their way to the base of skull. This includes CN III, CN IV, CN V1, CN V2 and CN VI. These nerves leave the brainstem and enter the subarachnoid space. They then pass over the petrous bone and pierce the dura to enter the cavernous sinus. Most of the nerves lie quite far away from the ICA, with the exception of CN VI. The cavernous sinus is located at the base of the skull, just above the air filled nasal sinuses. Sitting just above the cavernous sinus is the pituitary gland, which is responsible for the production of many hormones. Just superior to the pituitary lies the optic chiasm. If a mass lesion, such as a tumor or ruptured aneurysm, occurs in the cavernous sinus the subsequent swelling can become so great that not only are the CN affected, but also these surrounding structures.

Take Home Messages The components of the cavernous sinus include CN III, CN IV, CN V1, CN V2, CN VI and the ICA (along with the sympathetics). The pituitary gland and optic chiasm lie directly above the cavernous sinus.

K25928_C003.indd 77

12/01/2017 11:41

78

Chapter 3 The Cranial Nerves and the Brainstem

Figure 3.12 Cranial Nerve VII and the Types of Facial Droop

LMN: lower motor neuron; UMN: upper motor neuron.

K25928_C003.indd 78

12/01/2017 11:41

Cranial Nerve VII and the Types of Facial Droop

79

CN VII has a motor component, sensory component and a parasympathetic component. CN VII leaves the pons and enters the base of skull through the internal acoustic meatus (IAM). It continues through the bone of the skull and trifurcates once it reaches the geniculate ganglion. Each branch now leaves the skull through a different opening. The motor component of CN VII begins with the facial nucleus in the pons. It passes through the IAM and leaves the base of skull at the stylomastoid foramen. Here it divides into five branches: the temporal, zygomatic, buccal, mandibular and cervical branch, as in Fig. 3.12. These branches innervate the many muscles of facial expression. Lesions of CN VII will cause the face to droop, as the muscles that maintain resting tone are now weak. The appearance of the facial droop will be quite different depending if the causative lesion is located in the upper motor neuron (UMN), which travels from the motor cortex to synapse with the facial nucleus, or the lower motor neuron (LMN), which travels from the facial nucleus to the muscles. If you look closely at Fig. 3.12, you will notice that the neuron supplying the forehead is dually innervated from both the left and right motor cortex! This dual motor innervation is quite unique and only occurs for one other nerve, CN XII. However, the rest of the face is not dually innervated. Thus, a facial droop from an UMN lesion will spare the forehead (as it would still receive innervation from the other motor cortex), whereas a facial droop from a LMN lesion will cause the entire face to droop. The parasympathetic component of CN VII is controlled by the superior salivatory nucleus in the pons and is responsible for tear and saliva production. It innervates all salivary glands in the head with the exception of the largest gland, the parotid gland, which is controlled by CN IX. Finally, the sensory component of CN VII is controlled by the nucleus solitarius in the medulla, and is responsible for taste from the anterior two-thirds of the tongue.

Take Home Messages CN VII innervates the muscles of facial expression through the temporal, zygomatic, buccal, mandibular and cervical branches. An UMN type facial droop will spare the forehead; a LMN facial droop will involve the entire face. CN VII’s parasympathetic component innervates the lacrimal and salivary glands (except the parotid gland) and its sensory component provides taste to the anterior two-thirds of the tongue.

K25928_C003.indd 79

12/01/2017 11:41

80

Chapter 3 The Cranial Nerves and the Brainstem

Figure 3.13 The Functions of Cranial Nerve VIII and Cranial Nerve IX

K25928_C003.indd 80

12/01/2017 11:41

The Functions of Cranial Nerve VIII and Cranial Nerve IX

81

CN VIII should really be thought of as two separate nerves that travel together for a short time. CN VIII has two nuclei: the vestibular nucleus, responsible for balance, and the cochlear nucleus, responsible for hearing. CN VIII emerges from the brainstem at the interface between the pons and medulla. Along with CN VII it enters the base of the skull at the IAM and then immediately splits into the vestibular nerve and the cochlear nerve. The vestibular nerve innervates the semicircular canals near the inner ear. The semicircular canals work on a similar principle as the bubble level used in carpentry. The bubble tells you the position of the object you place it on; if the object is not level, the bubble is not in the center of the field. In addition, if you were to move that object, the bubble would move, giving you information about the acceleration of the object. The semicircular canals are filled with fluid and, utilizing the same principles, gives proprioceptive information about the position and acceleration of the head. The cochlear nerve innervates the cochlea. When a sound wave enters the ear it reverberates the tympanic membrane, which transmits a signal to the cochlea. The cochlea transduces this signal into a nerve impulse, which travels up the cochlear nerve into the brainstem. CN IX has a motor, sensory and parasympathetic component. CN IX enters the jugular foramen at the base of the skull. Upon reaching the inferior ganglion, it bifurcates. The parasympathetic component travels through the base of the skull as the lesser petrosal nerve and eventually passes through the foramen ovale to innervate the parotid gland, which is its sole function. The parotid gland is the main producer of saliva for the mouth. The parasympathetic nucleus is the inferior salivatory nucleus located in the pons. The sensory and motor components exit the jugular foramen together. The motor component only innervates a single muscle, the Stylopharyngeus, which is involved in talking and swallowing; it is controlled by the nucleus ambiguus in the medulla. CN IX has a grab bag of sensory functions; it provides input to a small portion of the ear, the back of the throat and the posterior one-third of the tongue. These functions are controlled by the nucleus solitarius.

Take Home Messages CN VIII innervates both the semicircular canal, which is involved in sensing the position and motion of the head, and the cochlea, which is involved in hearing. CN IX has a motor, sensory and parasympathetic component.

K25928_C003.indd 81

12/01/2017 11:41

82

Chapter 3 The Cranial Nerves and the Brainstem

Figure 3.14 The Many, Many Functions of Cranial Nerve X

K25928_C003.indd 82

12/01/2017 11:41

The Many, Many Functions of Cranial Nerve X

83

CN X has a motor, sensory and parasympathetic component. CN X enters and exits the base of skull through the jugular foramen. The motor component of CN X is controlled by the nucleus ambiguus in the medulla. It provides innervation to the pharyngeal muscles involved in swallowing, as well as laryngeal muscles for voice production. The sensory component of CN X is controlled by the nucleus solitarius in the medulla. It provides innervation to the taste receptors near the back of the throat. The parasympathetic component of CN X is controlled by the dorsal motor nucleus in the medulla. It provides parasympathetic innervation to nearly all the internal organs (Fig. 1.9). Indeed, the reason CN X is named the “vagus” nerve is after the term “vagabond,” which is a person that wanders from place to place, just as the parasympathetic component wanders from place to place in the body.

Take Home Messages The motor component of CN X controls the pharyngeal and laryngeal muscles. The sensory component is responsible for taste sensation at the back of the throat. The parasympathetic component provides innervation to nearly all of the internal organs.

K25928_C003.indd 83

12/01/2017 11:41

84

Chapter 3 The Cranial Nerves and the Brainstem

Figure 3.15 Motor Innervation to the Neck and the Tongue

LMN: lower motor neuron.

K25928_C003.indd 84

12/01/2017 11:41

Motor Innervation to the Neck and the Tongue

85

Like CN VIII, CN XI is more accurately described as two separate nerves, but for historical reasons has been referred to as a single one. CN XI has a “spinal part,” coming off the high cervical cord, and a “cranial part” coming from the medulla. The two merge very briefly to form CN XI. Soon thereafter the cranial part diverges and joins with CN X. The spinal part exits the base of the skull through the jugular foramen (along with CN IX and CN X) and continues into the muscles of the neck. Because CN XI is really two nerves, it has two sets of nuclei; the cranial component’s nucleus is the nucleus ambiguus in the medulla, and the spinal component’s nucleus originates in the gray matter of C1 to C5 in the spinal cord. CN XI only has a motor component. It provides innervation to the trapezius muscle as well as the sternocleidomastoid (SCM). Activation of the trapezius causes shrugging of the shoulders. Activation of the SCM causes the head to turn to the contralateral side (i.e., the left SCM causes the head to turn to the right). CN XII’s only function is to provide motor innervation to the muscles of the tongue, and is controlled by the hypoglossal nucleus in the medulla. It exits the base of the skull via the hypoglossal canal. The tongue is designed such that the muscles on one side of the tongue push it to the contralateral side as it extends out of the mouth. Straight protrusion of the tongue is caused by equal activation of muscles on both sides. Thus if one side of the tongue is weak, the tongue will deviate to that side, due to unopposed action of the healthy side. Note that, like the forehead component of CN VII, the hypoglossal nucleus receives innervation from both motor cortices. Because of this, UMN lesions of the hypoglossal nerve do not result in any clinically detectable weakness or symptoms. Only LMN type lesions can cause tongue weakness.

Take Home Messages CN XI provides motor innervation to the trapezius, which causes shrugging of the shoulder, and the SCM, which turns the head to the contralateral side. CN XII provides motor innervation to the tongue. Weakness of one side of the tongue, due to a LMN lesion, will cause the tongue to deviate toward the weak side. CN XII receives bilateral UMN innervation. Thus, a single UMN lesion does not have any clinically detectable effect on the tongue.

K25928_C003.indd 85

12/01/2017 11:41

86 Chapter 3 The Cranial Nerves and the Brainstem

Figure 3.16 Exit of the Cranial Nerves through the Base of the Skull

K25928_C003.indd 86

12/01/2017 11:41

Exit of the Cranial Nerves through the Base of the Skull

87

As a summary, and also because this topic tends to pop up in examinations, we have included Fig. 3.16, which shows all of the exit sites of the CN. On the left are the exits of the base of the skull and on the right are the nerves that pass through them.

Take Home Message Really, this is all important stuff.

K25928_C003.indd 87

12/01/2017 11:41

88 Chapter 3 The Cranial Nerves and the Brainstem

Figure 3.17 The Rule of 4

K25928_C003.indd 88

12/01/2017 11:41

The Rule of 4

89

The importance of understanding the cross sectional anatomy of the brainstem cannot be overemphasized. While most students approach the topic with trepidation, a few simple rules will allow you to quickly recreate the anatomy at any point in the brainstem. The brainstem is structured such that the medial components are supplied by a different artery than the lateral components. Thus, the goal is to localize to medial or lateral parts of the midbrain, pons or medulla. As long as we remember the number “4,” we are well on our way to doing this: • 4 CN lie in the midbrain (CN I – CN IV). • 4 CN lie in the pons (CN V – CN VIII). • 4 CN lie in the medulla (CN IX – CN XII). There are 4 important medial structures that begin with the letter “M”: 1. The Motor pathway (corticobulbar tract and corticospinal tract). 2. The Medial lemniscus (ML: the continuation of the dorsal columns). 3. The MLF. 4. The Motor nuclei that divide equally into the number 12 (i.e., CN III, CN IV, CN VI, and CN XII). All other nuclei (motor or sensory) lie in the lateral brainstem. There are 4 important lateral, or side, structures that being with the letter “S”: 1. The Spinothalamic pathway. 2. The Sympathetics. 3. The Spinocerebellar pathway. 4. The Sensory component of CN V. Neither the spinocerebellar pathway nor the sympathetics decussate. The above is known as the “Rule of 4” and was created by Peter Gates in Australia.

Take Home Messages Remembering these few simple rules will allow you to localize quickly and accurately. Study the Rule of 4 closely and use it to explain the Crossed Signs we saw in our Immediate Localization.

K25928_C003.indd 89

12/01/2017 11:41

90

Chapter 3 The Cranial Nerves and the Brainstem

Figure 3.18 The Vascular Territories of the Medulla

PICA: posterior inferior cerebellar artery.

K25928_C003.indd 90

12/01/2017 11:41

The Vascular Territories of the Medulla

91

Let’s apply the “Rule of 4” to the medulla and then work our way up the brainstem. What structures are in the midline? Based on our rule, we would predict the motor pathway (corticobulbar/corticospinal tract), ML, MLF and CN XII nucleus should be present. Figure 3.18 reveals that we are correct! The motor pathway is located in the most anterior portion. Posterior to it is the ML followed by the MLF and the CN XII nucleus. The CN XII fascicles extend across the medulla to exit anteriorly. What structures are in the side, or lateral, portion? Based on our rule, we would predict the spinothalamic tract (ST), sympathetics, spinocerebellar pathway and the spinal nucleus of CN V. Once again, we are correct! The spinocerebellar tract lies in the cerebellar peduncle and close to this lies the spinal trigeminal nucleus of CN V. Just anterior to this lies the ST. Throughout the entirety of the brainstem the sympathetics lie directly adjacent to the ST. Just anterior to the ST lies the inferior olive, which works closely with the cerebellum to ensure the smooth control of voluntary movement. Based on the equal division of CN across the brainstem, the nuclei for CN IX, CN X and CN XI should be present as well. They lie, as the “Rule of 4” predicts, in the lateral section. The nuclei for CN VIII straddle the border between the medulla and the pons. As we saw in Fig. 2.13, two arteries supply the medulla. The vertebral artery supplies the medial portion and the posterior inferior cerebellar artery (PICA) supplies the lateral portion. This pattern follows for the rest of the brainstem as well; the medial portion is supplied by the main artery (vertebral artery for medulla, basilar artery for the pons, and the posterior cerebral artery for the midbrain), and the lateral section is supplied by the cerebellar arteries [PICA for medulla, anterior inferior cerebellar artery (AICA) for pons, and superior cerebellar artery (SCA) for the midbrain].

Take Home Messages The medial medulla contains the corticospinal tract, ML, MLF and CN XII nuclei. The lateral medulla contains the ST, sympathetics, spinocerebellar tract, sensory nucleus of CN V, and the CN VIII, CN IX, CN X and CN XI nuclei. The medial medulla is supplied by the vertebral artery. The lateral medulla is supplied by the PICA artery.

K25928_C003.indd 91

12/01/2017 11:41

92

Chapter 3 The Cranial Nerves and the Brainstem

Figure 3.19 The Vascular Territories of the Pons

AICA: anterior inferior cerebellar artery; MLF: medial longitudinal fasciculus.

K25928_C003.indd 92

12/01/2017 11:41

The Vascular Territories of the Pons

93

Let’s begin with the medial section of the pons. From our rule, we would predict the midline structures to include the motor pathway (corticobulbar and corticospinal tracts), the ML, the MLF, and CN VI. Indeed, these are all present and in the same relative position as they were in the medulla. Just like CN XII in the medulla, CN VI is at the posterior end of the brainstem and its neurons extend across the pons to exit anteriorly. Employing the “Rule of 4” we would predict that the side, or lateral structures, would include the spinothalamic tract, sympathetics, spinocerebellar tract and the trigeminal nucleus. Figure  3.19 shows that this is indeed the case. Again, the sympathetics lie adjacent to the ST. Not far away lies the trigeminal nucleus. The spinocerebellar tract lies near the posterior side of the pons. Based on the equal division of CN across the brainstem, we would also predict the nuclei of CN VII and CN VIII to be present. Note that CN VII actually wraps around the CN VI nucleus. This is very important clinically; because of this, it is unlikely to have a LMN facial droop due to a lesion in the pons without also having an ipsilateral CN VI palsy. Finding dysfunction of one on clinical exam should prompt a diligent search for the other. As we saw in Fig. 2.13 the small perforators of the basilar artery supply the medial pons. The AICA supplies the lateral pons.

Take Home Messages The medial pons contains the corticospinal tract, ML, MLF and CN VI nuclei. The lateral pons contains the ST, sympathetics, spinocerebellar tract, trigeminal nucleus, and the CN VII and CN VIII nuclei. CN VII wraps around CN VI as it exits the pons. The medial pons is supplied by the basilar artery. The lateral pons is supplied by the AICA artery.

K25928_C003.indd 93

12/01/2017 11:41

94

Chapter 3 The Cranial Nerves and the Brainstem

Figure 3.20 The Vascular Territories of the Midbrain

MLF: medial longitudinal fasciculus; SCA: superior cerebellar artery.

K25928_C003.indd 94

12/01/2017 11:41

The Vascular Territories of the Midbrain

95

Before we apply the “Rule of 4” we should discuss some of the unique structures found in the midbrain. The red nucleus lies in the central part of the midbrain and works with the cerebellum to coordinate voluntary movement. Lesions of the red nucleus result in the same ataxia seen in cerebellar lesions. The substantia nigra secretes dopamine, which is an important neurotransmitter involved in voluntary movement. Lesions of the substantia nigra result in the tremor seen in Parkinson’s disease. The “Rule of 4” begins to break down in the midbrain, because we are quickly transitioning to cerebrum. However, it is still useful. We would predict that CN III and CN IV nuclei would be in the medial section of midbrain. Indeed, both the oculomotor nucleus and the Edinger–Westphal nucleus is present. The CN IV nucleus is in the midline but lies slightly more inferior than the cut we have shown in Fig. 3.20. We would also predict the MLF to be present in the posterior portion of the midline, which it is. The mesencephalic nucleus of CN V is also midline. Recall from Fig.  2.6 that at the level of the midbrain the sensory pathways begin to fuse. The ML has joined the ST in the more lateral aspect of the midbrain. Again the sympathetics are adjacent to the ST. The corticospinal and corticobulbar tracts lie in the anterior portion of the midbrain and extend from the midline all the way to the lateral aspect. In addition, the vascular supply of the midbrain can no longer be thought of as “medial vs. lateral.” Two different oblong territories exist; one supplied by the SCA and one supplied by the PCA.

Take Home Messages The midbrain contains the red nucleus and the substantia nigra, both of which help coordinate voluntary movement. The two vascular territories of the brain are supplied by the SCA and the PCA.

Suggestions for further reading: Blumenfeld H. Neuroanatomy Through Clinical Cases. Sinauer Associates, Inc, Sunderland, 2002. Lanning K, Foroozan R. Neuro-Ophthalmology Review Manual. Slack, Inc, Thorofare, NJ, 2013. Gates P. Clinical Neurology: a Primer. Churchill Livingstone: Elsevier, Sydney, 2010. Wilson-Pauwels , Stewart PA, Akesson EJ, Spacey SD. Cranial Nerves: Function and Dysfunction. People’s Medical Publishing House, Shelton, CT, 2010.

K25928_C003.indd 95

12/01/2017 11:41

Chapter

4

The Spinal Cord

Immediate Localization Only lesions of the spinal cord can produce the following pattern of long tract symptoms: 1. Crossed sensory signs – the patient will have ipsilateral loss of proprioception and vibration and contralateral loss of pinprick and temperature. 2. Sensory level or sensory band – the patient will complain of a distinct transverse level, below which they have impaired sensation of either the spinothalamic tract (ST), dorsal columns (DC) or both. Certain lesions can cause two sensory levels, resulting in a “band” or “zone” of sensory loss.

K25928_C004.indd 97

10/01/2017 10:53

98

Chapter 4 The Spinal Cord

Figure 4.1 The Spine

K25928_C004.indd 98

10/01/2017 10:53

The Spine

99

The spinal cord and the bones and ligaments that surround it are collectively referred to as the spine. Before we delve into the spinal cord itself, we should review these surrounding structures. As you’ll recall from Chapter 1, the bones that protect the spinal cord are called vertebrae and are named using a combination of letters and numbers. The spinal cord, based on changes to its internal morphology, is divided into four distinct sections: cervical, thoracic, lumbar and sacral. The vertebrae share this convention, and are also numbered according to position, i.e., the fifth cervical vertebra is named C5. As shown in Fig. 4.1, there are 7 cervical vertebrae (C1 – C7), 12 thoracic (T1 – T12), five lumbar (L1 – L5) and five sacral (S1 – S5). Examining the vertebrae themselves, we can see they are composed of a large vertebral body, two lateral projections called the transverse processes and a posterior projection called the spinous process. Between the vertebrae lie intervertebral discs; these discs act as shock absorbers and also allow the vertebrae to pivot about them, giving the spine mobility. The spine is held in place by a set of ligaments, the most prominent of which is the interspinous ligament that links spinous processes to each other. The spinal cord sits in the vertebral foramen (foramen is Latin for “opening”) between the vertebral body and the transverse processes. The nerve roots leave the spinal cord via the neural foramen, and continue into the body. The nerve roots are named according to which vertebrae they exit from. The spinal cord actually ends at the L1 vertebra; it does not span the entire length of the spine. The end of the spinal cord is called the conus medullaris. Nerve roots after L1 need to travel to their vertebra in order to exit the spine. Collectively, these travelling nerve roots are called the cauda equina (Latin for “horse’s tail”).

Take Home Messages The nerve roots of the spinal cord leave the vertebrae via the neural foramen. There are 7 cervical vertebrae (C1 – C7), 12 thoracic vertebrae (T1 – T12), five lumbar vertebrae (L1 – L5) and five sacral vertebrae (S1 – S5). The end of the spinal cord is called the conus medullaris and is located at L1.

K25928_C004.indd 99

10/01/2017 10:53

100

Chapter 4 The Spinal Cord

Figure 4.2 The Anatomy of the Spinal Cord

K25928_C004.indd 100

10/01/2017 10:53

The Anatomy of the Spinal Cord

101

Now that we have an understanding of the structure of the spine, we can examine the spinal cord itself. The spinal cord is the main interface between the peripheral nervous system (PNS) and central nervous system (CNS) (the other interface being the brainstem, as we saw in Chapter 3) so lesions of the spinal cord can produce many unique patterns of symptoms. Let’s study Fig. 4.2 from the outside in. A spinal nerve approaches the spinal cord. This nerve carries both sensory and motor neurons, but does so in a nonorganized fashion; both types of neuron are scattered haphazardly across the cross section of the nerve. However, as it approaches the spinal cord, the nerve begins to demonstrate somatotopic properties, which we know from our earlier chapters is a defining feature of the CNS. Just before the nerve enters the cord, it splits into a posterior root, which carries only sensory neurons, and an anterior root, which carries only motor neurons. Looking at the spinal cord itself, we see that it is composed of gray matter and white matter. Like in the cerebrum, the white matter houses the long tracts (corticospinal tract, ST and DC). The gray matter serves as a connection site, or synapse, between neurons. Recall from Chapter 1 that each neuron has a cell body. A group of cell bodies in the CNS is called a nucleus, and in the PNS it is called a ganglion. The cell bodies for the sensory neurons that make up the spinal nerve actually lie outside the spinal cord in the dorsal root ganglion (DRG). The nuclei for motor neurons that make up the spinal nerve lie in the gray matter of the spinal cord, in a part called the anterior horn cell (AHC). The gray matter is exquisitely somatotopically organized. Figure 4.2 shows that each type of information, motor or sensory, somatic or autonomic, has its own specific synapse location in the gray matter.

Take Home Messages Sensory neurons travel in the posterior root. The cell bodies for sensory neurons lie in the DRG. Motor neurons travel in the anterior root. The nuclei for motor neurons lie in the AHC.

K25928_C004.indd 101

10/01/2017 10:53

102

Chapter 4 The Spinal Cord

Figure 4.3 Regional Variations in Spinal Cord Anatomy

AHC: anterior horn cell.

K25928_C004.indd 102

10/01/2017 10:53

Regional Variations in Spinal Cord Anatomy

103

We mentioned earlier that the spinal cord was divided into different sections: cervical, thoracic, lumbar and sacral, based on differences in cross sectional morphology. These differences are highlighted in Fig. 4.3 and we will turn our attention to them now. In order to understand these differences, we need to know the area of the body each section of the spinal cord innervates. Essentially, the cervical cord supplies innervation to the arms, the thoracic cord to the trunk, and lumbar cord to the legs. The sacral part of the cord innervates the bowel, bladder and the genitals. These are generalizations and while, as we will see in Chapter 5, a few exceptions do occur, these suit our purposes nicely for now. As a muscle grows either in size or complexity the amount of physical neuroanatomical space needed to innervate it grows as well. Thus, it is much more challenging for the cervical cord to innervate the arms, than it is for the thoracic cord to innervate the small muscles of the trunk. The cervical spine simply requires more physical space in order to provide this innervation. The same is true in the lumbar cord, in order to innervate the legs. This results not only in a proportionally larger AHC in the cervical and lumbar sections, but also an overall enlargement in cord diameter for these sections. Figure  4.3 shows the cross sections of the spinal cord at each level. If you study them closely, you will notice that the amount of white matter decreases as you move down the cord. This is again because of a need, or lack there of, for neuroanatomical space. Consider the high cervical cord; at this level, motor neurons to each area of the body (arms, trunk, legs, and sacrum) are present. Similarly, sensory neurons from each area are also present. Thus, the cervical cord requires a lot of white matter in order to carry all of these neurons. Now consider the lumbar cord; motor neurons to the arm and thorax have already left the cord, so only motor neurons for the legs and sacrum are present. Similarly, ascending sensory pathways contain only sensory neurons from the sacrum and legs; sensory neurons from the trunk and arms have not yet joined the cord. Thus, relatively little white matter is needed at this level. This again demonstrates the need for physical tissue to transmit neural impulses.

Take Home Messages The cervical and lumbar enlargements refer both to a larger cord diameter and a larger AHC. This physical increase in tissue is needed in order to innervate the large muscles of the arms and leg. As one travels inferiorly down the cord the relative amount of white matter decreases because less information is being transmitted.

K25928_C004.indd 103

10/01/2017 10:53

104

Chapter 4 The Spinal Cord

Figure 4.4 Sensory Tracts in the Spinal Cord

K25928_C004.indd 104

10/01/2017 10:53

Sensory Tracts in the Spinal Cord

105

Before we continue on, let’s pause a moment and review the path of the sensory tracts in the spinal cord, as lesions to them produce the symptoms we mentioned in our Immediate Localization. Sensory neurons travel in the spinal nerve and eventually enter the posterior root. Here they pass by the DRG and eventually enter the posterior horn of the gray matter. If the sensory neurons carry vibration and proprioceptive information they leave the posterior root and travel up the ipsilateral DC. Eventually they synapse with their second order neurons at the level of the medulla and decussate. If the neurons carry pain or temperature information they enter the posterior horn of the gray matter and synapse with their second order neuron. They then immediately decussate and travel up the contralateral ST into the brainstem and thalamus. Remember from Chapter 2 that at the thalamus the two sensory pathways synapse with the same tertiary neuron, and continue together to the contralateral sensory cortex. Consider a complete lesion to the spinal cord at a certain level; this will produce an abrupt, transverse, loss of function of the sensory tracts. This transverse loss of function is called a sensory level; this cannot be caused by a lesion anywhere else in the nervous system. A sensory level can involve the ST, or DC or both, depending on the extent of the lesion. How do we explain sensory bands, and why do they only involve the ST? They are caused by a small lesion in the central gray matter that interrupts the decussation of the neurons as they enter the ST. The ST itself is undamaged, which means pain and temperature sensation is intact above and below the level the lesion. Usually these lesions are several spinal segments long, and produce the band shown in Fig. 4.4. Now consider the case where one-half of the spinal cord is affected, perhaps from a tumor pressing on it laterally. The DC would be affected, so our patient would have ipsilateral loss of vibration and proprioception. The ST is also affected but because these neurons decussate as they enter the spinal cord our patient would have loss of pain and temperature on the contralateral side of the body. This is referred to as crossed sensory signs.

Take Home Messages Sensory neurons for the ST are the only neurons to decussate in the spinal cord. The decussation of spinothalamic neurons and the lack of decussation of DC neurons allow for the unique patterns of dysfunction seen in sensory bands and crossed sensory signs.

K25928_C004.indd 105

10/01/2017 10:53

106

Chapter 4 The Spinal Cord

Figure 4.5 The Somatotopy of the Spinal Cord

K25928_C004.indd 106

10/01/2017 10:53

The Somatotopy of the Spinal Cord

107

As with the rest of the CNS, the long tracts of the spinal cord are somatotopically arranged. This is displayed in Fig. 4.5. Remember, as we established in Fig. 4.3, “cervical” refers to the cervical cord, providing innervation to the arm, “thoracic” to the thoracic cord providing innervation to the trunk, “lumbar” to the lumbar cord innervating the legs, and “sacral” to the sacral cord innervating the bowel, bladder and genitals. Let’s begin with the corticospinal tract. It is organized such that sacral neurons are lateral and cervical neurons are medial, with lumbar and thoracic lying between them. As you can see from Fig. 4.5, neurons exit the corticospinal tract medially; as we progress down the cord, first the cervical neurons will exit, then the thoracic neurons, followed by the lumbar neurons and finally the sacral neurons. The ST is organized the same way as the corticospinal tract: cervical neurons are medial and sacral neurons are lateral. However, the DC are opposite: sacral neurons are medial and cervical neurons are lateral. Another way of seeing it is that the ST adds neurons medially and the DC adds neurons laterally. For example, as we ascend the cord, neurons from the lumbar section are added medially to the ST, and then thoracic neurons are added medially to those. However, for the DC, lumbar neurons are added laterally, and thoracic neurons are added laterally to those. Knowledge of somatotopic properties is critical because it helps us to identify whether the lesion lies inside or outside the spinal cord. For example, consider a tumor pressing on the lateral part of the corticospinal tract. As the corticospinal tract is compressed, the first neurons to be affected are the sacral ones, causing dysfunction of the bowel and the bladder. As the tumor grows, it would then start to compress lumbar neurons, causing leg weakness, and then the cervical neurons, causing arm weakness. Conversely, if the tumor was pressing on the corticospinal tract from inside the spinal cord, one would expect the opposite pattern; first the cervical neurons would be affected causing arm weakness, and then the trunk, legs and finally the bowel, bladder and genitals.

Take Home Message An easy way to remember the somatotopy of the tracts is that the tracts with the word “spinal” in them organize their neurons such that Cervical is Central and Sacral is Side. The DC have the opposite somatotopy.

K25928_C004.indd 107

10/01/2017 10:53

108

Chapter 4 The Spinal Cord

Figure 4.6 The Stretch Reflex

Redrawn with permission from Blumenfeld H. Neuroanatomy Through Clinical Cases. Sunderland: Sinauer Associates, Inc, Sunderland, 2002.

K25928_C004.indd 108

10/01/2017 10:53

The Stretch Reflex

109

When the layperson discusses someone’s reflexes, they are usually referencing that person’s response time. Clinically, we examine for a specific reflex called the stretch reflex. Each orthopedic joint in the body has muscles that actively move it and passively stabilize it. The underlying principle of the stretch reflex is that if the body senses a joint is being moved in an unstable fashion, it will restabilize it by activating certain muscles and deactivating other muscles. This is depicted in Fig. 4.6. If we strike the triceps tendon, receptors in the triceps will, erroneously, believe the elbow joint is being rapidly and unstably moved because of the stretch of the tendon induced by the hammer. This will automatically cause the triceps to contract and try to shorten the stretched tendon; this also results in extension of the forearm. How exactly does this reflex arc occur? Proprioceptive information about the stretch induced by the hammer is sensed by the muscle spindle and travels in a sensory neuron, past the DRG and the gray matter of the cord. Here, it synapses with two neurons. One neuron is excitatory, telling the agonist muscle, in this case the tricep, to contract. The other is an interneuron, which is inhibitory, telling the antagonist muscle, in this case the biceps, to relax. By having simultaneous contraction of the triceps and relaxation of the biceps, the forearm extends rapidly, pulling the elbow back into stability. While this reflex arc does not involve the upper motor neuron (UMN), the UMN does act to inhibit or dampen the lower motor neuron (LMN), as we will see shortly. Clinical grading of reflexes is important. Reflexes are graded on a five-point scale (0 to 4+). The scale is as follows: • • • • •

0 Completely absent reflex; 1+ Decreased reflex; 2+ Normal reflex; 3+ Increased reflex; 4+ Increased reflex with induction of clonus (a rhythmic muscle contraction we will discuss further in a moment).

When we say that a reflex is increased or decreased, we are judging both its speed and amplitude.

Take Home Message The stretch reflex causes the agonist muscle that is stretched to contract, and the antagonist muscle to relax.

K25928_C004.indd 109

10/01/2017 10:53

110

Chapter 4 The Spinal Cord

Figure 4.7 Upper and Lower Motor Neuron Lesions

K25928_C004.indd 110

10/01/2017 10:53

Upper and Lower Motor Neuron Lesions

111

As we saw earlier, the motor pathway is composed of two neurons: an UMN and a LMN. An UMN lesion will produce very different symptoms than a LMN lesion. Recognizing these different clinical presentations is often the first step in localizing. In order to understand these different presentations, we need to understand how the UMN, LMN, and muscle interact. In general, as one progresses down the chain from UMN, to LMN and to muscle, there is a dampening or inhibitory affect; the UMN dampens the LMN which dampens the muscle. Without the dampening affect, downstream components will fire both spontaneously and randomly. As we just saw, the reflex arc acts at the level of the spinal cord. The UMN acts to dampen the LMN in this pathway; thus, lesions of the UMN present with increased reflexes, as the LMN is no longer dampened. However, since the LMN is part of the reflex arc, damage to it will result in decreased reflexes. From the point of view of the muscle, as long as it is still connected to its LMN, it is fully innervated. It has no idea that the LMN innervating it is actually controlled by an UMN. UMN lesions do not have any affect on the muscle, other than weakness. Lesions to the LMN, on the other hand, will produce significant atrophy of the muscle, since the muscle no longer has a functional connection to the nervous system. When a LMN is damaged, it can begin to fire inappropriately and without stimulus. This process is sometimes called aberrant firing. These random firings of the LMN cause chaotic and irregular contractions of the muscle called fasciculations. Resting tone is defined as the resistance of muscles to passive movement provided by an external stimulus (usually an examiner). Tone is increased in an UMN lesion (i.e., it is harder for the examiner to move affected muscles groups), and decreased (i.e., it is easy for the examiner to move affected muscle groups) in a LMN lesion.

Take Home Messages The motor system is designed such that each component, UMN, LMN or muscle, dampens the component that follows it. UMN lesions are associated with increased tone and increased reflexes. LMN lesions are associated with decreased tone and reflexes as well as fasciculations and atrophy.

K25928_C004.indd 111

10/01/2017 10:53

112

Chapter 4 The Spinal Cord

Figure 4.8 Features of an Upper Motor Neuron Lesion

K25928_C004.indd 112

10/01/2017 10:53

Features of an Upper Motor Neuron Lesion

113

UMN lesions produce several additional clinical findings that have no LMN counterpart. The first is the presence of an up-going toe, also called the Babinski sign, after the neurologist that first described it. In an UMN, if the foot is stroked along the lateral border, as in Fig. 4.8, the first toe will extend upwards. Note: the reverse is not true. If, when the foot is stroked, the toe moves downwards, all that can be concluded is that an UMN lesion is not present, not that there is a LMN lesion (i.e., a down-going toe can be a normal finding). The mechanism by which a toe moves up or down is complicated and of little practical value. The second clinical finding is the presence of clonus, which is a series of involuntary, rhythmic, muscular contractions that are induced by movement. Usually this is elicited by having the examiner quickly and forcefully hyperflex a joint, as depicted with the ankle joint in Fig. 4.8. In this instance, the ankle would repeatedly oscillate back and forth; each oscillation is termed a “beat” of clonus. More than three beats of clonus is indicative of an UMN lesion. Occasionally, reflex testing can spontaneously induce clonus. As we just saw, an UMN lesion is associated with increased tone. However, there are actually two types of increased tone. Rigidity is a velocity independent increase in tone throughout the entire range of motion that the muscle is put through. The amount of resistance exerted by the muscle is totally independent of how quickly the examiner moves it; the analogy is often made to moving a lead pipe. Spasticity, on the other hand, is a velocity dependent increase in tone throughout only part of the range of motion. As the examiner moves the muscle, there appears to be no or minimal increased tone, however if he quickly then jerks the muscle, he feels a sharp increased resistance, often coined a “spastic catch.” The analogy here is made to opening a Swiss army knife.

Take Home Messages An up-going toe is indicative of an UMN lesion. A down-going toe is present in both LMN lesions and normal physiology. More than three beats of clonus is indicative of an UMN lesion. Rigidity is a velocity independent increase in tone. Spasticity is a velocity dependent increase in tone.

K25928_C004.indd 113

10/01/2017 10:53

114

Chapter 4 The Spinal Cord

Figure 4.9 The Arterial Supply to the Spinal Cord

PICA: posterior inferior cerebellar artery.

K25928_C004.indd 114

10/01/2017 10:53

The Arterial Supply to the Spinal Cord

115

The spinal cord is supplied by three main arteries: the anterior spinal artery, which supplies the anterior two-thirds of the spinal cord, and two posterior spinal arteries, which together supply the posterior one-third of the cord. As you’ll recall from Chapter 2, the vertebral arteries are a branch of the subclavian artery and travel up, through the vertebra into the head to supply the posterior circulation. Just before the two vertebral arteries merge to form the basilar artery, they each give off branches that quickly unify to form the anterior spinal artery. This artery travels longitudinally down the length of the spinal cord. Similarly, the posterior spinal artery is a branch of the very proximal posterior inferior cerebellar artery (PICA). As we have seen in Chapter 2, there are considerable variations in the posterior circulation between individuals, and in some the posterior spinal artery is actually a branch of the vertebral artery. Just as the cerebrum has collateral vasculature in the form of the Circle of Willis, the spinal cord has collateral vasculature through the mighty aorta itself. Throughout the thorax, the aorta gives off small segmental arteries, which branch into the anterior radicular artery and the posterior radicular artery. The anterior radicular artery travels to and merges with the anterior spinal artery, and the posterior radicular artery travels to and merges with the posterior spinal artery. In this way, the “circle of vasculature” for the spinal cord is completed.

Take Home Messages The anterior spinal artery is derived from the two vertebral arteries and supplies the anterior two-thirds of the spinal cord. The posterior spinal artery is a branch of the PICA (or in some cases the vertebral artery) and supplies the posterior one-third of the spinal cord. The spinal cord vasculature has a rich collateral vasculature linking the posterior cerebral circulation directly to the aorta.

K25928_C004.indd 115

10/01/2017 10:53

116

Chapter 4 The Spinal Cord

Figure 4.10 The Approach to Spinal Cord Damage

LMN: lower motor neuron; UMN: upper motor neuron.

K25928_C004.indd 116

10/01/2017 10:53

The Approach to Spinal Cord Damage

117

We’ve covered a lot in this chapter. Let’s reinforce and consolidate our knowledge of the anatomy of the spinal cord by working through a case. Consider an extrinsic lesion compressing the cord at C5. This lesion will produce two distinct sets of symptoms, one from CNS dysfunction (interruption of the long tracts) and one from PNS dysfunction (interruption of the neurons in the anterior and/or posterior roots). We’ll begin with the root dysfunction first. As our lesion presses on the cord, we will get LMN signs in the muscles innervated by C5 due to compression of either the AHC or the anterior root. Similarly, if the DRG or posterior root is compressed, we will get sensory dysfunction in the area supplied by C5 (i.e., the C5 dermatome: see Chapter 5). In this way, recognizing PNS signs (LMN weakness and dermatomal sensory loss) localizes the spinal cord lesion longitudinally. Now lets consider the long tract dysfunction. If our lesion at C5 grows sufficiently large, it will move beyond the DRG/AHC and begin to compress the long tracts. This interrupts the corticospinal tract and causes UMN weakness below the level of the lesion. The affected muscle groups can be predicted based on our somatotopic knowledge of the corticospinal tract. In our example, we would expect sacral signs first, then the legs, and finally the arms to be affected. Similarly, there will be a complete sensory loss below the level of the lesion due to interruption of the ST and DC. Again, the areas of the body that will be affected first can be predicted based on the somatotopy of the sensory tracts. In this way, recognizing CNS signs (UMN weakness and complete sensory loss) localizes the spinal cord lesion in cross section; it tells us how much of the spinal cord is involved. For quick reference, the chart in Fig. 4.10 gives an estimation of where the lesion is, based on motor signs.

Take Home Messages Signs of PNS damage (LMN weakness and dermatomal sensory loss) occur at the level of the lesion, and localize the lesion longitudinally. Signs of CNS damage (UMN weakness and complete sensory loss), occur below the level of the lesion, and allow us to determine how much of the cord is lesioned.

Suggestions for further reading: Blumenfeld H. Neuroanatomy Through Clinical Cases. Sunderland: Sinauer Associates, Inc, Sunderland, 2002. Lindsay, KW, Bone I, and Fuller G. Neurology and Neurosurgery Illustrated. Churchill Livingstone: Elsevier, Edinburgh, 2010.

K25928_C004.indd 117

10/01/2017 10:53

Chapter

5

The Peripheral Nervous System

Suggestive Localization Unfortunately the peripheral nervous system (PNS) is so varied that it does not have an Immediate Localization. However, as we have seen, the central nervous system (CNS) tends to produce lesions that affect one-half of the body, or at least the entirety of a limb. PNS lesions tend to produce irregular patterns of loss in a single limb only. For example, the ulnar nerve doesn’t even provide sensory innervation to the entire hand. Similarly, PNS lesions tend to only affect some of the muscles of a limb; again, dysfunction of the ulnar nerve produces weakness in only some muscles of the hand. While exceptions are always possible, whenever you are faced with such an incomplete pattern of symptoms, a PNS lesion should be the first thing that springs to mind.

K25928_C005.indd 119

10/01/2017 10:52

120

Chapter 5 The Peripheral Nervous System

Figure 5.1 Lesion Types in the Peripheral Nervous System

Redrawn with permission from Blumenfeld H. Neuroanatomy Through Clinical Cases. Sunderland: Sinauer Associates, Inc, Sunderland, 2002.

K25928_C005.indd 120

10/01/2017 10:52

Lesion Types in the Peripheral Nervous System

121

As we see in Fig. 5.1, nerve roots exit the spinal cord and travel through a complex network, called a plexus. As they travel through the plexus they merge to form peripheral nerves. Figure 5.1 demonstrates the brachial plexus, which innervates the arm; the lumbosacral plexus serves to innervate the legs. Nerves that innervate the trunk do not travel through a plexus, they arise directly from the spinal cord. Neuropathy is a nonspecific term used to describe a lesion located somewhere in the PNS. A lesion affecting the nerve root itself results in a radiculopathy, whereas a lesion to the plexus results in a plexopathy. If the nerve itself is lesioned it is called a: • Mononeuropathy if one individual nerve is lesioned. • Mononeuropathy multiplex if two or more unrelated nerves are lesioned, resulting in multifocal disease (i.e., dysfunction of a nerve in an arm and nerve in a leg). • Polyneuropathy if the disease process is generalized, resulting in a symmetric involvement of nerves on both sides of the body (i.e., the nerves involved in the right leg are the same as the nerves involved in the left leg). People will often refer to a mononeuropathy as simply a neuropathy, but this is technically incorrect. Examining Fig. 5.1 note that, uniquely, the C8 nerve root has no corresponding vertebra. This means that up to the C7 vertebra, nerve roots leave the cord above their corresponding vertebra; after C7 they leave below it.

Take Home Messages Nerve roots travel through a plexus in order to merge and become peripheral nerves. The brachial plexus innervates the arms, and the lumbosacral plexus innervates the legs. Neuropathies are named according to how many individual nerves have lesions.

K25928_C005.indd 121

10/01/2017 10:52

122

Chapter 5 The Peripheral Nervous System

Figure 5.2 The Brachial Plexus

K25928_C005.indd 122

10/01/2017 10:52

The Brachial Plexus

123

Let’s consider the brachial plexus in further detail. All innervation to the arm is composed of the C5 – T1 nerve roots, which travel through the brachial plexus before becoming individual peripheral nerves. It is important to be able to draw the brachial plexus, and we will turn our attention to that in a moment. The brachial plexus is divided into five parts, beginning with the roots C5 – T1. These five roots merge into three trunks, named the upper, middle and lower trunk. Each trunk then bifurcates into an anterior and posterior division. These six divisions merge to form three cords, called lateral, posterior and medial. Finally, the cords form five branches, which are the peripheral nerves. They are the axillary nerve, musculocutaneous nerve, radial nerve, median nerve and ulnar nerve. For our purposes, we only need to focus on these five large nerves in order to fully understand the arm, rather than the myriad of small branches they give off.

Take Home Messages The brachial plexus is composed of roots (5), trunks (3), divisions (6), cords (3) and branches (5). The major peripheral nerves of the arm are: the axillary nerve, radial nerve, musculocutaneous nerve, radial nerve and ulnar nerve.

K25928_C005.indd 123

10/01/2017 10:52

124

Chapter 5 The Peripheral Nervous System

Figure 5.3 How to Draw the Brachial Plexus

K25928_C005.indd 124

10/01/2017 10:52

How to Draw the Brachial Plexus

125

Though the brachial plexus seems daunting, it can be very easily remembered by drawing three “Y”s, a “W” and an “X.” Begin by drawing three “Y”s, two running in one direction, and one running in the opposite direction, and label them C5 – T1 as in Fig. 5.3. Now join the top and bottom lines together by adding a “W” to the far end, as shown. An “X” is then added between the first two lines. Finally, join the last two lines together with a slash. Those are all the important connections of the brachial plexus. Label the ends of the figure as the individual peripheral nerves, as shown. Examining the figure, we can see that the musculocutaneous nerve is made up of C5, C6 and C7. The ulnar nerve is made up of C8 and T1. The median and radial nerves have contributions from all the nerve roots, C5 – T1. From Fig. 5.3, it would appear that the axillary nerve is also made up of all the nerve roots, C5 – T1. However, the nerve roots destined for the axillary nerve simply do not mix with the other nerve roots in the posterior cord and the axillary nerve is only composed of C5 and C6. It is one of the peculiarities of neuroanatomy that, unfortunately, needs to be committed to memory.

Take Home Messages The musculocutaneous nerve is composed of nerve roots C5, C6 and C7. The median nerve is composed of nerve roots C5 – T1. The ulnar nerve is composed of nerve roots C8 – T1. The axillary nerve is composed of nerve roots C5 and C6. The radial nerve is composed of nerve roots C5 – T1.

K25928_C005.indd 125

10/01/2017 10:52

126

Chapter 5 The Peripheral Nervous System

Figure 5.4 Sensory Innervation and Common Reflexes

K25928_C005.indd 126

10/01/2017 10:52

Sensory Innervation and Common Reflexes

127

Before we move on to cover the individual nerves of the arm, it is worth a moment to pause and consider some potentially confusing terminology. As we mentioned, nerve roots mix together as they travel through the plexus, and merge to form individual nerves (i.e., C5, C6 and C7 merge to form the musculocutaneous nerve). Via the mixing in the plexus, a single nerve root can contribute to multiple different nerves (i.e., C5 and C6 also form the axillary nerve and C7 contributes to the median, ulnar and radial nerves). Thus, a single nerve root will have multiple sensory and motor functions, by contributing to multiple different nerves. The region of skin innervated by a single nerve root is called that nerve root’s dermatome. Similarly, the muscles innervated by a single nerve root are called its myotome. When mapping the sensory innervation of an area, such as the arm in Fig. 5.4, one can speak in terms of a) dermatomal innervation provided by the nerve root, or in terms of b) innervation provided by the peripheral nerve. Note that they are not the same. For example, the sensory innervation of the musculocutaneous nerve (C5, C6 and C7) should be contrasted to the sensory innervation provided individually by C5, C6 and C7. Why is this the case? Though C5, C6 and C7 all contribute to the musculocutaneous nerve, they also contribute to other nerves that provide sensory innervation (such as the axillary nerve for C5 and C6, and the median, ulnar and radial nerve for C7). It is therefore quite important to see if a patient’s sensory loss matches the pattern produced by a lesion to a peripheral nerve, or to a lesion of a nerve root. We will cover myotomes in a moment. As we saw in Fig. 4.6, reflexes are a test of both sensory and motor function. The nerve roots examined when testing common reflexes are shown in Fig.  5.4, and thankfully can be remembered easily by recalling the “1, 2, Buckle My Shoe” rhyme. 1,2 Buckle my shoe (ankle reflex mediated by S1 and S2), 3,4 Kick the door (knee reflex mediated by L3 and L4), 5,6 Pick up sticks (biceps reflex mediated by C5 and C6), 7,8 Lay them straight (triceps reflex mediated by C7 and C8).

Take Home Messages The area of skin innervated by a single nerve root is called its dermatome. The muscles innervated by a single nerve root are called its myotome.

K25928_C005.indd 127

10/01/2017 10:52

128

Chapter 5 The Peripheral Nervous System

Figure 5.5 Dominant Myotomes

Some individual figures adapted from Waxman SG. Clinical Neuroanatomy. Lange, New York, 2013.

K25928_C005.indd 128

10/01/2017 10:52

Dominant Myotomes

129

While each nerve root contributes to multiple muscles, via the mixing that occurs in the plexus, anatomical studies show that usually one (or at most two) nerve root provides the dominant innervation to a muscle. This can be very useful clinically when there is a concern of an injury to the spinal cord and accompanying nerve roots. By assessing a few quick muscle groups, the examiner can quickly glean which nerve roots are intact and which ones are damaged. Figure  5.5 shows the dominant myotome for each nerve root. Thankfully there is a pattern! Nerve roots tend to linearly follow major muscle groups as one moves down the body. Let’s review this now. We begin with the head and see that C1 is responsible for neck flexion. C2 is responsible for neck extension. C3 is responsible for lateral neck flexion to both the left and the right. As we continue down the body, we see that C4 elevates or shrugs the shoulders. C5 innervates the abductors of the shoulder. Moving into the arm, we see that C6 innervates two muscle groups: those responsible for elbow flexion and wrist extension. C7 also innervates two muscle groups: those responsible for elbow extension and wrist flexion. Note that the elbow and wrist are always opposite. If a nerve root contributes to elbow flexion it provides wrist extension, and vice versa. C8 is responsible for finger flexion and T1 is responsible for finger abduction. As we move down into the legs, the nerve roots innervate the anterior muscles, and then circle around the foot, and work their way back up the posterior muscles. L2 is responsible for hip flexion and L3 for knee extension. Both L4 and L5 innervate the muscle for ankle dorsiflexion and L5 alone innervates the muscle that extends the first toe. S1 is responsible for ankle plantar flexion and hip extension. As we move up the posterior part of the leg we see that S2 is responsible for knee flexion.

Take Home Message One can remember the dominant myotomes by remembering that nerve roots linearly follow the muscles groups as one works down the body and then circle up the posterior part of the leg.

K25928_C005.indd 129

10/01/2017 10:52

130

Chapter 5 The Peripheral Nervous System

Figure 5.6 The Axillary Nerve (C5, C6)

The Musculocutaneous Nerve (C5, C6, C7)

Some individual figures adapted from Waxman SG. Clinical Neuroanatomy. Lange, New York, 2013.

K25928_C005.indd 130

10/01/2017 10:52

The Axillary (C5, C6) and Musculocutaneous (C5, C6, C7) Nerves

131

The first nerve we will consider is the axillary nerve, which is made up of nerve roots C5 and C6. The axillary nerve comes off the posterior cord and travels a short distance before wrapping around and ultimately becoming anterior to the humerus. Clinically, it is responsible for innervation of the deltoid, which abducts the shoulder beyond the first 20°. The first 20° involve the muscles in the rotator cuff. The axillary nerve is also responsible for sensory innervation to the upper lateral arm. Remember to compare this to the C5 and C6 dermatome. The musculocutaneous nerve comes off the lateral cord and is composed of C5, C6 and C7. This nerve travels anterior to the humerus and terminates in the upper arm, innervating the biceps, which flexes the forearm. Before terminating, the musculocutaneous nerve gives off a branch, the lateral cutaneous nerve of the forearm. This nerve’s only function is to provide sensation to the lateral aspect of the forearm.

Take Home Messages The axillary nerve innervates the deltoid, which is responsible for abduction of the arm beyond the initial 20°. The musculocutaneous nerve innervates the biceps, which flexes the elbow. The musculocutaneous nerve also has a branch, the lateral cutaneous nerve of the forearm, which provides sensation to the lateral forearm.

K25928_C005.indd 131

10/01/2017 10:52

132

Chapter 5 The Peripheral Nervous System

Figure 5.7 The Radial Nerve (C5-T1)

Some individual figures adapted from Waxman SG. Clinical Neuroanatomy. Lange, New York, 2013.

K25928_C005.indd 132

10/01/2017 10:52

The Radial Nerve (C5-T1)

133

The radial nerve is composed of nerve roots C5 – T1 and comes off the posterior cord. It initially travels posterior to the humerus, and branches to innervate the triceps, before wrapping around to become anterior to the elbow. At the elbow, it branches into a motor and sensory component. The motor component, the deep branch, travels into the posterior forearm. The sensory component, known as the superficial branch, travels down to the dorsum of the hand. An easy way to remember the motor function of the radial nerve is that it supplies all the extensor muscles in the arm, whether it is the forearm, wrist, finger or thumb. The powerful triceps extend the forearm. The extensor carpi (‘carpi’ is Latin for wrist) extends the wrist. Extension of the fingers is provided by the extensor digitorum. Finally, extension of the thumb is provided by the extensor pollicis (‘pollicis’ is Latin for thumb). In addition, the radial nerve also innervates the plainly named supinator, which supinates the forearm. The radial nerve supplies sensation to the posterior area of the arm, forearm, and to the posterior aspect of the first three digits of the hand. Note some movements listed, such as extension of the wrist, are actually provided by two muscles, one on the radial side (i.e., the extensor carpi radialis) and one on the ulnar side (i.e., the extensor carpi ulnaris). In addition, some muscles have multiple heads, usually a longer one, “longus,” and a shorter one, “brevis,” such as extensor pollicis longus, and extensor pollicis brevis. While more accurate, it is not particularly clinically relevant at this stage of training, and we mention it only so as not to cause confusion with what is listed in other texts.

Take Home Messages The radial nerve innervates the triceps and then divides into a solely motor component, the deep branch, and a solely sensory component, the superficial branch. The radial nerve innervates all the extensors of the arm, as well as the supinator of the forearm.

K25928_C005.indd 133

10/01/2017 10:52

134

Chapter 5 The Peripheral Nervous System

Figure 5.8 The Ulnar Nerve (C8, T1)

Some individual figures adapted from Waxman SG. Clinical Neuroanatomy. Lange, New York, 2013.

K25928_C005.indd 134

10/01/2017 10:52

The Ulnar Nerve (C8, T1)

135

The ulnar nerve is composed of nerve roots C8 and T1 and comes off the medial cord. It travels through the upper arm medial to the humerus, and doesn’t provide any function, motor or sensory, until it reaches the forearm. It continues in the forearm and travels into the hand through Guyon’s canal at the wrist. Guyon’s canal is an important area because it can be involved in a compressive neuropathy. The ulnar nerve is very important because it supplies nearly all the muscles of the hand, with a few exceptions that we will discuss shortly. Clinically important muscles in the hand include the adductor pollicis, which adducts the thumb, and the dorsal interossei, which spread, or abduct the fingers. In addition, the ulnar nerve is also responsible for flexion of the 4th and 5th digit via the flexor digitorum. The ulnar nerve has one motor function outside of the hand, and that is to flex the wrist via the flexor carpi. The sensory area supplied by the ulnar nerve is confined to the one-half of the 4th digit and the entire 5th digit, as shown in Fig. 5.8.

Take Home Messages As the ulnar nerve enters the hand, it passes through Guyon’s canal, which is often the site of a compressive neuropathy. The ulnar nerve innervates nearly all the muscles of the hand. The ulnar nerve supplies sensation to the entire 5th digit, but only to one half of the 4th digit.

K25928_C005.indd 135

10/01/2017 10:52

136

Chapter 5 The Peripheral Nervous System

Figure 5.9 The Median Nerve (C5-T1)

Some individual figures adapted from Waxman SG. Clinical Neuroanatomy. Lange, New York, 2013.

K25928_C005.indd 136

10/01/2017 10:52

The Median Nerve (C5-T1)

137

The median nerve is composed of nerve roots C5 – T1 and is formed by the union of lateral and medial cord. The median nerve travels down into the forearm, where it provides innervation to several muscles. It continues into the hand through the carpal tunnel, which can often be the site of a compressive neuropathy. The median nerve has a grab bag of motor functions, and some argue it is best remembered as the exception of the radial and ulnar nerve. Its sole forearm function is pronation of the forearm, which is carried out by the pronator teres. It is responsible for three of the five movements of the thumb, including opposition by the opponens pollicis, abduction by the abductor pollicis, and flexion by the flexor pollicis. Finally, it also flexes the second and third digits via the flexor digitorum. The median nerve provides sensation to the palmar aspect of first, second, and third digits, and to one-half of the fourth digit.

Take Home Messages As the median nerve enters the hand, it passes through the carpal tunnel, which is often the site of a compressive neuropathy. The median nerve provides motor input to the opponens pollicis, abductor pollicis, flexor pollicis, and the second and third tendons of the flexor digitorum. All other hand muscles are supplied by the ulnar nerve.

K25928_C005.indd 137

10/01/2017 10:52

138

Chapter 5 The Peripheral Nervous System

Figure 5.10 How to Draw the Lumbosacral Plexus

K25928_C005.indd 138

10/01/2017 10:52

How to Draw the Lumbosacral Plexus

139

Unlike the brachial plexus, the lumbosacral plexus is not divided into distinct subsections (i.e., “trunks,” “divisions,” and “cords”). However, it is still important to know which nerve roots contribute to the major nerves of the leg. Whereas the brachial plexus was easily remembered with the letters “Y,” “W” and “X,” the lumbosacral plexus will be remembered with numbers “4,” “5” and “3.” Draw 7 straight lines labeled L2 to S3. Starting at L4, place a total of 4 bifurcations: one on each of L4, L5, S1 and S2 as shown in Fig. 5.10. Join these together to form the common peroneal nerve. Proceed a little further downstream on L4 and now draw a total of 5 bifurcations, just as before, on L4 to S3; join these together to form the tibial nerve. Initially, the common peroneal and tibial nerve travel together in the same nerve sheath. As a result, early anatomists thought they represented one nerve and called it the sciatic nerve. Unfortunately, this erroneous distinction continues to this day. Now draw 3 bifurcations beginning with L2, and moving downwards, just as before. Join these together to form the obturator nerve. We are nearly finished. Join the first three lines (L2 – L4) together to form the femoral nerve. Join the next set of three lines starting from L5 (L5 – S2) together to form the inferior gluteal nerve. Finally, join the three lines L4, L5 and S1 together to form the superior gluteal nerve. There are many more branches of the lumbosacral plexus, but the above scheme covers the major clinically relevant nerves.

Take Home Messages The obturator nerve is composed of L2 – L4. The femoral nerve is also composed of L2 – L4. The superior gluteal nerve is composed of L4 – S1. The inferior gluteal nerve is composed of L5 – S2. The common peroneal nerve is composed of L4 – S2. The tibial nerve is composed of L4 – S3. The common peroneal nerve and tibial nerve initially travel in the same nerve sheath and are collectively referred to as the sciatic nerve.

K25928_C005.indd 139

10/01/2017 10:52

140

Chapter 5 The Peripheral Nervous System

Figure 5.11 The Nerves and Muscles of the Hip

Some individual figures adapted from Waxman SG. Clinical Neuroanatomy. Lange, New York, 2013.

K25928_C005.indd 140

10/01/2017 10:52

The Nerves and Muscles of the Hip

141

The nerves of the arm have many branches and typically supply many different muscles. When we looked at them it made sense to examine the path and branches of each nerve first, and then its associated sensory and motor functions. This is in contrast to the leg, where each nerve typically only has one or two motor functions. Thus, a more useful approach is to first examine the movements of each joint and then the nerve that supplies the muscle responsible for that movement. We will begin with the hip. The four cardinal movements of the hip are flexion, extension, abduction and adduction. Hip flexion is facilitated by the iliopsoas muscle, which lies in the posterior compartment of the abdominal cavity. It attaches the spine to the anterior side of the femur. Since this muscle is so close to the lumbar plexus, it is not served by an individual nerve, but is innervated directly by nerve roots L1 – L3. Hip extension is achieved by the powerful gluteus maximus, which attaches the sacrum to the posterior side of the femur. It is innervated by the inferior gluteal nerve (L5, S1 and S2). The inferior gluteal nerve does not have a sensory component. Hip adduction is provided by the obturator nerve (L2, L3 and L4), which innervates a group of muscles simply referred to as the adductors. They attach the pelvis to the medial side of the femur. The obturator nerve is infamously associated with trauma from overzealous use of forceps during childbirth. The obturator has a modest sensory component, providing innervation to a small area of the medial thigh. Hip abduction is achieved by the gluteus minimus and gluteus medius through the superior gluteal nerve (L4, L5 and S1). They attach the iliac crest to the femur at the greater trochanter. The superior gluteal nerve does not have a sensory component.

Take Home Messages The iliopsoas is responsible for hip flexion and is innervated directly by L1 – L3. The gluteus maximus is responsible for hip extension and is innervated by the inferior gluteal nerve. The adductors are responsible for hip adduction and are innervated by the obturator nerve. The gluteus minimus and gluteus medius are responsible for hip abduction and are innervated by the superior gluteal nerve.

K25928_C005.indd 141

10/01/2017 10:52

142

Chapter 5 The Peripheral Nervous System

Figure 5.12 The Nerves and Muscles of the Knee

Some individual figures adapted from Waxman SG. Clinical Neuroanatomy. Lange, New York, 2013.

K25928_C005.indd 142

10/01/2017 10:53

The Nerves and Muscles of the Knee

143

The knee only has two possible movements: extension and flexion. Knee extension is performed by the quadriceps which, as the name implies, is actually made up of four different muscles (the rectus femoris, vastus lateralis, vastus medialis, and vastus intermedius). Practically, however, they can be thought of as one group. The quadriceps attach proximally to both the femur and pelvis, and distally to the tibia. The quadriceps are innervated by the femoral nerve (L2, L3 and L4). Knee extension is so critical to life, as it is the main muscle involved in standing, that innervating the quads is the sole motor function of the massive femoral nerve. The sensory distribution of the femoral nerve is huge, and covers more than half of the leg, as shown in Fig. 5.12. As we mentioned, the common peroneal nerve and tibial nerve initially travel together in the thigh as the sciatic nerve (L4, L5, S1, S2, S3). The sciatic nerve provides innervation to the hamstrings. Again, this is a composite muscle and is made up of the semi-tendinous muscle, the semi-membranous muscle and the biceps femoris. The hamstrings attach the posterior part of the pelvis to the bones of the lower leg, the tibia and fibula. Contraction of the hamstring results in knee flexion. The sciatic nerve’s sensory component is simply the summation of the sensory component of the common peroneal and tibial nerves (which we will cover next), and is shown in Fig. 5.12.

Take Home Messages The quadriceps are responsible for knee extension and are innervated by the femoral nerve. The hamstrings are responsible for knee flexion and are innervated by the sciatic nerve.

K25928_C005.indd 143

10/01/2017 10:53

144

Chapter 5 The Peripheral Nervous System

Figure 5.13 The Nerves and Muscles of the Ankle: I

Some individual figures adapted from Waxman SG. Clinical Neuroanatomy. Lange, New York, 2013.

K25928_C005.indd 144

10/01/2017 10:53

The Nerves and Muscles of the Ankle: I

145

Like the hip joint, the ankle has four possible types of movement: foot dorsiflexion, in which the angle between the foot and the leg is decreased, foot plantar flexion, in which the angle between the foot and the leg is increased, foot inversion and foot eversion. Routinely, the toes are not assessed, with one major exception: extension of the first toe. The muscles responsible for all these movements are innervated by the divisions of the sciatic nerve: the tibial nerve, responsible for foot plantar flexion and foot inversion, and the common peroneal nerve, responsible for foot dorsiflexion, foot eversion and extension of the first toe. We will begin with the tibial nerve. The sciatic nerve branches into the tibial nerve and common peroneal nerve just superior and posterior to the knee. The tibial nerve travels posteriorly into the foot. On its way, it innervates the gastrocnemius and soleus, which together attach the femur, tibia and fibula to the posterior aspect of the foot, and are responsible for ankle plantar flexion. Slightly more distally, the tibial nerve also innervates the tibialis posterior, which is responsible for foot inversion. The tibial nerve provides sensory innervation to the entire sole of the foot.

Take Home Messages The gastrocnemius and soleus are innervated by the tibial nerve and are responsible for ankle plantar flexion. The tibialis posterior is also innervated by the tibial nerve and is responsible for inversion of the foot.

K25928_C005.indd 145

10/01/2017 10:53

146

Chapter 5 The Peripheral Nervous System

Figure 5.14 The Nerves and Muscles of the Ankle: II

Some individual figures adapted from Waxman SG. Clinical Neuroanatomy. Lange, New York, 2013.

K25928_C005.indd 146

10/01/2017 10:53

The Nerves and Muscles of the Ankle: II

147

As we mentioned, the common peroneal nerve begins superior and posterior to the knee. It immediately wraps around and enters the anterior compartment of the leg. As it does so, it divides into the superficial peroneal nerve and the deep peroneal nerve. The deep peroneal nerve supplies the tibialis anterior. This powerful muscle is the primary dorsiflexor of the foot. Thus, problems with the tibialis anterior result in an infamous clinical presentation, the “foot drop.” The deep peroneal nerve also supplies the extensor hallucis longus, which is responsible for extension of the first toe. The superficial peroneal nerve supplies the peroneus muscle. This muscle attaches the lateral part of the fibula to the 5th metatarsal and is responsible for foot eversion. The sensory distribution of the common peroneal nerve is shown in Fig. 5.14 and includes the lateral aspect of the calf, as well as the dorsal aspect of the foot. With that, we’ve covered all the elements of the PNS, from the individual nerve roots, to the plexuses, and finally the individual nerves themselves. For your reference we’ve included Fig. 5.15 and Fig. 5.16 as a summary of everything we’ve discussed.

Take Home Messages The powerful tibialis anterior is responsible for ankle dorsiflexion and is innervated by the deep peroneal nerve. The extensor hallucis longus is responsible for extension of the first toe, and is also innervated by the deep peroneal nerve. The peroneus muscle is responsible for foot eversion and is innervated by the superficial peroneal nerve.

K25928_C005.indd 147

10/01/2017 10:53

148

Chapter 5 The Peripheral Nervous System

Figure 5.15 Sensory Map of the Peripheral Nervous System

K25928_C005.indd 148

10/01/2017 10:53

Sensory Map of the Peripheral Nervous System

K25928_C005.indd 149

149

10/01/2017 10:53

150 Chapter 5 The Peripheral Nervous System

Figure 5.16 Summary of the Peripheral Nervous System

K25928_C005.indd 150

10/01/2017 10:53

Summary of the Peripheral Nervous System

151

Suggestions for further reading: Waxman SG. Clinical Neuroanatomy. Lange, New York, 2013. O’Brien MD. Aids to the Examination of the Peripheral Nervous System.Saunders, Edinburgh, 2010.

K25928_C005.indd 151

10/01/2017 10:53

Chapter

6

Localization Primer

K25928_C006.indd 153

10/01/2017 10:51

154 Chapter 6 Localization Primer

Our algorithm for localization is shown in Fig. 6.1.

Figure 6.1 Localization Algorithm

K25928_C006.indd 154

CNS: central nervous system; LMN: lower motor neuron; PNS: peripheral nervous system; UMN: upper motor neuron.

10/01/2017 10:51

Localization Algorithm

155

We’ve reviewed the neuroanatomy of the cerebrum, brainstem and cranial nerves (CN), spinal cord as well as the peripheral nervous system (PNS). Your road map of the nervous system is complete, and you can start to practice localizing in the accompanying 25 cases. But before you do, let’s discuss how to go about applying the neuroanatomy you’ve learned. It’s easy to get overwhelmed with information when you are reading a neurology case, and thus it can be confusing to know where to start in the process of localization. Should you focus on the patient’s visual field defects, or his sensory complaints, or perhaps his hemibody weakness? While there are multiple ways to approach any neurology problem, some localization algorithms are much more straightforward than others, and the one we present here can be applied to the vast majority of neurological cases you will see. Our first step is to decide if the lesion is in the CNS (UMN) or the PNS (LMN). Then we localize longitudinally; we decide if the lesion is at the level of the cortex, internal capsule, brainstem, or spinal cord for the CNS, or at the nerve root, plexus, or nerve for the PNS. Deciding on this produces our initial localization (i.e., cervical spinal cord). Once we have decided on this, we can draw that level in cross section and then decide what components are affected, which produces our final localization (i.e., the right half of the spinal cord at C6).

The localization process always begins by answering the following question: Is the patient’s weakness upper motor neuron (UMN), lower motor neuron (LMN) or both? Categorizing the patient’s weakness into either UMN or LMN is the easiest way to determine if the lesion lies in the central nervous system (CNS) or the PNS, respectively. Depending on the answer to this question, our algorithm branches into three paths:

K25928_C006.indd 155

10/01/2017 10:51

156

Chapter 6 Localization Primer

Path 1: the patient has UMN weakness.

Step 1: Does the pattern of long tract symptoms suggest a localization? This is a very important question to answer, as the pattern of long tract signs can often yield an Immediate Localization (such as the crossed signs of the brainstem or the sensory level of the spinal cord). Even if an Immediate Localization isn’t revealed, a much shorter list of localizations can often be produced from this information.

Step 2: What is the highest level of dysfunction? Determine the highest neuroanatomical level possible. See if the case involves any of the higher functions of the cortex (aphasia, apraxia, or agnosia), CN palsies (brainstem) or sensory levels (spinal cord). For example, if the patient presents with right sided weakness and also has aphasia then the neuroanatomical level is the cortex. However, if the patient instead has right sided weakness and a CN III palsy, the neuroanatomical level is the brainstem. Similarly right sided weakness and a sensory level would localize to the spinal cord. At this point we will have finished localizing longitudinally and have our initial localization.

Step 3: What is affected? Since we now know the initial localization, we can draw that level in cross section and determine what components are affected. This will give us our final localization.

K25928_C006.indd 156

10/01/2017 10:51

Localization Algorithm

157

Path 2: the patient has LMN weakness. If the patient has only LMN weakness then we know the lesion lies in the PNS. Localizing in the PNS is very different than in the CNS, and is more of a “trial and error” comparison. Possible localizations in the PNS include the nerve root (a radiculopathy), the plexus (plexopathy), and the nerve (neuropathy).

Step 1: Do the symptoms correspond to a single myotome and dermatome? Figure 5.5 is our table of dominant myotomes. These muscles serve as a screen to see if the patient has a radiculopathy. If only one of the muscles listed in Fig. 5.5 is affected, go to Fig. 5.15 and find the root’s corresponding dermatome. If the patient’s sensory loss matches this dermatome then you can safely conclude the patient has a radiculopathy. If the patient has more than one of the muscles in Fig. 5.5 affected, or the sensory loss doesn’t match the dermatome, proceed to Step 2.

Step 2: Do the symptoms correspond to a single nerve? Now see if the affected muscles and sensory loss correspond to a single nerve. If they do, then the next step is to figure out where the lesion is along the path of the nerve. To do so, begin distally, and walk up the length of the nerve, until you come across a muscle that has not been affected by the lesion. If the symptoms do not match to a radiculopathy or neuropathy, then by process of elimination, the lesion must lie in the plexus.

K25928_C006.indd 157

10/01/2017 10:51

158

Chapter 6 Localization Primer

Path 3: the patient has both UMN and LMN weakness. This can only be produced by a lesion of the spinal cord. Based on whether both the arms and the legs are affected, determine whether the lesion lies in the cervical, thoracic, lumbar or sacral spinal cord.

Don’t worry if the above algorithm seems a little opaque; solutions are provided for all of the cases so you’ll see how we employ it. Note that the above algorithm assumes that a focal lesion exists. The nervous system, especially the PNS, is vulnerable to a whole host of diffuse disease processes that can affect the entire nervous system at once, resulting in a general, gradual loss of function. In this case the idea is to localize to the component of the nervous system (i.e., peripheral nerves vs. spinal cord) that is diseased, as this greatly narrows down the differential diagnosis. No algorithm will work 100% of the time and exceptions always exist (don’t worry, we will cover the most common ones in the Cases section). However, following the logic laid out above will always give you a good idea of where the lesion is in the vast majority of cases.

K25928_C006.indd 158

10/01/2017 10:51

Case Primer

159

Case Primer Before you sink your teeth into the cases, we need briefly to talk about how to do a typical neurological screening exam. Depending on the clinical situation you’re faced with, a much more in-depth exam may need to be performed. The format below assumes you have a patient who is not confused and is cooperative (by no means does this occur all the time, particularly in the early hours of the morning when on call). The neurological exam is unique in medicine as it is quite dependent on patient cooperation, and sometimes simple inspection is all that can be accomplished. However, given an ideal patient, a screening examination tests the following components:

Language: The patient may be aphasic. In order to classify it, note the patient’s fluency and whether they comprehend verbal commands. Finally, see if they can repeat a sentence.

Cranial nerves: A formal exam usually begins by assessing visual acuity using a Snellen eye chart, but in routine screening exam this is usually omitted. CN II is tested by assessing for a visual field defect. CN III, IV and VI are assessed by seeing if the eyes can move in all the directions of gaze. The other components of CN III are checked by seeing if there is ptosis of the eyelid and whether the pupil reacts to light (note this also tests CN II). CN V is checked by seeing if sensation is intact in the face. CN VII is assessed by noting whether the patient has a facial droop. Determine if the facial droop involves the forehead (LMN facial droop) or not (UMN facial droop). CN VIII is not routinely assessed in a screening exam. CN IX and X are assessed together by seeing if the patient’s palate raises symmetrically. CN XI is tested by seeing if they have full power in head turning and shoulder shrug. Finally, CN XII is tested by seeing if, when the tongue is stuck out, it is straight, implying normal power, or if it is deviated to one side, implying weakness.

Motor system: Before assessing power, inspect the limbs briefly and look for atrophy and fasciculations. Then assess tone, remembering that increased tone is a sign of UMN weakness. Then power can be assessed; the first part of Fig. 6.2 shows what muscle groups are routinely checked in a typical screening exam. However, if the patient complains of very specific weakness, such as hand or foot weakness, more muscle groups may need to be tested.

K25928_C006.indd 159

10/01/2017 10:51

160

Chapter 6 Localization Primer

Figure 6.2 Muscle Testing in a Screening Neurological Exam

Some individual figures adapted from Waxman SG. Clinical Neuroanatomy. Lange, New York, 2013.

K25928_C006.indd 160

10/01/2017 10:51

Muscle Testing in a Screening Neurological Exam

161

Power is assessed using the Medical Research Council Grading of Muscle Power Scale (MRC Scale), which is shown in the bottom half of Fig. 6.2. The MRC Scale does an admirable job of trying to evaluate power in as objective a fashion as possible, but ultimately is still subject to considerable bias. For example, when examining a 91-yearold the “full resistance” applied by the examiner is often significantly less than that which is applied when examining a 24-year-old, and may show considerable interexaminer variability. Importantly, for a muscle to be graded at a certain level of power, it must be able to move against the resistance applied by the examiner across the entire range of motion of the muscle. This is a common source of grading error. For example, if a patient can only move their bicep the first 20° against gravity, then they cannot be scored as an MRC grade 3. Many clinicians also employ unofficial subgrades, the most common of which is the 4 -, 4.0 and 4+ designation, which indicates that the examiner is applying mild, moderate and significant, but still not full, resistance, respectively. Since this is so common we have chosen to use this convention in our Cases (Chapter 7). Finally, the reflexes are examined.

Sensory system: The spinothalamic tract (ST) (pain and temperature) and dorsal columns (DC) (vibration and proprioception) should be checked in all four limbs. There is no need to test both modalities carried in a single tract; typically pain is assessed for the ST (using a pin, to see if the patient recognizes it is sharp), and vibration for the DC (using a tuning fork).

Coordination and gait: Finger to nose and heel to shin testing is examined first. These tests can be difficult to interpret if the patient is weak. Finally, if the patient is able, it is often very instructive to watch their gait to look for any subtle abnormalities.

K25928_C006.indd 161

10/01/2017 10:51

162

Chapter 6 Localization Primer

Figure 6.3 Muscle Table

K25928_C006.indd 162

10/01/2017 10:51

Muscle Table

163

Figure 6.3 lists many of the muscles that can be assessed on neurological examination. When recording the power of muscle groups, clinicians usually use a chart format. An unfortunate lack of convention has developed wherein some clinicians will write down the name of the muscle tested (i.e., biceps) and others will instead identify the muscle based on its function (i.e., elbow flexor). Usually the function of the muscle is written down because it can be shortened (i.e., elbow flexor becomes EF). However, notable exceptions occur and it is ultimately arbitrary. You need to be familiar with all potential names, so we have included this chart so you can always see what muscle groups we reference in the cases.

K25928_C006.indd 163

10/01/2017 10:51

164

Chapter 6 Localization Primer

Localization Key Figure 6.4

ACA: anterior cerebral artery; FEF: frontal eye fields; MCA: middle cerebral artery; TCM: transcortical motor; UMN: upper motor neuron.

K25928_C006.indd 164

10/01/2017 10:51

Localization Key

165

Figure 6.5

PCA: posterior cerebral arteries; UMN: upper motor neuron.

K25928_C006.indd 165

10/01/2017 10:51

166

Chapter 6 Localization Primer

Figure 6.6

INO: internuclear ophthalmoplegia; LMN: lower motor neuron; UMN: upper motor neuron.

K25928_C006.indd 166

10/01/2017 10:51

Localization Key

167

Figure 6.7

LMN: lower motor neuron; UMN: upper motor neuron.

K25928_C006.indd 167

10/01/2017 10:51

168

Chapter 6 Localization Primer

Figure 6.8

LMN: lower motor neuron; UMN: upper motor neuron.

K25928_C006.indd 168

10/01/2017 10:51

Localization Key

169

Figure 6.9

AHC: anterior horn cell; LMN: lower motor neuron; UMN: upper motor neuron.

K25928_C006.indd 169

10/01/2017 10:51

170

Chapter 6 Localization Primer

Figure 6.10

AHC: anterior horn cell.

K25928_C006.indd 170

10/01/2017 10:51

Localization Key

171

Figure 6.11

AHC: anterior horn cell; LMN: lower motor neuron; UMN: upper motor neuron.

K25928_C006.indd 171

10/01/2017 10:51

172

Chapter 6 Localization Primer

Figure 6.12

K25928_C006.indd 172

10/01/2017 10:51

Localization Key

173

Figure 6.13

LMN: lower motor neuron; UMN: upper motor neuron. Some individual figures adapted from Waxman SG. Clinical Neuroanatomy. Lange, New York, 2013.

K25928_C006.indd 173

10/01/2017 10:51

174

Chapter 6 Localization Primer

Figure 6.14

Some individual figures adapted from Waxman SG. Clinical Neuroanatomy. Lange, New York, 2013.

K25928_C006.indd 174

10/01/2017 10:51

Localization Key

175

Figure 6.15

Some individual figures adapted from Waxman SG. Clinical Neuroanatomy. Lange, New York, 2013.

K25928_C006.indd 175

10/01/2017 10:51

176

Chapter 6 Localization Primer

Figure 6.16

Some individual figures adapted from Waxman SG. Clinical Neuroanatomy. Lange, New York, 2013.

K25928_C006.indd 176

10/01/2017 10:51

Part Two The Cases

K25928_C007.indd 177

12/01/2017 12:18

Case 1: The 72-year-old woman who talked only gibberish

179

Case 1: The 72-year-old woman who talked only gibberish You are asked by your friends on the Internal Medicine team to come down and see a 72-year-old woman who is in their clinic for routine follow-up of her diabetes. She lives alone. When her daughter went to pick her up for her appointment, she found the patient “talking gibberish,” but decided not to take her to the Emergency Department, since they were “going to go see a doctor anyway.” When you see the patient she is now agitated. She says very little and what she does say is nonsensical. She cannot repeat, nor does she obey your instructions. This frustrates your neurological examination. However, you persevere and assess her cranial nerves (CN). She appears to have lost her right visual field in both eyes. Her eyes are driven to the left and she cannot look to the right. She has a right sided facial droop that spares her forehead. Her tongue and palate appear midline. Motor examination reveals increased tone on the right side, with 3+ reflexes. Her toe is upgoing on the right. Formal power testing is impossible, but you notice she is not moving her right side. She is also not responsive to pain on the right side. Coordination is impossible to assess. Where is the lesion? What is going on here?

K25928_C007.indd 179

12/01/2017 12:18

180

Case 1: Findings

Case 1: Findings

AF: arcuate fasciculus; MCA: middle cerebral artery.

K25928_C007.indd 180

12/01/2017 12:18

Case 1: Solution

181

Case 1: Solution 1. Does the patient have upper motor neuron (UMN) weakness, lower motor neuron (LMN) weakness or both? Our patient has increased reflexes, increased tone and an upgoing toe, all signs of an UMN type weakness. Thus, we know that the lesion lies in the central nervous system (CNS). 2. Does the pattern of long tract symptoms suggest a localization? In this case, our patient has weakness of the right face, arm and leg. Her facial weakness does not involve the forehead, thus this is an UMN type facial droop. Our sensory testing is less than ideal, but we do know she has impairment to pinprick on the right, indicating the spinothalamic tract (ST) is damaged. Unfortunately this pattern is nonspecific, and possible localizations include the cortex, internal capsule and brainstem. 3. What is the highest level of dysfunction? Reviewing our symptoms, we see that our patient has an UMN facial droop. We therefore know that the lesion must be above the level of the pons, or our patient would have a LMN facial droop from involvement of the facial nucleus. Our patient has lost the right sided aspect of her visual field in both eyes; she has a right sided homonymous hemianopsia (RHH). In addition, she appears to be globally aphasic, as her fluency, comprehension and ability to repeat are all impaired; aphasia is one the symptoms that is an Immediate Localization to the cortex. Thus, the cortex is our initial localization, and we now must decide what parts of it are affected. 4. What is affected? Our patient has right sided UMN type weakness and right sided spinothalamic loss. Since these tracts decussate at the level of the medulla, the left motor cortex and sensory cortex must be involved. Recall that the frontal eye fields (FEF) drive the eyes to the contralateral side. Since our patient’s eyes are deviated to the left, this means that the right FEF are unopposed; the left FEF have also been lesioned. The global aphasia means that both Broca’s area and Wernicke’s area are affected. The RHH can only be produced by a lesion of the visual pathways that is posterior to the optic chiasm (Fig. 3.5). Putting it all together, we see that nearly all of the left parietal and temporal lobe is involved, as well as some of the frontal lobe. Reviewing Fig. 2.14, we see that this area corresponds to the vascular territory supplied by the middle cerebral artery (MCA). Indeed, this patient was unfortunate enough to have suffered a complete occlusion of her left MCA.

K25928_C007.indd 181

12/01/2017 12:18

182

Case 1: Clinical Pearl

Figure 7.1 Causes of Stroke

Mozaffarian D, et al. Heart disease and stroke statistics – 2015 update, Circulation 2015;131:e310.

K25928_C007.indd 182

12/01/2017 12:18

Case 1: Clinical Pearl

183

Clinical Pearl: Causes of Stroke A stroke, also called a cerebral infarction, is caused by an interruption of blood flow to an area of the brain and results in the permanent loss of brain tissue. While stroke is the fifth leading cause of death in the USA, it is the number one cause of adult disability. Understandably, the economic impact of this disability is tremendous; Americans spent over $34 billion on the direct cost of treating stroke in 2013. This number, while impressive, ignores the enormous costs associated with the loss of productivity of stroke victims; many are unable to return to work or have to do so on a part time basis and must rely on social assistance. The interruption of blood flow that causes a stroke can either be the result of the occlusion of a blood vessel, causing ischemia or the result of a rupture of a blood vessel, causing a hemorrhage. 80% of strokes are ischemic in origin, and 20% result from hemorrhage. Hemorrhagic strokes are usually due to focal bleeding from rupture of the brain tissue and associated blood vessels. The patient usually has a history of poorly controlled blood pressure, leading to weakening of the arterial wall. Ischemic strokes are usually caused by a blood clot that occludes the artery. If the blood clot was generated near the site of the stroke it is called a thrombus. If the blood clot broke off from another location and travelled to the site of the stroke it is called an embolus. The difference between a thrombus and an embolus is somewhat arbitrary and is of little practical significance. There are several major etiologies of ischemic stroke. Carotid artery disease and cardioembolism each account for 25% of ischemic stroke cases. While carotid stenosis can serve as a source for embolism in ischemic stroke, it should be noted that no neurological symptoms are caused by the stenosis itself; “neurological angina” does not exist, unlike chest pain due to stenosed cardiac arteries. Rarely, ischemic strokes can be caused by insufficient blood pressure to perfuse the brain, which is usually associated with cardiac arrest. The most common cause of cardiac embolism is from arrhythmias like atrial fibrillation; fibrillation results in static blood which naturally clots, forming an embolus. This embolus is then pumped by the heart into the arterial circulation. Other causes of embolus formation include prosthetic valves, congestive heart failure, and myocardial infarction. If the underlying problem is not correctable, the patient will need to take anticoagulation medication. Lacunar stroke, also called small vessel disease, refers to thrombus formation in the small penetrating arteries of the cerebral vascular system, usually coming off the MCA or basilar arteries. These will be covered later. Note that, despite aggressive investigation, about 20% of strokes remain cryptogenic, meaning no cause can be found.

K25928_C007.indd 183

12/01/2017 12:18

Case 2: The 79-year-old man who whose legs felt funny

185

Case 2: The 79-year-old man who whose legs felt funny You are asked to see a 79-year-old man who has had a recent string of unfortunate luck. He developed intense abdominal pain and was diagnosed with a ruptured abdominal aortic aneurysm for which he was operated on emergently. He survived the operation but had a difficult postoperative course and has spent the last week in the intensive care unit (ICU) in a medically induced coma. When he awoke from the coma yesterday, he was thankful to be alive but quickly realized he could not move his legs. When the ICU team removed his catheters they noted he had both bowel and bladder incontinence. He complains to the nurses that his legs feel “funny.” When you examine him, you find his language to be normal. His pupils, visual fields, and eye movements are also normal. He does not have a facial droop and his tongue is not deviated. Motor examination of the arms is also normal. However, you find a spastic tone in both legs. His reflexes are 3+ in the knee bilaterally, and 4+ in the ankle bilaterally (which, by definition, induces subsequent clonus). His power examination is listed below, but he is barely antigravity in the proximal part of both legs. Toes are upgoing bilaterally. His vibration sense is intact everywhere. Pinprick is intact in the arms, but he states the pin feels dull in both legs. You run the pin down his trunk and he states that all of a sudden it feels dull around the umbilicus. However, you find that his vibration sense is intact everywhere. Coordination was not tested given the weakness.

Power Examination Right

Left

Deltoid

5

5

Bicep

5

5

Tricep

5

5

WE

5

5

FE

5

5

HF

3

3

KE

3

3

KF

3

3

ADF

2

2

APF

2

2

Where is the lesion? What is going on here?

K25928_C007.indd 185

12/01/2017 12:18

186

Case 2: Findings

Case 2: Findings

K25928_C007.indd 186

12/01/2017 12:18

Case 2: Solution

187

Case 2: Solution 1. Does the patient have UMN weakness, LMN weakness or both? Our patient has increased reflexes, spastic tone and upgoing toes, which are all signs of an UMN lesion. Thus, we know that the lesion lies in the CNS. 2. Does the pattern of long tract symptoms suggest a localization? Our patient has bilateral UMN type leg weakness. This has two main localizations and they lie at opposite ends of the CNS. The first is a lesion to the thoracic or lumbar spinal cord. The second is a cortical lesion that involves the medial aspect of both motor cortices; this would not cause face and arm weakness as the neurons to these areas lie on the lateral part of the motor cortex (Fig. 2.1). In order to differentiate between the two, we have to hope for a clue in the sensory exam. In this case we are quite fortunate as the sensory examination reveals an Immediate Localization; our patient has a sensory level. He complains of loss of pinprick below the level of the umbilicus. A sensory level can only be produced by a spinal cord lesion (see Chapter 4). 3. What is the highest level of dysfunction? Our patient has a sensory level to pinprick at the umbilicus; consulting our dermatome map (Fig. 5.15) shows this area is innervated by T10. Thus our initial localization is the spinal cord at the level of T10. 4. What is affected? Since we now know that we are in thoracic spinal cord, we can draw it in cross section. Our patient has UMN weakness in both legs, so we know that both corticospinal tracts are involved. He does not have any loss of vibration, so the dorsal columns (DC) are spared. However, the loss of pinprick below the level of T10 corresponds to lesions in both STs. Joining these areas together, we see that he has almost two-thirds of his cord involved. Our final localization is the anterior two-thirds of the spinal cord at T10. The pattern described is known as anterior cord syndrome as it is typically caused by an anterior spinal artery stroke. In this case, prolonged clamping of the aorta during the emergency surgery likely caused unavoidable ischemia to the spinal cord.

K25928_C007.indd 187

12/01/2017 12:18

188

Case 2: Clinical Pearl

Figure 7.2 Other Patterns of Intrinsic Spinal Cord Disease

DC: dorsal column; ST: spinothalamic tract; SACD: subacute combined degeneration; LMN: lower motor neuron; UMN: upper motor neuron.

K25928_C007.indd 188

12/01/2017 12:18

Case 2: Clinical Pearl

189

Clinical Pearl: Other Patterns of Intrinsic Spinal Cord Disease For reasons that are incompletely understood, the spinal cord is particularly vulnerable to many different kinds of nutritional and infectious insults. We will cover a few of the more common ones; they are shown in Fig. 7.2. Tabes dorsalis Tabes Dorsalis is a late complication of syphilis caused by infection with Treponema pallidum. It typically presents anywhere from 3 to 30 years after the initial infection and affects the dorsal root ganglion (DRG), posterior root and the DC. Patients initially complain of painful paresthesias (tingling sensation) due to DRG dysfunction. Soon thereafter they develop a sensory ataxia from the lack of proprioception, due to the involvement of the DC. On examination, these patients have poor vibration sense, and are areflexic due to interruption of the reflex arc from the DRG lesion. Tabes dorsalis has become rare with the advent of modern antibiotics, however it has experienced a reemergence due to the human immunodeficiency virus (HIV) epidemic. Subacute combined degeneration Subacute combined degeneration (SACD) is caused by a deficiency of vitamin B12, but both copper and zinc deficiencies produce a very similar clinical picture. Vitamin B12 deficiency first affects the peripheral nerves, causing a painful neuropathy. As it progresses it then affects the DC causing a sensory ataxia. Eventually, the corticospinal tract becomes involved, resulting in an UMN type weakness. On examination, patients have weakness and spasticity from the UMN type lesion. However, they are areflexic because by the time the corticospinal tract is involved, the diffuse neuropathy is so advanced that muscles can no longer respond to the reflex stimulus. Poliomyelitis While the poliovirus endemic of the first half of the 20th century is over, poliomyelitis remains a common topic for examinations. Poliomyelitis causes an acute flaccid paralysis, due to attack on the anterior horn cell (AHC) in the spinal cord. This causes a typical LMN picture; atrophy and wasting of muscles, decreased tone, and significant weakness. Due to the extreme weakness, patients lose their reflexes, as the muscles can no longer respond to the reflex stimulus. Initial treatment was mainly supportive and led to the development of the infamous “iron lung,” until a vaccine became available in the 1950s.

K25928_C007.indd 189

12/01/2017 12:18

Case 3: The 21-year-old man who was near death

191

Case 3: The 21-year-old man who was near death You have decided to do some locum work to help pay down your medical school debt and are stationed in a remote region of Northern Ontario in Canada. All of a sudden a horde of twenty–somethings burst through the Emergency Department’s doors and demand you immediately see their friend, as they are convinced he is near death. You find a 21-year-old man who seems to be clutching his right arm in pain, but is able to tell you his story. He got lost on a camping trip with his fellow overzealous twenty–somethings, and climbed a tree to get his bearings (you don’t bother to ask how much alcohol was involved, as the tiny Emergency Department room now reeks of it). Unfortunately he fell out of the tree but he was able to briefly grab a branch with his right hand, slowing his descent. He rejoined his friends several hours later and then realized that he couldn’t move his right arm (which you suspect he discovered when he went to grab another beer bottle). At this point, one of his friends noticed his pupils were not equal and insisted he come to the Emergency Department. Initial x-rays are negative for any fractures, so you wonder about a neurological problem. His language is normal. He has full visual fields, but his right pupil is 2 mm wide and his left pupil is 3 mm wide. You turn off the lights and the right stays 2 mm, and the left becomes 4 mm. He has full extraocular eye movements. Sensation in the face is normal and he does not have a facial droop. His palate and tongue are midline. You move onto the arm and notice that the right shoulder is red and inflamed. In addition, the right arm seems fixed in an odd posture with the wrist hyperflexed. Formal power testing is recorded below. His bicep reflex is decreased, but all other reflexes are normal. His tone was decreased in the right arm but normal elsewhere. He had downgoing toes. His sensory examination shows a loss of pinprick over the entire lateral aspect of the right arm. Coordination was not tested in the left arm, due to loss of strength, but was normal everywhere else. Power Examination Right

Left

Deltoid

3

5

Bicep

4-

5

Tricep

5

5

WE

3

5

WF

5

5

FE

5

5

DI

5

5

FF

5

5

EP

5

5

K25928_C007.indd 191

Where is the lesion? What is going on here?

12/01/2017 12:18

192

Case 3: Findings

Case 3: Findings

Some individual figures adapted from Waxman SG. Clinical Neuroanatomy. Lange, New York, 2013.

K25928_C007.indd 192

12/01/2017 12:18

Case 3: Solution

193

Case 3: Solution 1. Does the patient have UMN weakness, LMN weakness or both? Our patient complains of arm weakness in the context of decreased right arm tone and a decreased biceps reflex. This strongly suggests a LMN type problem, and we can conclude that our lesion lies in the peripheral nervous system (PNS). Possible localizations include a radiculopathy, plexopathy and neuropathy. 2. Do the symptoms correspond to a single myotome and dermatome? Checking our table of dominant myotomes (Fig. 5.5) we see that the deltoid is innervated by C5. However, both the bicep and extensor carpi are innervated by C6. This implies that dysfunction of a single nerve root can’t cause our patient’s presentation. 3. Does the pattern of symptoms correspond to a single nerve? In this case the three muscles that are weak correspond to three different nerves; the deltoid is innervated by the axillary nerve, the bicep is innervated by the musculocutaneous nerve and the extensor carpi are innervated by the radial nerve. We can safely conclude that our patient’s symptoms are not due to a neuropathy. By process of elimination we conclude that our patient has a brachial plexus problem. Our patient’s muscle weakness and sensory loss are explained by a lesion to both C5 and C6. The only place where C5 and C6 run together without the involvement of C7, C8 and T1 occurs in the Upper Trunk, which is our final localization. This is also called an Erb–Duchenne Palsy, and is most commonly seen in newborns due to a traumatic delivery affecting the plexus, but can happen whenever traction is placed on the shoulder, as in our case. Paralysis of the deltoid, bicep and extensor carpi cause the “Bellman’s tip” posturing of the arm (so called, as back in the 1870s when it was initially described, it was a social faux pas for the bellman or waiter to be facing the patron when receiving a tip). Now don’t worry, we haven’t forgot about the pupils. Our patient has an anisocoria (unequal pupil size) whose difference is greater in dark than in light. This dysfunction is known as Horner’s syndrome, which we will cover more in detail soon. Horner’s syndrome is caused by a lesion to the sympathetics, and is another clue that the lesion, in this case, must lie in the brachial plexus.

K25928_C007.indd 193

12/01/2017 12:18

194 Case 3: Clinical Pearl

Figure 7.3 Autonomic Neurotransmitters

K25928_C007.indd 194

Ach: acetylcholine ; NE: norepinephrine. 12/01/2017 12:18

Case 3: Clinical Pearl

195

Clinical Pearl: Autonomic Neurotransmitters As we mentioned in Chapter 1, neurons communicate with each other at a synapse through various chemicals called neurotransmitters. Sadly, the nomenclature about these neurotransmitters is both unwieldy and largely redundant, which often causes confusion. We’ll turn our attention to them now. There are two principal neurotransmitters employed by the nervous system; acetylcholine (Ach) and norepinephrine (NE). If a neuron releases Ach it is said to be cholinergic. If it releases NE it is said to be adrenergic. Let’s consider the motor, parasympathetic and sympathetic branches of the nervous system. We’ll begin with the motor system as it’s the simplest. The LMN innervates a muscle. To communicate with it, the LMN releases neurotransmitters into the neuromuscular junction (NMJ). This neurotransmitter is Ach, and it causes the muscle to contract. Recall that the autonomic system is composed of two neurons; the preganglionic neuron and the postganglionic neuron. The parasympathethic system is exclusively cholinergic; both the preganglionic neuron and the postganglionic neuron employ Ach. The sympathetic system has evolved to be more complicated. The preganglionic neuron uses Ach as its neurotransmitter. While the vast majority (about 98%) of the postganglionic neurons of the sympathetic system uses NE as their neurotransmitter. However, some postganglionic neurons that innervate sweat glands use Ach as their neurotransmitter. Waiting for chemicals to diffuse across the NMJ is a slow process. The adrenal glands produce NE which is needed immediately upon a dangerous “fight or flight” situation occurring. Thus, for the adrenals, the sympathetic system only employs one single neuron. That neuron releases Ach to the adrenals, which then release NE directly into the bloodstream. Neurotransmitter receptors are named according to what chemicals they respond to. There are actually two Ach receptors; a nicotinic acetylcholine receptor and a muscarinic acetylcholine receptor; while they both respond principally to Ach, they also respond weakly to nicotine and muscarine, respectively. The motor system’s Ach receptor is the nicotinic subtype. For the autonomic nervous system the preganglionic neuron is always nicotinic. The postganglionic neuron in the parasympathethic system is muscarinic. The postganglionic neuron that innervates the sweat glands is muscarinic. Finally, the single neuron that innervates the adrenal gland is nicotinic. Knowing which Ach receptors are nicotinic and which are muscarinic is important because certain drugs preferentially affect one type of Ach receptor.

K25928_C007.indd 195

12/01/2017 12:18

Case 4: The 71-year-old man who slurred his words

197

Case 4: The 71-year-old man who slurred his words You are about to sit down to a difficult conversation with a patient’s family when you are paged over to the cardiac catheterization lab. There, a particularly pale and distressed cardiology fellow tells you that the 71-year-old man he has just performed a coronary angiogram on is now complaining of slurred speech. When you talk to the patient, you confirm the cardiologist’s finding of dysarthria. Despite struggling with pronunciation you find his speech to be coherent and said at an essentially normal rate. He is able to understand everything you say to him and is able to repeat. Examining his CN you find his pupils and eye movements are normal, and he has full visual fields. He does not have a facial droop and facial sensation is normal. However, when he sticks his tongue out, you find that it is deviated sharply to the left. Formal power examination is recorded below, but he is weak on the right. His reflexes are 3+ on the right, and he has an upgoing right toe. Tone is normal. Sensory examination to pinprick was normal. However, he complains of right sided vibration loss in the arms and the legs. Coordination testing was felt to be normal given the patient’s weakness.

Power Examination Right

Left

Deltoid

4

5

Bicep

4

5

Tricep

4-

5

WE

4-

5

FE

4-

5

HF

4-

5

KE

4-

5

KF

4-

5

ADF

4-

5

APF

4

5

Where is the lesion? What is going on here?

K25928_C007.indd 197

12/01/2017 12:18

198

Case 4: Findings

Case 4: Findings

K25928_C007.indd 198

12/01/2017 12:18

Case 4: Solution

199

Case 4: Solution 1. Does the patient have UMN weakness, LMN weakness or both? Our patient has weakness, increased reflexes and an upgoing toe, all indicative of UMN type problem; our lesion lies in the CNS. Likely, the lesion is so acute that he has not had the chance to develop increased tone, which can take up to 48 hours. 2. Does the pattern of long tract symptoms suggest a localization? Our patient’s weakness is caused by a corticospinal tract problem and his vibration loss indicates a DC problem. Unfortunately, since both of these symptoms occur on his right side, they are quite nonspecific. Possible localizations include the cortex, internal capsule, brainstem and cervical spine. 3. What is the highest level of dysfunction? A cervical spine lesion could not account for tongue deviation, so we know our lesion has to be higher than that level. Our patient does not have any signs, such as aphasia, or apraxia to suggest a cortical lesion. We must return to the tongue, which is innervated by CN XII. Recall from Fig. 3.15 that the hypoglossal nucleus of CN XII is dually innervated. Thus, only a LMN lesion of CN XII will produce clinical symptoms. The hypoglossal nucleus lies in the medulla, which is our initial localization. If you look carefully, you’ll realize our patient’s pattern of weakness is an Immediate Localization of the brainstem. Our patient’s tongue deviates to the left, which means that it is the left part of his tongue that is weak. However, he has right sided weakness of the arm and leg. This is an example of crossed signs and can only be produced by a brainstem lesion (see Chapter 3). 4. What is affected? We can now draw the medulla in cross section and begin to fill in affected components. Our patient has UMN weakness in the right arm and leg so the left corticospinal tract is involved. Similarly, he has loss of vibration in the right arm and leg, so the left DC must be affected as well. He has left sided tongue weakness, so the left hypoglossal nucleus is lesioned. Putting it all together, our final localization is the medial aspect of the left medulla; indeed, the above is known as medial medullary syndrome. An embolus was likely dislodged during the coronary angiogram and shot up into the vertebral artery, causing a stroke.

K25928_C007.indd 199

12/01/2017 12:18

200

Case 4: Clinical Pearl

Figure 7.4 Locked-in Syndrome

MLF: medial longitudinal fasciculus.

K25928_C007.indd 200

12/01/2017 12:18

Case 4: Clinical Pearl

201

Clinical Pearl: Locked-in Syndrome Locked-in syndrome is a devastating condition in which patients are fully aware but have nearly complete paralysis of the face, arms and legs. Patients lose all voluntary muscle control with the sole exception of the ability to move their eyes vertically. This subtle finding can easily be missed by inexperienced clinicians who erroneously conclude the patient is in a coma. Classically, locked-in syndrome is caused by bilateral lesions of the medial pons, which is somewhat similar to the anatomy we have just described in the medial medullary syndrome above. As an exercise, try to predict the structures that would be involved by using the Rule of 4. The following structures would be affected, as shown in Fig. 7.4: 1) Bilateral corticospinal tracts – leading to an UMN type paralysis of the arms and the legs. 2) Bilateral corticobulbar tracts – leading to a LMN type paralysis of all the subsequent CN in the medulla including: • CN IX and CN X – resulting in the inability to swallow and talk. • CN XI – resulting in the inability to shrug and move the head. • CN XII – resulting in the inability to move the tongue. 3) Bilateral involvement of CN VII – resulting in LMN facial weakness. 4) Bilateral involvement of CN VI and the medial longitudinal fasciculus (MLF) – leading to the inability to generate horizontal eye movements. Vertical eye movements, which are initiated by muscles controlled by CN III in the midbrain, are unaffected. 5) Bilateral involvement of the DC – resulting in loss of proprioception and vibration. Note that the ST is spared, allowing the patient to feel pain and temperature. Locked-in syndrome is a terrible clinical condition caused by an opportune lesion that affects a very densely populated neuroanatomical area. These lesions are typically caused by a stroke. Locked-in syndrome has a dismal prognosis with 65% of patients dying within 1 year due to respiratory failure.

K25928_C007.indd 201

12/01/2017 12:18

Case 5: Your 20-year-old brother, who had too much to drink

203

Case 5: Your 20-year-old brother, who had too much to drink After getting the results of his MCAT, your brother, confident his future is now assured, imbibed a significant amount of alcohol. You find him the next morning passed out, with his right arm draped over a chair. Being the merciful soul you are, you decide to wake him up and make him breakfast. When he does finally rouse, in addition to having a massive hangover, he is shocked to find that he cannot use his right hand to grab the cup of coffee you have generously brewed him. Startled, he tried to look at his right palm, but couldn’t turn his right forearm. In between sips of tea, you examine him carefully. His language examination is normal, though his pronunciation is still somewhat slurred, but you chalk this up to his party the night before. He has full visual fields, and his pupils are equal and reactive to light. He has full eye movements, with no facial droop and his tongue and palate are midline. His formal power examination is shown below, but his legs are normal strength. His reflexes are normal bilaterally and he has downgoing toes. You feel confident that his tone is decreased in the right arm. His sensory examination is normal, with the exception that he could not feel the pin in the lateral aspect of the dorsal arm from the elbow to the hand. You feel that the problems he has with coordination and gait are largely due to the alcohol, and dismiss them. Power Examination Right

Left

Deltoid

5

5

Bicep

5

5

Tricep

5

5

SUP

3

5

PRO

5

5

WE

3

5

WF

5

5

FE

3

5

DI

5

5

FF

5

5

EP

3

5

ADP

5

5

FP

5

5

OP

5

5

ABP

5

5

Where is the lesion? What is going on here?

K25928_C007.indd 203

12/01/2017 12:18

204

Case 5: Findings

Case 5: Findings

Some individual figures adapted from Waxman SG. Clinical Neuroanatomy. Lange, New York, 2013.

K25928_C007.indd 204

12/01/2017 12:18

Case 5: Solution

205

Case 5: Solution 1. Does the patient have UMN weakness, LMN weakness or both? Our patient has weakness of his right arm. An UMN type lesion is certainly possible, but it would have to be rather small to affect the arm only. In addition, he does not have any of the stigmata of UMN type weakness; his reflexes and tone are normal and his toes are downgoing. It is thus more likely that he has a LMN type problem. Since his symptoms arose overnight they are too acute to cause some of the findings associated with LMN problems, such as fasciculations and atrophy. Potential localizations include a radiculopathy, plexopathy or neuropathy. 2. Do the symptoms correspond to a single myotome and dermatome? Let’s check our table of dominant myotomes (Fig. 5.5); the only muscle on this table that our patient has affected is his extensor carpi, which are innervated by C6. However, the biceps are also innervated by C6, and our patient has normal power here. Given this we must conclude that C6 is unaffected, and our patient does not have any dysfunction of the nerve roots. 3. Does the pattern of symptoms correspond to a single nerve? Overall, our patient has weakness of the following muscles: • • • •

Extensor pollicis; Extensor digitorum; Wrist extensors (extensor carpi); Supinator muscle.

Put another way, our patient has weakness of the extensors of the fingers, thumb, and wrist. Remember, the radial nerve supplies all of the extensors of the arm, so that would seem to be a logical nerve to start with. Checking Fig. 5.7 we see that the radial nerve also supplies the supinator, which is weak in our patient. In addition, the radial nerve’s sensory area corresponds very well to our patient’s complaints. The radial nerve is our initial localization. Let’s draw the path of the radial nerve to see where our lesion is located. Working from distal to proximal, we see we are at least at the level of the elbow, given the fact that the superficial branch is involved, as our patient has sensory complaints. Extensor carpi is weak so we know the lesion must be at least a little higher than the elbow. The last muscle innervated by the radial nerve is the triceps, which is of normal power. Our lesion must be below this. Thus, our final localization is somewhere about two-thirds of the way up the radial nerve, below the branch that innervates the triceps.

K25928_C007.indd 205

12/01/2017 12:18

206

Case 5: Solution

In this case the lesion is in a part of the humerus called the spiral groove, which is a frequent cause of external compression of the radial nerve. The key feature is that the triceps are normal (both power and reflex). This is in opposition to the more common compression at the level of the axilla, which would involve all of the muscles supplied by the radial nerve. Both compression at the spiral groove and at the axilla are examples of what is often referred to as a Saturday Night Palsy, so named because the patient usually consumes too much alcohol (or other drugs), and passes out in an atypical position. If this position is such that his arm drapes over a hard surface, such as a chair, then the radial nerve is typically compressed causing a transient loss of function.

K25928_C007.indd 206

12/01/2017 12:18

Case 5: Clinical Pearl

207

Figure 7.5 Carpal Tunnel Syndrome

Some individual figures adapted from Waxman SG. Clinical Neuroanatomy. Lange, New York, 2013.

K25928_C007.indd 207

12/01/2017 12:18

208

Case 5: Clinical Pearl

Clinical Pearl: Carpal Tunnel Syndrome The most frequent mononeuropathy seen in clinical practice is compression of the median nerve as it passes through the carpal tunnel in the wrist. The first sign of carpal tunnel syndrome (CTS) is intermittent numbness and tingling in the first three digits and the lateral aspect of the fourth digit. Patients may notice that this occurs most often when the hand is flexed at the wrist, such as after typing, or upon waking (most people have their wrists flexed during sleep). Eventually the numbness can progress to significant pain. About 65% of patients will have symptoms in both hands. If left untreated, patients will soon experience muscle weakness of the flexor pollicis, opponens pollicis, abductor pollicis and the second and third digits of the flexor digitorum. Note that because the lesion is at the wrist, the pronator teres is unaffected. As the median nerve is the main innervator of the thumb, patients with long-standing CTS will get significant atrophy of the thenar eminence. Long-term atrophy of median supplied muscles will result in the hand taking on a deformed shape called the Preacher’s Hand, or the Hand of Benediction, as it is similar to configuration of the hand of a priest when he blesses someone. In this case, when a patient tries to make a fist, he’ll be able to flex the 4th and 5th digits only; the second and third digits will be hyperextended due to unopposed action of the extensor digitorum. The thumb will be both extended and adducted due to the unopposed action of the adductor pollicis (ulnar nerve) and the extensor pollicis (radial nerve). First line treatment of CTS involves splinting of the wrist to keep it in a neutral position and avoid compressing the median nerve. Corticosteroid injections can also be performed. For truly refractory cases, surgical decompression can be performed.

K25928_C007.indd 208

12/01/2017 12:18

Case 6: The 34-year-old woman who had nothing wrong with her

209

Case 6: The 34-year-old woman who had nothing wrong with her You receive a phone call from the Emergency Department asking you to come down immediately and see a 34-year-old woman. You arrive and find her husband frantically pacing about the room. He grabs you quickly and tells you, “you have to help her, we have a 10-day-old baby at home.” Her husband states she was fine and talking to him when she, mid sentence, screamed in incredible pain, put her hand to the right side of her head, and then collapsed on the ground. When he went to pick her up, he noticed she had cut herself quite badly on the left side. However, when he went to grab a towel to cover the bleeding, she asked him what he was doing, and fervently denied anything was wrong. When you examine her she initially does not interact with you at all, despite you raising your voice loudly. Your benevolent attending suggests you try examining her from the other side of the bed. You walk to her right side and she greets you with a “hello doctor.” On formal language exam, she is somewhat slow to respond as she complains of significant pain, but you find her fluency, repetition and comprehension to be normal. She interrupts you several times to ask to go home because “nothing is wrong with me.” You see that her eyes are driven strongly to the right side. Her pupils are equal and reactive to light. She does not register anything on the left part of her visual field. You notice a facial droop on the left, but she is able to move her eyebrows. Her palate and tongue are midline. She has normal power on the right, but when you ask her to move the left side she fails to do so. However, you have noticed the left side moves spontaneously, albeit weakly, as the two of you are talking. Your attending interrupts and grabs her left hand and holds it infront of her face and asks “whose hand is this?”, to which she replied “yours.” She then looks puzzled and demands to know how your attending got a hold of her wedding ring. Her tone and reflexes are increased on the left side. She has an upgoing left toe. She denies feeling any sensory feeling on the left side, despite you inflicting significant painful stimulus. Coordination is not tested given her profound weakness on the left. Where is the lesion? What is going on here?

K25928_C007.indd 209

12/01/2017 12:18

210

Case 6: Findings

Case 6: Findings

K25928_C007.indd 210

12/01/2017 12:18

Case 6: Solution

211

Case 6: Solution 1. Does the patient have UMN weakness, LMN weakness or both? Our patient has increased tone, reflexes and an upgoing toe also on the left, indicative of an UMN type weakness; our lesion lies in the CNS. 2. Does the pattern of long tract symptoms suggest a localization? Our patient has weakness of the left face, arm and leg, so we know the right corticospinal tract is involved. She also denies any feeling on the left side, so the right ST is affected as well. This doesn’t give us much information to go on; potential localizations include the cortex, internal capsule and brainstem. 3. What is the highest level of dysfunction? Since our patient is able to lift her eyebrows, we know she has an UMN type facial droop, and our lesion must be above the level of the pons. Our patient has some very odd symptoms on her left side, which we will cover in a moment. Her eyes are driven strongly to the right. Since the FEF drive the eyes to the contralateral side, this implies that either the left side is hyperactive (commonly see in a seizure), or the right side is hypoactive, due to some sort of destructive lesion. Our patient’s presentation is not in keeping with a seizure; we can conclude that the right FEF are not working, and thus our initial localization is the cortex. 4. What is affected? Reviewing our patient’s symptoms we know that the right motor and sensory cortices are affected, as is our patient’s right FEF. The patient’s many left sided symptoms are all just manifestations of hemi-neglect, which is often seen in right sided cortical lesions. She continually states that nothing is wrong with her, despite her inability to move her left arm. In fact she was even unable to recognize her left arm when it was shown to her (a classic finding for hemi-neglect, as is the wedding ring confusion). It is often difficult to distinguish a left sided visual field cut from left sided neglect, but the fact that she did not register your voice and presence is more indicative of hemi-neglect. Her inability to voluntarily move her left arm with preserved spontaneous movement is indicative of an apraxia (which is an Immediate Localization to the cortex). Both hemi-neglect and apraxia have multiple potential localizations, but it is safe to say that at least parts of the frontal and parietal lobes are involved. Putting it all together it seems that our final localization is almost the entire right hemisphere.

K25928_C007.indd 211

12/01/2017 12:18

212

Case 6: Solutions

In this case, the patient had an instantaneous and severe headache, commonly called a thunderclap headache, which is usually indicative of a subarachnoid hemorrhage. Indeed, this patient had a right sided MCA aneurysm that ruptured causing the subarachnoid hemorrhage. For reasons that are not completely understood aneurysms have a higher rate of rupture in the immediate postpartum period than during pregnancy.

K25928_C007.indd 212

12/01/2017 12:18

Case 6: Clinical Pearl

213

Figure 7.6 Levels of Consciousness and the Glasgow Coma Scale

K25928_C007.indd 213

12/01/2017 12:18

214

Case 6: Clinical Pearl

Clinical Pearl: Levels of Consciousness and the Glasgow Coma Scale An aneurysmal rupture that does not receive prompt treatment can quickly cause a decreased level of consciousness (LOC). There is a lot of confusion around the terminology of the different levels of consciousness, so let’s review them now. The reticular activating system (RAS) is a circuit in the brain that is responsible for arousal and regulating the sleep wake cycle. Structural damage to this system will produce a decreased LOC. The chief components of the RAS include the hypothalamus, thalamus and the reticular formation in the midbrain. There are several distinct levels of consciousness that one should be familiar with. We’ll begin with the most depressed state, coma, and work our way up from there. A coma is a state in which the patient is completely unresponsive and unarousable, even to painful stimuli. Primitive brain reflexes may or may not still be present. A vegetative state is where the patient gains the “vegetative” function of sleep wake cycles and has periods where his or her eyes are open. However, any “responses” they show are purely reflexive and they have no awareness of their environment. It is a true case of “the lights are on but no one is home.” A patient who is stuporous is one that requires painful stimuli in order stay awake. The obtunded patient is more alert than this and only requires verbal stimuli to stay awake. Drowsiness is a subjective term that usually corresponds to a patient who is fully alert but responds slower than the examiner would like. Beyond this patients are fully alert. The above descriptors can be somewhat cumbersome in emergency situations when information needs to be communicated quickly; to remedy this the Glasgow Coma Scale was designed, as shown in Fig. 7.6. It is scored from 3 to 15; the minimum score is not 0. The maximum score is 4 for eye opening, 5 for verbal response and 6 for motor response. It can remembered using the following mneumonic: • We test movement of the EYES (4 letters = max score 4); • We test how VOCAL someone is (5 letters = max score 5); • We test how MOBILE someone is (6 letters = max score 6). The scoring systems for eye opening and verbal response are fairly self-explanatory. The motor scoring can be a source of confusion. A score of 6 means the patient obeys you perfectly. A score of 5 means that when you apply a painful stimulus, the patient swats at you. A score of 4 means that the patient will try to withdraw the limb where you are applying pain. A score of 1 indicates that there is no response by the patient to painful stimuli. The postures associated with a score of 3 or 2 are called decorticate and decerebrate, respectively. They are shown in Fig. 7.6 and sometimes also called flexor response, and extensor response, respectively, given the position of the arms. They are both indicative of severe brain damage and carry a guarded prognosis.

K25928_C007.indd 214

12/01/2017 12:18

Case 7: The 26-year-old woman who collapsed

215

Case 7: The 26-year-old woman who collapsed You and your senior resident have just finished seeing a patient in the Emergency Department and are looking forward to getting some much needed rest, when a paramedic stops you and asks you take a look at another patient they are just wheeling in. The paramedics were called by a bystander who saw a young woman collapse while waiting to cross the street. The paramedics are quite concerned because her right pupil is large and unreactive to light. Your senior resident lets you examine the patient as she watches. When you find the patient, she is stuporous and, by definition, only responds to pain. This certainly limits your neurological exam, but you can still glean a fair amount of information. In order to look at the right pupil, you have to lift her right eyelid as it is quite ptotic compared to the left side. Once you do, you confirm the paramedic’s finding of a large, dilated pupil. However, you do find that it reacts to light, just not as much as the left pupil. Since your senior resident is watching, you note the size of each pupil in dark and light conditions, which is recorded below. The right eye appears pulled to the right side and appears lower than the left one. It cannot adduct or move upward. She does not appear to have a facial droop. You open her mouth and see that her tongue and palate are midline. On motor exam, she has normal tone, her reflexes are 2+ throughout and she has downgoing toes. She cannot participate in formal power testing, but she moves all four limbs to painful stimulus, which tells you that if weakness is present, it is not severe. Sensory testing beyond the perception of pain is not possible, nor is coordination testing.

Pupil Examination Dark Conditions

Light Conditions

Right: 6mm Left: 4mm

Right: 5mm Left: 2mm

Where is the lesion? What is going on here?

K25928_C007.indd 215

12/01/2017 12:18

216

Case 7: Findings

Case 7: Findings

K25928_C007.indd 216

12/01/2017 12:18

Case 7: Solution

217

Case 7: Solution This case highlights one of the very few areas of neuroanatomy that should be committed to memory. Because of this, in addition to the fact that our patient has no long tract signs whatsoever, we make a break from our traditional localizing algorithm (though it would still work for this case; if you don’t believe me, give it a try). Our patient presents with an anisocoria (pupillary asymmetry). As we saw in Fig.  3.6, pupillary size is controlled by the balance of forces from the sympathetics, which act to dilate the pupil, and the parasympathetics, which act to constrict the pupil. The sympathetics are carried to the pupil by CN V1 and the parasympathetics are carried by CN III. A lesion to the parasympathetics will cause the constrictive force to the pupil to diminish; the sympathetic dilatory force will be largely unchecked resulting in a large pupil that barely constricts (if at all) to light. It seems that this matches the symptoms of our patient in her right eye! What about the other symptoms of the right eye? We see that our patient has severe ptosis of the right eyelid. The levator palpebrae superioris is the main muscle responsible for eye opening (the other is Muller’s Muscle which is innervated by the sympathetics, but it only plays a minor role) and is also innervated by CN III. Our patient’s right eye appears “down and out.” The resting position of the eye is determined by the net balance of the relative contributions of the muscles that surround it. If the eye is being pulled down and out, then the lateral rectus (CN VI) and the superior oblique (CN IV) muscles are unopposed. Checking Fig. 3.7 we see that all the weak muscles are innervated by CN III. It would appear that our lesion is somewhere along the path of CN III. The exact location of the lesion can’t be determined but given the patient’s presentation it is likely near the brainstem, as brainstem compression can cause changes in levels of consciousness. In this case the patient had a rupture of an aneurysm of the PCOM artery, which caused immediate compression of CN III and her subsequent presentation.

The triad of a mydriatic (dilated) pupil, ptosis of the eyelid and the eye appearing “down and out,” represents a CN III palsy, and should be committed to memory.

In this case, identifying the CN III palsy was quite obvious; however, as we’ll see shortly, there can be partial CN III palsies that are not accompanied by the eyelid ptosis and eye

K25928_C007.indd 217

12/01/2017 12:18

218

Case 7: Solution

position changes. The pupil may still react to light and only be subtly different than the unaffected pupil. How do you approach the case then? The key is to note the pupillary sizes in light and dark, and see under which conditions the anisocoria is greatest. Let’s use our example above. In the dark the anisocoria between the two pupils is 2  mm (6 mm − 4 mm). In the light, the anisocoria has grown to 3    mm (5  mm  − 2  mm). The anisocoria is greatest in the light; the light conditions test the integrity of the parasympathetic system. Since the difference is greatest here, we can conclude that there is a parasympathetic problem, and that the larger pupil (which should be constricting, but isn’t) is the dysfunctional one.

Stated another way: Anisocoria greater in light conditions than dark conditions represents a CN III palsy, and the larger pupil is the abnormal one.

K25928_C007.indd 218

12/01/2017 12:18

Case 7: Clinical Pearl

219

Figure 7.7 Medical vs. Surgical CN III Palsies

ACOM: anterior communicating artery; MCA: middle cerebral artery; PCOM: posterior communicating arteries. Adapted from Kline LB and Foroozan R. Neuro-Ophthalmology Review Manual. Slack Inc, Thorofare, NJ, 2013.

K25928_C007.indd 219

12/01/2017 12:18

220

Case 7: Clinical Pearl

Clinical Pearl: Medical vs. Surgical CN III Palsies As we have just seen, a complete CN III palsy consists of a dilated, unreactive or minimally reactive pupil, severe ptosis of the eyelid, and an eye that is “down and out”, due to the unopposed actions of the lateral rectus and superior oblique muscles. Such patients are said to have a “complete” CN III palsy, since the entire triad is present. However, a partial palsy is also possible. There are two types of partial palsies, as shown in Fig. 7.7; in order to understand them we need to review the anatomy of CN III. Somatotopically, the motor function (eye movement and eye opening) of CN III is located in the central part of the nerve. The parasympathetic fibers line the outside of the nerve. Patients with vascular risk factors, such as hypertension and poorly controlled diabetes, often infarct the central part of CN III. In doing so, the parasympathetics are spared, as they have a different vascular supply. These patients present with eyelid ptosis and an eye that is “down and out,” but the pupils are of equal size. Clinically, this is often called a medical CN III palsy, as the management is medical optimization of vascular risk factors in order to prevent future infarcts. This is in contrast to a surgical CN III palsy. In this case, an expanding mass (often feared to be a ruptured posterior communicating arteries (PCOM) aneurysm, as in our case) extrinsically compresses CN III. If patients present very soon after aneurysm rupture, only the parasympathetic component of CN III is affected; patients will have an enlarged pupil but full eye movements and no eyelid ptosis. As the pressure increases, they rest of the nerve will be compressed, leading to the full triad of a “complete” CN III palsy. Whenever a patient presents with an anisocoria, clinicians fear that the enlarged pupil represents a CN III palsy that has occurred due to a ruptured PCOM aneurysm. This is for good reason; it carries a mortality rate of 50% and of those that do survive, 30% are so neurologically impaired they require nursing home care. About 5% of the general population has an unruptured cerebral aneurysm; of these people, 20% will have multiple aneurysms. The most common type are saccular aneurysms, also called berry aneurysms. These aneurysms have a distinct neck, and typically occur at the branch points of the cerebral circulation. Anterior communicating artery (ACOM) and PCOM aneurysms are the most common type, at about 30% each. The next most common occur at the branching of the MCA in the Sylvian fissure and represent about 20%.

K25928_C007.indd 220

12/01/2017 12:18

Case 8: The 61-year-old woman with the large pupil

221

Case 8: The 61-year-old woman with the large pupil Several days ago you admitted a 61-year-old woman with a transient ischemic attack (TIA), which caused some temporary right arm weakness. A computed tomography (CT) angiogram showed 88% stenosis of the left internal carotid artery, so she underwent left carotid endarterectomy. She seemed fine post operatively. While you are rounding, you receive an urgent page from the medical student from the previous case... he states that the patient’s right pupil is enlarged. He is worried that her brain is herniating and wants to call neurosurgery and the ICU. When you arrive, you find a now very worried but completely alert patient. Language examination shows normal fluency, comprehension and repetition. Examining the eyes, you see that indeed, her left pupil is 2 mm in diameter, and her right pupil is 3  mm. Both react to light. When you shut off the light, her left pupil remains 2 mm, but her right becomes 4 mm. You question whether she has a mild ptosis of the left eyelid. She does not have a visual field defect, and she has full eye movements. She does not have a facial droop and her tongue and palate are midline. Her motor exam, including tone, reflexes and power, are normal. Sensory examination is normal to pinprick and vibration. Her coordination examination is normal. After the previous case, the medical student read all about “partial” CN III palsies and is quite happy he identified it early before the patient had a “complete” palsy. He picks up the phone to page neurosurgery but you stop him. You reassure both the medical student and, more importantly, the patient, that she is not dying, and that the problem will likely get better in a few days without any need for treatment. Where is the lesion? What is going on here?

K25928_C007.indd 221

12/01/2017 12:18

222

Case 8: Findings

Case 8: Findings

K25928_C007.indd 222

12/01/2017 12:19

Case 8: Solution

223

Case 8: Solution Since we don’t have any long tract signs, and this case seems eerily similar to the previous one, we’ll again depart from our traditional localization algorithm. Once again, the patient has an anisocoria; this means that there is a problem either with the sympathetic innervation (pupillary dilation) or parasympathetic innervation (pupillary constriction). There is an important subtlety to note before we begin our analysis; in this case we do not even know which pupil is abnormal. The medical student has assumed the worst and is fearful that the one on the right is abnormal because it is enlarged. In fact, the left side is more likely to be abnormal as there is a slight ptosis of the eyelid on that side. However, many people will have a slight eyelid ptosis if you look hard enough, so this is hardly reliable. We mentioned in the last case that the pupils should be examined both in light conditions and dark conditions. If the anisocoria is largest in light conditions, this represents a failure on behalf of the parasympathetic system to constrict. It turns out that the corollary is also true. If the anisocoria is largest in dark conditions, when the sympathetic system should most be active, then there is a lesion to the sympathetic system and the smaller pupil (which should be dilating, but isn’t) is the abnormal one. Let’s examine our case. In the light, when the parasympathetics are active, the anisocoria is 1 mm (3 mm − 2 mm). In the darkness when the sympathetics are active, the anisocoria has increased to 2 mm (4 mm − 2 mm). The smaller pupil, the left one, is the abnormal one. Stating it another way: Anisocoria greater in dark conditions than light conditions represents a sympathetic problem, and the smaller pupil is the abnormal one.

A lesion to the sympathetic system also explains the mild ptosis; the sympathetic system innervates Muller’s muscle, which is one of the muscles response for opening the eye.

Lesions of the sympathetic system result in a clinical triad called Horner’s syndrome: miosis (small pupil), ptosis of the eyelid, and anhidrosis (lack of sweating). The sympathetics do not decussate so this all occurs on the ipsilateral side of the lesion.

K25928_C007.indd 223

12/01/2017 12:19

224

Case 8: Solution

Ideally, one would like to localize the lesion further; the sympathetic pathway is quite vast, as shown in Fig. 3.6, and the causative lesion could lie in the brainstem, spinal cord, or periphery. However, this requires pharmacological testing, which we will turn our attention to next. In this case, the patient developed a Horner’s syndrome due to the local manipulation of the sympathetic fibers that lay on the carotid artery during the endarterectomy procedure, which is a known complication. It resolved several days later. The medical student never confused a CN III palsy with a Horner’s syndrome again.

K25928_C007.indd 224

12/01/2017 12:19

Case 8: Clinical Pearl

225

Figure 7.8 Horner’s Syndrome

NE: norepinephrine.

K25928_C007.indd 225

12/01/2017 12:19

226

Case 8: Clinical Pearl

Clinical Pearl: Horner’s Syndrome As we mentioned, the triad of eyelid ptosis, miosis (small pupil) and anhidrosis (lack of sweating) is referred to as Horner’s syndrome and is caused by interruption of the sympathetic system. As you can see from Fig. 7.8, the sympathetic system is divided into three neurons. The 1st order neuron begins in the hypothalamus and synapses in the gray matter of the spinal cord at the C8/T1 level. The 2nd order neuron travels from there into the sympathetic chain and synapses in the superior cervical ganglion. Finally, the 3rd order neuron travels from the superior cervical ganglion, up the internal carotid artery and into the end muscles. Certain chemical tests can be used to both confirm a Horner’s syndrome, and also help localize between 1st order, 2nd order and 3rd order lesions. In the era of MRI they have fallen somewhat out of favor but are still used by many clinicians. The mechanisms of these tests are complicated and beyond the scope of this text but the results can be remembered easily enough. The first step is to confirm that a Horner’s syndrome is present. To do this 10% cocaine eye drops are instilled into both eyes. One can think of cocaine as an irritant to the eye; any irritant will activate the sympathetic system. In the normal eye the sympathetic system activates and the pupil dilates. In the abnormal eye the sympathetic system is lesioned, so the pupil remains unchanged, confirming the Horner’s syndrome. Hydroxyamphetamine drops (1%) can help us to localize where the lesion is along the sympathetic pathway. Recall from Fig.  7.3 that the sympathetic system uses the preganglionic neurotransmitter Ach (1st and 2nd order neuron), and the postganglionic neurotransmitter NE (3rd order neuron). Hydroxyamphetamine acts as a replacement for Ach and can be used to stimulate the 3rd order neuron. Let’s consider two scenarios. In the first, there is a lesion to the 1st or 2nd order neuron. In this case the 3rd order neuron can still respond to Ach, but it is not being secreted by the 2nd order neuron. Here, Hydroxyamphetamine will stimulate the functioning 3rd order neuron to release NE and the pupil will dilate. What if there is a lesion of the 3rd order neuron? The 1st and 2nd order neurons appropriately secrete Ach, but the 3rd order neuron cannot respond to it. In this case Hydroxyamphetamine will have no effect and the pupil remains constricted. Since 1st and 2nd order neurons both employ Ach, there is no pharmacological way to differentiate between the two.

K25928_C007.indd 226

12/01/2017 12:19

Case 9: The 39-year-old woman who didn’t feel her sunburn

227

Case 9: The 39-year-old woman who didn’t feel her sunburn You’re quite puzzled by a 39-year-old woman whom you are seeing early one morning. She previously enjoyed perfect health other than being the victim of an assault 15 years prior. She states that about 8 months ago she began to feel numbness and tingling on the 4th and 5th digits of her hands bilaterally. Her mother told her it was carpal tunnel syndrome and she dismissed it. On a recent trip to the beach, she fell asleep and inadvertently received an overzealous tan all over her body. Upon waking she was startled to realize that she only felt the burn in her legs and abdomen; despite being sunburned over her chest, she did not feel any pain in that area. She grew concerned and drove to the Emergency Department, where she now sits across from you. When you examine her, you find normal fluency, comprehension and repetition of language. CN examination reveals equally sized pupils, full eye movements, full visual fields, no facial droop, normal facial sensation and a tongue and palate that are midline. Motor examination reveals normal reflexes and normal tone. Her power examination is totally normal in the legs, but her arms show mild distal weakness, as recorded below. Her toes are downgoing. Her vibration testing is normal, but her pinprick examination is confusing. When you run the pin down her trunk, she feels it normally, but then states it becomes dull just below shoulder level. It remains dull until just below the level of the breasts, where it becomes sharp again. The pin is also dull over the entire medial aspect of both arms as well as the 4th and 5th digits of the hand. Finger to nose and heel to shin coordination testing is normal.

Power Examination Right

Left

Deltoid

5

5

Bicep

5

5

Tricep

5

5

WE

5

5

WF

5

5

FE

5

5

FF

4

4

DI

4

4

EP

4

4

FP

4

4

OP

4

4

Where is the lesion? What is going on here?

K25928_C007.indd 227

12/01/2017 12:19

228

Case 9: Findings

Case 9: Findings

K25928_C007.indd 228

12/01/2017 12:19

Case 9: Solution

229

Case 9: Solution 1. Does the patient have UMN weakness, LMN weakness or both? Our patient has weakness in her hands and thumbs; unfortunately there are no other signs such as reflexes or tone to help guide us, so we cannot classify the pattern. 2. Does the pattern of long tract symptoms suggest a localization? Since the motor system does not help us, we must turn our attention to the sensory symptoms. Thankfully these efforts do not go unrewarded, as the pattern of sensory loss is one we examined in an Immediate Localization! Our patient complains of a sensory band between the shoulders and just below the breasts; this can only be produced by a spinal cord lesion (see Chapter 4). Thus our initial localization is the spinal cord. 3. What is the highest level of dysfunction? The highest sensory symptom our patient experiences is the loss of pinprick just below shoulder level. She also complains of loss of pinprick down the medial aspect of the arm. Consulting our dermatome map (Fig. 5.15) we see that these areas correspond roughly to the C8/T1 area. This is also consistent with the weakness she has in her hands and thumbs, as these muscles are innervated by C8 and T1. We can refine our localization to the C8/T1 level of the spinal cord. 4. What is affected? Knowing that the lesion is at C8/T1, we can draw the cross section of the cord at that level. Our patient has bilateral loss of pinprick, which implies the ST is involved. The only lesion that can produce this lies at the very heart of the gray matter in the spinal cord. As we reviewed in Fig. 4.4, sensory neurons enter the cord and decussate to the contralateral ST. A lesion in the central grey matter would block this decussation and produce symptoms at the dermatome. As the lesion grows, it extends longitudinally to involve more and more dermatomes until a “band” of sensory loss occurs, as in this case. Eventually it begins to grow axially and encompasses the AHC, causing bilateral LMN type weakness. In this case that was the C8/ T1 AHC, which caused the hand weakness. The pattern described here is known as central cord syndrome. It is most commonly caused by a syringomyelia, which is a fluid filled cavity that forms many years after an initial trauma; its pathogenesis is far from understood.

K25928_C007.indd 229

12/01/2017 12:19

230

Case 9: Clinical Pearl

Figure 7.9 Central Cord Syndrome

DC: dorsal column; LMN: lower motor neuron; ST: spinothalamic tract; UMN: upper motor neuron.

K25928_C007.indd 230

12/01/2017 12:19

Case 9: Clinical Pearl

231

Clinical Pearl: Central Cord Syndrome Central cord syndrome is one of the most common types of spinal cord lesions seen in clinical practice. As mentioned, it is usually due to the formation of a syringomyelia which occurs in response to a long preceding trauma. The cervical spine is particularly prone to syringomyelia formation, usually from an injury involving hyperextension of the neck, such as the “whiplash” injury that occurs in motor vehicle accidents. Reviewing the symptoms produced by an expanding central cord lesion is a great way to cement our knowledge of spinal cord anatomy, so we will do that now. Stage 1: Small lesion Let’s consider a small lesion occurring at C5. The lesion is initially so small that the only thing it does is interrupt the decussation of sensory neurons on the way to the contralateral ST. This results in a bilateral loss of pinprick and temperature in the C5 dermatome distribution, as we have outlined in the figure. Stage 2: Longitudinally growing lesion Central cord lesions tend to expand longitudinally before growing axially. Some time has elapsed and our lesion now extends down to T4. It still does not involve any of the long tracts but it now interrupts the decussation of sensory neurons from multiple levels (in this case C5–T4). There is now a band of sensory loss (sometimes called a suspended sensory loss) to pinprick and temperature. Another way to think of it is that the patient has two sensory levels, one at C5 and the other at T4, and there is a zone of loss in between. The sensory loss depicted in Fig.  7.9 (Stage 2), involving the trunk and the arms, is sometimes called a “cape” or “shawl” loss, as it traces the outline of someone wearing a small cape or shawl. Stage 3: Transversely growing lesion Now our lesion has been allowed to grow transversely. All aspects of the DC are affected, causing proprioceptive and vibration loss below the level of the lesion. The AHC are enveloped bilaterally, causing a LMN type weakness in the C5 muscles. The lesion envelops most of the ST and corticospinal tracts on both sides. This causes a near complete loss of these functions below the level of the lesion. However, re-familiarize yourself with Fig. 4.5 and look at what is NOT involved; the sacral elements of the corticospinal and STs. They lie the most lateral and so will be the last to be affected. This is referred to as “sacral sparing” as the patient now has, what appears on first glance, to be a complete loss of neurological function from the neck down but retains sensory and motor function of the bowel and bladder.

K25928_C007.indd 231

12/01/2017 12:19

Case 10: The 49-year-old man who was told he had carpal tunnel syndrome

233

Case 10: The 49-year-old man who was told he had carpal tunnel syndrome You are asked to see a 49-year-old man with a bizarre sounding history. He tells you that about 2 years ago he started to notice difficulty typing with his left hand. He started to drop objects from it and was told he needed carpal tunnel surgery. He proceeded with surgery but found that it didn’t help. He then found that the same thing began to happen in his right hand. On the advice of a friend, he sought out a chiropractor who told him he had a disc pressing on his cervical spine, but many rounds of chiropractic manipulation failed to help. Around this time his wife began to notice that parts of his arms would “twitch” excessively for a few moments, and then go back to normal. When you examine him you find his language to be entirely normal. On CN examination you find he has full eye movements, full visual fields, and pupils that are equal and reactive to light. His facial sensation is normal and he does not have a facial droop. His palate and tongue are midline, though his tongue seems to be unable to stay still. You move onto his arms and find extensive atrophy, especially in his hands, worse on the left than the right. You agree with his wife that his muscles seem to “twitch” in the upper part of both arms. You examine his legs and find the same twitching is his right thigh and calf, but none on his left leg. You wonder about some mild atrophy of his right quadriceps. His power examination is shown below but he is quite weak other than the left leg, which is normal. You expect to find decreased tone in his arms, but his tone is actually normal. Furthermore he actually has increased reflexes in both arms. His tone is decreased in the right leg but he has 2+ knee and ankle reflexes. His left leg has normal tone and 2+ reflexes. He also has downgoing toes bilaterally. His sensory examination to pinprick and vibration is completely normal. Coordination testing, given his weakness, is normal. Power Examination Right

Left

Deltoid

4

4

Bicep

4

4-

Tricep

4

4-

WE

3

3

FE

4-

3

HF

4-

5

KE

4

5

KF

4

5

ADF

4-

5

APF

4

5

Where is the lesion? What is going on here?

K25928_C007.indd 233

12/01/2017 12:19

234

Case 10: Findings

Case 10: Findings

UMN: upper motor neuron; LMN: lower motor neuron.

K25928_C007.indd 234

12/01/2017 12:19

Case 10: Solution

235

Case 10: Solution 1. Does the patient have UMN weakness, LMN weakness or both? This is a confusing case. The argument for both an UMN pattern and a LMN pattern can be made. Our patient has atrophy of both arms, and the right leg, and has fasciculations of the right side, all in keeping with a LMN pattern. However, he also has increased reflexes in the arms and normal tone (preserved, or normal, tone or reflexes in the context of atrophied muscle groups are considered to be signs of an UMN problem), suggesting an UMN pattern. 2. Does the pattern of long tract symptoms suggest a localization? Our patient doesn’t have any sensory abnormalities to help us in our localization. When you see a combined pattern of UMN and LMN weakness, only two possibilities exist: • a focal lesion is affecting the brainstem or spinal cord in such a way that it produces LMN weakness at the level of the lesion, due to interruption of the nerve roots, and UMN weakness below the level of the lesion, due to interruption of the corticospinal tract. • a diffuse disease process is targeting one aspect of the nervous system; in this case, motor neurons in the AHC, causing the LMN findings and motor neurons in the cortex, causing the UMN findings. 3. What is the highest level of dysfunction? Before we say that there is a diffuse disease process affecting motor neurons, we must make sure that a single focal lesion isn’t responsible for our patient’s symptoms. Since our patient’s CN are normal, the only possible focal lesion would be in the cervical spine; this could certainly cause both arm and leg symptoms. However, it is important to recognize why this is NOT the case in this example. A lesion to the cervical spine would produce LMN symptoms at the level of compression, and UMN symptoms below the lesion. So we would expect our patient to have some LMN symptoms in the arm (fasciculation of the upper arm and atrophy of the shoulders) corresponding to the root involvement, and UMN symptoms below that (the increased reflexes and preserved tone). So far so good. But in the right leg the patient has fasciculations and atrophy, with downgoing toes. These are signs of LMN disease. It is impossible for a lesion of the cervical spine to do this; a cervical spine lesion can only cause UMN weakness of the legs (Fig. 4.10). In this case a focal lesion is impossible. We are forced to conclude that this patient has a disease process targeting the motor neurons. In this case that disease is amyotrophic lateral sclerosis (ALS), which we now discuss.

K25928_C007.indd 235

12/01/2017 12:19

236

Case 10: Clinical Pearl

Figure 7.10 Amyotrophic Lateral Sclerosis

K25928_C007.indd 236

12/01/2017 12:19

Case 10: Clinical Pearl

237

Clinical Pearl: Amyotrophic Lateral Sclerosis ALS, also known as Lou Gehrig’s disease after a famous case, is a devastating neurological disorder that has little effective treatment despite tremendous investigation and study. ALS exclusively targets motor neurons, so patients should only complain of weakness. Any sort of sensory, coordination or autonomic findings on examination virtually excludes the disorder. ALS targets both the UMNs of the motor cortex and the LMNs of the CN nuclei in the brainstem and the AHC in the spinal cord. The El Escorial World Federation of Neurology criteria, drafted in 1994, are often used to diagnose ALS. The criteria divide the body into four distinct regions, as shown in Fig. 7.10. Each region corresponds to a different part of the neuraxis. It includes the bulbar region, which corresponds to the brainstem, cervical spine, thoracic spine and lumbosacral spine. A definite diagnosis of ALS requires the combination of UMN and LMN findings present in each of at least three regions. For example, the patient may have increased tone and fasciculations in the leg (lumbosacral region), increased reflexes and atrophy of the arm (cervical region), and an increased jaw jerk reflex and fasciculations of the tongue (bulbar region). Under certain circumstances the patient can be diagnosed when they only have UMN findings on clinical examination if they have additional evidence from certain electrophysiological studies such as electromyogram (EMG). On average, patients live about 3 years after being diagnosed. The mechanism of death is usually aspiration pneumonia from bulbar weakness, or primary respiratory muscle failure. Two interventions have been shown to prolong life; external ventilation devices, such as bilevel positive airway pressure (BiPAP) machines, and the oral medication Riluzole, a glutamate antagonist, which has been shown to increase survival by about 3 months.

K25928_C007.indd 237

12/01/2017 12:19

Case 11: The 68-year-old man with previous TIAs

239

Case 11: The 68-year-old man with previous TIAs You are seeing a 68-year-old man with symptoms concerning for stroke. As the patient is being offloaded from the ambulance, you talk to his daughter who says that her father was seen for TIAs or “mini strokes” about 3  months ago, and was started on aspirin and other medications. She states her father experienced two episodes of right sided weakness with “garbled speech” and agitation. His daughter remembers to tell you, “he had his neck arteries scanned.” When you arrive you note that not only is he speaking nonsense but his speech is also dysarthric. He says very little, and appears to have great difficultly saying any particular word, which frustrates him greatly. When he does manage to get a word out, it appears to be random. He cannot repeat a sentence. He does appear to understand what you’re saying, which you find of great relief as you proceed with the rest of your neurological exam. He has full visual fields and pupils that are equal and reactive to light. However, his eyes are driven to the left and he cannot look to the right. You note a right sided facial droop that spares the forehead. His tongue and palate are midline. Motor examination shows increased tone on his right side, and he even has five beats of clonus at the right ankle. Reflexes are 3+ in the right arm and leg. He has an upgoing toe on the right. Power testing is shown below but he is weak on the right side. He also has impaired sense to pinprick examination in the right arm and leg. Vibration could not reliably be assessed. Coordination testing could not be carried out given his level of weakness.

Power Examination Right

Left

Deltoid

3

5

Bicep

4

5

Tricep

4-

5

WE

4-

5

FE

4-

5

HF

4+

5

KE

4+

5

KF

4

5

ADF

4+

5

APF

4

5

Where is the lesion? What is going on here?

K25928_C007.indd 239

12/01/2017 12:19

240

Case 11: Findings

Case 11: Findings

B: Broca’s area; C: conception tract; MCA: middle cerebral artery; W: Wernicke’s area.

K25928_C007.indd 240

12/01/2017 12:19

Case 11: Solution

241

Case 11: Solution 1. Does the patient have UMN weakness, LMN weakness or both? Our patient has all the stigmata of an UMN process; increased tone, increased reflexes, an upgoing toe, and a pathological amount of clonus at the right ankle. We can safely say that our lesion lies in the CNS. 2. Does the pattern of long tract symptoms suggest a localization? Our patient has weakness and decreased sensation involving his right face, arm and leg. This isn’t a lot of information to go on; possible localizations include the cortex, internal capsule and brainstem. 3. What is the highest level of dysfunction? Reviewing our symptoms, we see that our patient has an UMN facial droop, so we know the lesion must lie above the level of the pons. However, his language examination reveals an Immediate Localization; in addition to his dysarthria he is also aphasic. He has trouble getting any words out (decreased fluency), and cannot repeat. However, understanding is intact. This is a problem with the generation of language and corresponds to a Broca’s aphasia. Aphasia can only be produced by a cortical lesion, which is our initial localization. 4. What is affected? Reviewing our patient’s symptoms, we see that he has weakness and sensory loss on the right. The long tracts have decussated by the time they reach the cortex, so we know that our patient’s right sided symptoms are produced by a lesion of the left cortex. Broca’s aphasia is caused by a lesion in Broca’s area, which sits in the inferior part of the frontal lobe. Finally, our patient’s eyes are driven to the left. Remember that the FEF drive the eyes to the contralateral side. Since his eyes are driven to the left, it means that nothing is opposing the FEF on the right; the left FEF must be affected by the lesion. Putting it all together, we see that almost the entire lateral aspect of the left frontal lobe is involved. This territory is supplied by the superior branch of the MCA. Imaging later revealed that this unfortunate fellow suffered a stroke in this location.

K25928_C007.indd 241

12/01/2017 12:19

242

Case 11: Clinical Pearl

Figure 7.11 Carotid Stenosis

Barnett HJ, et al. Beneficial effect of carotid endarterectomy in symptomatic patients with high grade carotid stenosis. NEJM 1991;325:445– 453. Brott TG, et al. Stenting vs endarterectomy for treatment of carotid artery stenosis. NEJM 2010;363:11– 23.

K25928_C007.indd 242

12/01/2017 12:19

Case 11: Clinical Pearl

243

Clinical Pearl: Carotid Stenosis Plaque in the carotid arteries, as in this case, can often serve as a nidus for embolism in the anterior circulation (anterior cerebral artery (ACA) and MCA territory), and is a common cause of stroke or transient ischemic attack (TIA). A TIA, as the name implies, occurs when an area of brain is deprived of blood, which causes neurological symptoms. However, this blood supply is soon restored, leading to a resolution of symptoms and no permanent neurological damage. All patients who present with an anterior circulation TIA or stroke should undergo vessel imaging to see if there is significant plaque causing a stenosis of the carotid artery. This naturally leads to the question of how much stenosis of the carotid artery is significant enough to warrant treatment? Further, should the treatment be aggressive medical therapy or more surgical in nature? These questions were answered in the landmark North American Symptomatic Carotid Endarterectomy Trial (NASCET trial) in 1991. It demonstrated that for patients with a stenosis occluding more than 70% of the diameter of the internal carotid arteries (ICA), surgery was more effective then medical management; at 2 years, the rate of re-stroke was 26% in the medical group and 9% in the surgical group. Still, this result has been questioned in recent years, as medical treatment has advanced significantly over the last 25 years. Note that the above was for symptomatic stenosis. It is only valid for patients who have had an anterior circulation TIA or stroke believed to be caused by the carotid stenosis. It cannot be generalized to asymptomatic patients. Patients in the surgical arm underwent a carotid endarterectomy. This procedure was first performed in 1954 and is shown in the bottom left of Fig. 7.11. The carotid artery is clamped, and the brain is perfused via the collaterals in the Circle of Willis. The ICA is then cut, and the plaque removed. Carotid stenting is a newer technique, which is less invasive. In this procedure, a stent is fed up into the carotid artery and then deployed over the plaque. Eventually, the body will cover the stent with a new layer of tissue, effectively sealing up the plaque. The Carotid Revascularization Endarterectomy Stenting Trial (CREST trial) compared endarterectomy to stenting. It demonstrated that the two procedures are equally effective. However, younger patients (< 70 years old) experienced fewer complications with stenting and older patients (> 70 years old) had fewer complications with endarterectomy. A trial comparing carotid stenting to medical management has yet to be performed.

K25928_C007.indd 243

12/01/2017 12:19

Case 12: The 55-year-old man with seizures

245

Case 12: The 55-year-old man with seizures You are spending an afternoon catching up on some paperwork in the staff lounge when you’re paged by an Emergency Doctor who says she needs you to come see a patient right away. When you arrive, she tells you that a 55-year-old man came into the Emergency Department with a 4-hour history of right sided weakness. She diagnosed him with a left MCA stroke and was going to call you to come see him later that day, but she’s now concerned that he’s having seizures and wants you to assess him immediately. You find a quite obtunded patient whose responses to your questions are limited to a short few words. Despite this, you feel that, while confused, he does not have any sort of obvious language problem, and he does participate in your neurological exam. He sounds dysarthric and his wife confirms that this is not his normal speech. He has full visual fields and his pupils are equal and reactive to light. However, his eyes appear driven to the right, and he cannot get them to cross midline in order to look left. Upon close inspection you also notice a subtle droop of the entire face on the left hand side. You ask the patient to raise his eyebrows, but he is unable to do so on the left side. The tongue and palate are midline. On motor exam, you find increased tone in the right arm and leg. He also has 3+ reflexes on the right. Formal power examination is shown below, but he is weak on the right side. He has an upgoing toe on the right. Sensation is normal to pinprick. Given his obtunded state, vibration testing isn’t possible. Coordination was felt to be normal given the weakness.

Power Examination Right

Left

Deltoid

4+

5

Bicep

4+

5

Tricep

4

5

WE

4-

5

FE

4-

5

HF

4+

5

KE

4+

5

KF

4

5

ADF

4

5

APF

4

5

Where is the lesion? What is going on here? Why did the Emergency Doctor think the patient was having seizures?

K25928_C007.indd 245

12/01/2017 12:19

246

Case 12: Findings

Case 12: Findings

K25928_C007.indd 246

12/01/2017 12:19

Case 12: Solution

247

Case 12: Solution 1. Does the patient have UMN weakness, LMN weakness or both? Our patient has increased reflexes, increased tone and an upgoing toe, all signs of an UMN type weakness. Thus, we know that the lesion lies in the CNS. 2. Does the pattern of long tract symptoms suggest a localization? In this case, our patient has right sided weakness of the arm and leg, and left sided weakness of the face; this is an example of crossed signs, and is an Immediate Localization to the brainstem (see Chapter 3). 3. What is the highest level of dysfunction? Our patient has a facial droop affecting all of his face on the left side, including the forehead. This is an example of a LMN type facial droop. A LMN facial droop can be caused by a lesion to either the facial nerve itself (PNS) or to the facial nucleus and its neurons, which are located in the pons (CNS). Since we know our lesion is in the CNS, we can safely say it is the latter; our initial localization is the pons. 4. What is affected? Since we know the location of our lesion, we can draw the pons in cross section. Let’s fill in what we know. Our patient has UMN weakness of the right arm and leg, so the left corticospinal tract is involved. In addition, he has left sided LMN facial weakness, so the left facial nucleus must be affected. Note how the facial neuron wraps around the abducens nucleus of CN VI. It would be quite difficult to have a lesion to the facial nucleus and NOT involve CN VI. Let’s return to our patient; he cannot initiate gaze to the left with either eye. Remember that the abducens initiates horizontal gaze for both eyes (Fig. 3.9). So our patient also must have left abducens nucleus involvement. For this reason, a patient with a LMN type facial droop (CN VII) should always be carefully assessed to see if they have an ipsilateral CN VI involvement (sometimes referred to colloquially as “make sure seven doesn’t have six”). This syndrome is called Millard Gubler syndrome, after the physicians that described it. It can be associated with strokes of the basilar artery, which is likely what happened here. We will now turn our attention to why the Emergency Doctor thought the patient was seizing.

K25928_C007.indd 247

12/01/2017 12:19

248

Case 12: Clinical Pearl

Figure 7.12 Wrong Way Eyes

FEF: frontal eye fields; MLF: medial longitudinal fasciculus; PPRF: paramedian pontine reticular formation. Top portion redrawn with permission from Greenberg D, et al. Clinical Neurology. McGraw–Hill Education, New York, 2015.

K25928_C007.indd 248

12/01/2017 12:19

Case 12: Clinical Pearl

249

Clinical Pearl: Wrong Way Eyes Voluntary eye movement is initiated by the FEF in the cortex. The FEF synapse with the contralateral abducens nucleus, which is responsible for initiating horizontal gaze. In doing so, the left FEF drive the eyes to the right and vice versa. Lesions of the cortex, such as in the common MCA stroke, damage the FEF. Let’s consider what would happen if the left FEF was damaged from an MCA stroke (Lesion 1 in Fig. 7.12). The right FEF would be completely unopposed and since the FEF drive the eyes to the contralateral side, the eyes would drift to the left. The left MCA stroke would also damage the left motor cortex, causing right sided weakness of the face, arm and legs. Thus, the eyes are deviated to the opposite side of the weakness; clinicians sometimes remember this by the inelegant saying “the eyes look away from the paralyzed side in disgust.” However, the patient in our case had a lesion of the pons (Lesion 2 in Fig. 7.12). Review his symptoms; his eyes were deviated toward his weak arms and legs, or the “wrong way” compared to the much, much more common cortical lesion caused by the MCA stroke. This is because the abducens nucleus initiates gaze to the ipsilateral side for both eyes; the right CN VI nucleus initiates gaze to the right side for both eyes, via the MLF. Thus, a lesion to the right CN VI nucleus would result in eyes deviated to the left side. A right lesion in the pons affecting the corticospinal tract would also result in left sided weakness as the corticospinal tract decussates in the medulla. Thus, the patient looks toward the weak side, or the “wrong way.” A seizure hyperactivates an area of the cortex (Lesion 3 in Fig. 7.12). What if our patient was having a focal seizure that started in the left hemisphere? Then his left FEF would be hyperactive and overcome the normal right FEF, and his eyes would be driven to the right. Once the seizure spreads to the left motor cortex, it would cause right sided face, arm and leg seizure activity. If this activity is prolonged, the muscles will continue to seize until they’re totally exhausted and can no longer contract at which point they’d appear weak. However, the eyes would still be deviated to the right. Thus the eyes would look toward the weak side, or the “wrong way.” It was because of these “wrong way” eyes, that the Emergency Doctor erroneously concluded that the patient was having a seizure post stroke. In the absence of a clinical seizure, “wrong way” eyes is another way to immediately localize a lesion to the pons.

K25928_C007.indd 249

12/01/2017 12:19

Case 13: The 61-year-old man with the encouraging wife

251

Case 13: The 61-year-old man with the encouraging wife You are seeing a 61-year-old man who reluctantly came to the Emergency Department for right sided weakness. When you talk to him, he, with some gentle “encouragement” from his wife, tells you he fell this morning getting out of bed because his right leg wouldn’t work. He then went to grab his glasses but he found his right hand to be clumsy and he dropped them. His wife thinks his speech is slurred but he disagrees. When examining his chart, you notice multiple entries by physicians about poor compliance to medications for his diabetes and high blood pressure. His language examination shows normal fluency, comprehension and repetition. You agree with his wife that he is dysarthric. CN examination reveals full visual fields, pupils that are equal and reactive to light and he has full eye movements. He has a facial droop on the right but he is able to move both eyebrows. His tongue and palate are midline. Motor examination shows increased tone and 3+ reflexes on the right, with upgoing toe on that side. His formal power examination is shown below but he is weak on the right side. Sensory examination was normal to pinprick and vibration. Coordination testing was felt to be normal, given the weakness. As you walk out of the room, you glance at the monitor and note his blood pressure is 189/79.

Power Examination Right

Left

Deltoid

4+

5

Bicep

4

5

Tricep

4-

5

WE

4-

5

FE

4-

5

HF

4

5

KE

4

5

KF

4-

5

ADF

4-

5

APF

4-

5

Where is the lesion? What is going on here?

K25928_C007.indd 251

12/01/2017 12:19

252

Case 13: Findings

Case 13: Findings

K25928_C007.indd 252

12/01/2017 12:19

Case 13: Solution

253

Case 13: Solution 1. Does the patient have UMN weakness, LMN weakness or both? Our patient has increased tone and reflexes as well as an upgoing toe, which are all signs of an UMN type weakness. Thus, we know that the lesion lies in the CNS. 2. Does the pattern of long tract symptoms suggest a localization? Our patient’s sensory examination is normal, but he has weakness of the right face, arm and leg. This pattern is nonspecific and could be caused by a lesion in the cortex, internal capsule or brainstem. 3. What is the highest level of dysfunction? A facial droop that spares the forehead represents an UMN CN VII lesion. As we’ve seen previously, this means that the lesion must be above the level of the facial nucleus in the pons. Unfortunately this is the only refinement we can make to our localization. 4. What is affected? In this case, there are not any further symptoms to guide our localization. However, the paucity of symptoms itself can suggest a localization. In order to cause such extensive right sided weakness, a cortical lesion would have to be very large, such as in an MCA stroke. However, our patient does not have any of the other symptoms we would expect with such a large cortical lesion, like visual field defects, aphasia, apraxia or neglect. This makes a cortical localization unlikely. Small lesions in very neuroanatomically dense areas can present with significant symptoms. In this case the patient had a stroke of the posterior limb of the internal capsule. As we saw in Fig. 2.4, motor neurons to the face, arm and leg pass through this very small area; hence, small lesions here can have devastating large effects. These types of infarcts are called lacunar strokes and are quite common.

K25928_C007.indd 253

12/01/2017 12:19

254

Case 13: Clinical Pearl

Figure 7.13 Lacunar Strokes

AICA: anterior inferior cerebellar artery; PCA: posterior cerebral arteries; PICA: posterior inferior cerebellar artery; SCA: superior cerebellar artery.

K25928_C007.indd 254

12/01/2017 12:19

Case 13: Clinical Pearl

255

Clinical Pearl: Lacunar Strokes Lacunar strokes (Lacunar is Latin for “tiny lake”) are small infarcts that occur in neuroanatomically dense areas of the brain. Despite their small size, because of their strategic location, they produce tremendous symptoms. The arteries involved are usually the tiny perforating arteries of the MCA, collectively called the lenticulostriate arteries or the perforating arteries of the basilar, called the basilar perforators. Because the culprit is these tiny arteries, lacunar strokes are also referred to as small vessel disease, in contrast to strokes from the much larger carotid and vertebral artery, which are known as large vessel disease. Lacunar strokes are often thought to be due to a pathologic processes called lipohyalinosis. This typically happens in patients with poorly controlled blood pressure and diabetes. As a person’s blood pressure increases, the friable walls of small vessels must thicken, or hypertrophy, in order to withstand the pressure. Uncontrolled diabetes accelerates this process. Eventually one of two processes occur. In the first, the continued high blood pressure overcomes the ability of the vessels walls to hypertrophy and a small hemorrhage occurs. In the second, the arterial walls hypertrophy so much that effective blood flow to the brain is cut off, and the brain undergoes infarction. Lacunar stroke patients often have a “honeymoon” period for the first 48 hours, where their symptoms can vary significantly in severity, before the dysfunction becomes permanent. The pathophysiology behind this is not understood.

K25928_C007.indd 255

12/01/2017 12:19

Case 14: The 39-year-old woman who couldn’t see out her left eye

257

Case 14: The 39-year-old woman who couldn’t see out her left eye You are called into the Emergency Department in the early hours of the morning to see a 39-year-old woman. Her friends tell you she was out with them celebrating; 2 weeks prior she had been in hospital for the removal of a breast tumor which turned out to be benign. While partying, the patient’s friends were startled to see that her right pupil appeared much larger than the left. They tried to convince her to go to the hospital, but it was no use. Soon thereafter, she began to complain of seeing double, but chalked it up to the copious amount of alcohol she was enjoying. Only when she lost vision in her left eye did she finally agree to come to the hospital. When you see her, she is agitated and yelling that, “the purple children in the corner need to stop running around my bed!” You check her toxicology screen, which is negative for drugs and try to reassure her, but you are only partially successful. Still, from what you can piece together you feel that her language examination shows normal fluency, repetition and comprehension. You agree with her inebriated friends; her right pupil is quite large, and appears not to react to light. The eye also appears down and to the right side; it does not adduct or move upward. She has a moderate ptosis of the right eyelid. She has lost her left visual field in both eyes. She also has a left sided facial droop which does not involve the forehead. Her tongue and palate are midline. Motor examination shows normal tone, but 3+ reflexes on the left with an upgoing toe on the same side. You find a mild weakness in her left arm and leg, as shown below. Vibration testing and pinprick, though not the most reliable, appear normal. You move onto coordination testing but as you do she slumps over, and will not rouse to sternal rub. You call for an emergency intubation and watch as she is wheeled to the ICU.

Power Examination

Where is the lesion? What is going on here?

K25928_C007.indd 257

Right

Left

Deltoid

5

4+

Bicep

5

4+

Tricep

5

4

WE

5

4

FE

5

4

HF

5

4+

KE

5

4+

KF

5

4

ADF

5

4+

APF

5

4

12/01/2017 12:19

258

Case 14: Findings

Case 14: Findings

K25928_C007.indd 258

12/01/2017 12:19

Case 14: Solution

259

Case 14: Solution 1. Does the patient have UMN weakness, LMN weakness or both? Our patient has increased reflexes and an upgoing toe, both signs of an UMN type weakness. Thus, we know that the lesion lies in the CNS. 2. Does the pattern of long tract symptoms suggest a localization? Our patient does not have any sensory symptoms, so we can conclude the DC and ST are not involved. She has UMN weakness of the left arms and legs, and as the forehead is spared, a left UMN facial droop. As we’ve seen before, this potentially localizes to the cortex, internal capsule and brainstem. 3. What is the highest level of dysfunction? Our patient has lost her left visual field in both eyes; she has a left homonymous hemianopsia (LHH). Note that she erroneously, as most patients do, misinterpreted this as visual loss from her left eye. A homonymous hemianopsia can only be produced by a lesion to the cortex, which is our initial localization. 4. What is affected? Let’s return to our patient’s LHH. Since the optic neurons decussate at the chiasm, we know that a left visual field defect is caused by a right cortical lesion. Further, because it is a homonymous hemianopsia we know that the lesion must be posterior to the chiasm (Fig. 3.5). What about our patients other visual symptoms? Her right eye appears “down and out” and she has ptosis of the right eyelid. Already you should be thinking of a right CN III palsy. As we saw earlier, a CN III palsy causes dysfunction of parasympathetic innervation to the pupil. If the parasympathetic system is lesioned, then the pupil receives unopposed dilatation from the functioning sympathetic system, which is the case here. Thus, her right CN III or its nucleus is involved. CN III lies in the midbrain. Reviewing our symptoms, we see she has UMN weakness in her face, arm and leg on the left side. The corticospinal and corticobulbar tracts at the level of the midbrain are likely involved. But what about our patient’s hallucinations and personality change? How can she have a midbrain lesion AND a cortical lesion? This case is an example of the top of the basilar syndrome. Here, an embolus (in this case it was from a dissected vertebral artery, likely due to intubation process from her breast surgery), traveled to the proximal or “top” of the basilar, and lodged itself there. It then acts as a further source of emboli causing infarctions of areas supplied by the posterior cerebral arteries (PCA) and superior

K25928_C007.indd 259

12/01/2017 12:19

260

Case 14: Solution

cerebellar artery (SCA). Let’s review the timeline of our patient’s symptoms. She began with pupil dysfunction and double vision; the first embolus did not travel far and infarcted the right midbrain causing the CN III lesion and UMN weakness. She then developed the LHH; the second embolus traveled a little farther into the PCA and caused a right sided occipital lobe stroke. Then she developed agitation and hallucinations; the third embolus infarcted her right thalamus, causing personality change and hallucinations. She then became comatose; another embolus infarcted her other thalamus. Bilateral thalamic infarctions result in a depressed level of consciousness, as we saw in Chapter 3. Whenever you see anyone who develops new neurological dysfunction in a stepwise fashion, as above, you should be concerned about top of the basilar syndrome. This is a true medical emergency as it can have disastrous outcomes, as we saw in our unfortunate case above.

K25928_C007.indd 260

12/01/2017 12:19

Case 14: Clinical Pearl

261

Figure 7.14 Brain Herniation Syndromes

K25928_C007.indd 261

12/01/2017 12:19

262

Case 14: Clinical Pearl

Clinical Pearl: Brain Herniation Syndromes When faced with the case above many clinicians would have been concerned that some sort of mass was causing a fatal herniation of brain tissue. Herniation occurs when tissue from one anatomical compartment inappropriately expands into another anatomical compartment. This can occur in the brain, often due to some sort of expanding mass (such as a hemorrhage, tumor or abscess), which increases the intracranial pressure (ICP). There are several types of brain herniation: 1) Subfalcine: The medial cortex herniates underneath the falx cerebri, sometimes tearing itself in the process and causing leg weakness. Most cases are asymptomatic but warn of impending herniation at other sites. This is the most common type of herniation. 2) Central: The temporal lobes and basal ganglia herniate through the tentorium cerebelli. Initially, compression of the hypothalamus causes a loss of sympathetic function, resulting in small but reactive pupils. However, soon the midbrain is compressed causing larger, unreactive pupils and decreased LOC. As herniation continues the basilar artery is torn which is uniformly fatal. 3) Uncal: The medial temporal lobe, also called the uncus, herniates through the tentorium cerebelli and begins to compress the midbrain causing an ipsilateral CN III palsy. Soon thereafter, there will be decreased LOC. Importantly, paralysis can be on either side of the body. If the ipsilateral midbrain is compressed first, then a contralateral weakness occurs. If the midbrain is compressed such that it moves as a whole, it will be compressed by the contralateral tentorium cerebelli (in an area called Kernohan’s Notch), causing paralysis on the same side as the CN III palsy. Thus, it is more reliable to localize based on the location of the CN III palsy, not the body weakness. 4) Upward: The cerebellum and brainstem herniate superiorly through the tentorium cerebelli; this is caused by an expanding mass in the cerebellum. Compression of blood vessels can cause cerebellar infarcts. As the mass expands it compresses the fourth ventricle, causing an obstructive hydrocephalus, which rapidly worsens the already high ICP. 5) Tonsillar: The cerebellum and brainstem herniate through the foramen magnum. Decreased LOC and paralysis accompany deformations to the brainstem. Autonomic instabilities involving blood pressure, pulse and respiratory rate ensue, and death follows shortly after. Colloquially, tonsillar herniation is known as ‘coning’. Any herniation syndrome requires prompt neurosurgical intervention with either resection of the offending mass lesion or by providing an alternate way for the brain to release pressure. This is usually achieved with the placement of a temporary external drain.

K25928_C007.indd 262

12/01/2017 12:19

Case 15: The 46-year-old woman who kept tripping over her right foot

263

Case 15: The 46-year-old woman who kept tripping over her right foot You have been asked by a family doctor friend of yours to see a 46-year-old female migrant strawberry picker. Through her son, who translates for her, she states that over the last 2 months, she has had difficulty walking. Her son volunteers that she seems to trip over her right foot. She demonstrates this for you as she walks. You notice that she seems to hyperflex her right hip in order to get her foot to clear the floor and when she fails to do so you hear the foot loudly slap against the ground. With the aid of her son, you find her language to be of normal fluency, repetition and comprehension. Her pupils are equal and she has full eye movements and full visual fields. She does not have a facial droop and her tongue and palate are midline. Her formal power examination is recorded below, but she has normal power in her arms. She has normal tone and reflexes everywhere. Her toes are downgoing. Her sensory examination shows an almost complete loss of pinprick on the dorsal aspect of the foot and the lateral part of the leg up to the knee. Coordination testing was normal, except in the affected leg, due to weakness.

Power Examination Right

Left

HF

5

5

HE

5

5

KF

5

5

KE

5

5

ADF

2

5

APF

5

5

INV

5

5

EV

3

5

EHL

3

5

Where is the lesion? What is going on here?

K25928_C007.indd 263

12/01/2017 12:19

264

Case 15: Findings

Case 15: Findings

Some individual figures adapted from Waxman SG. Clinical Neuroanatomy. Lange, New York, 2013.

K25928_C007.indd 264

12/01/2017 12:19

Case 15: Solution

265

Case 15: Solution 1. Does the patient have UMN weakness, LMN weakness or both? Our patient has isolated weakness of her right foot. This could be caused by an UMN lesion but is unlikely as the lesion would have to be quite small to only involve the foot. Our patient’s tone and reflexes are normal and her toes are downgoing, which are not in keeping with an UMN type problem. It would appear that the foot weakness is more in keeping with a LMN presentation and that the lesion lies in the PNS. Possible localizations include a radiculopathy, plexopathy and neuropathy. 2. Do the symptoms correspond to a single myotome and dermatome? Consulting our table of dominant myotomes (Fig.  5.5) we see that ankle dorsiflexion (ADF) is innervated by L4/L5, and the extensor hallucis longus (EHL) by L5. The foot evertors are not included in our table as they have innervation from many different spinal roots. It seems that the common root among the affected muscles is L5. What does the L5 dermatome look like? Consulting the table of dermatomes (Fig. 5.15) it appears that the L5 dermatome matches our patient’s sensory loss. An L5 radiculopathy perfectly explains our patient’s symptoms… humor me though and see if you can match the dysfunction to a single nerve. 3. Does the pattern of symptoms correspond to a single nerve? Our patient has weakness of the ankle dorsiflexion, first toe extension, and foot eversion. Recall from Chapter 5 that the muscles responsible for these functions are all innervated by the common peroneal nerve. In addition, we see that the sensory innervation from the common peroneal nerve matches our patient’s complaints. We seem to be in quite a pickle; an L5 radiculopathy and a common peroneal neuropathy both account for our patient’s presentation. Which is the correct one? Differentiating between an L5 radiculopathy and a common peroneal neuropathy is a classic neuroanatomy localization question. It is probably so popular both because of its educational value in demonstrating the neuroanatomy of the foot, and because the management of the two possibilities is so different. An L5 radiculopathy may require urgent neurosurgical decompression; a peroneal neuropathy will likely improve significantly with conservative management.

K25928_C007.indd 265

12/01/2017 12:19

266

Case 15: Solution

How do we approach such a problem? Firstly, note that the sensory examination is useless since the L5 dermatome and the common peroneal sensory area are virtually identical. Whenever you are confronted by a radiculopathy vs. neuropathy problem (such as an L3 radiculopathy vs. femoral neuropathy, or C8 radiculopathy vs. ulnar neuropathy), the key is to find a muscle that is supplied by the same nerve root but a different nerve. If that muscle is weak then the lesion lies in the nerve root. However, if that muscle is normal strength, the lesion lies in the nerve. For this case the key muscle turns out to be tibilais posterior, which inverts the foot. This muscle is supplied by L5, but is innervated by the tibial nerve, not the common peroneal nerve. We see that our patient’s tibialis posterior strength is normal. Thus we can conclude that our patient has a peroneal neuropathy. The peroneal nerve is exquisitely sensitive to compression. It runs right beside the head of the fibula. Whenever a person crosses their legs, they can cause the nerve to be pressed against the fibula. People who do this excessively have been known to cause transient palsies of their peroneal nerve. The same occurs whenever a person squats, so professions that due this often (bricklayers, floorers, or certain fruit pickers), also can cause significant damage to the nerve, giving it the nickname, the strawberry picker’s palsy.

K25928_C007.indd 266

12/01/2017 12:19

Case 15: Clinical Pearl

267

Figure 7.15 Detailed Anatomy of the Lumbosacral Region

K25928_C007.indd 267

12/01/2017 12:19

268

Case 15: Clinical Pearl

Clinical Pearl: Detailed Anatomy of the Lumbosacral Region In our previous case, our main consideration was a L5 radiculopathy or a common peroneal neuropathy. The neuroanatomy of the cauda equina can be challenging due to its varied presentations, so we should take a moment to review it here. The most common cause of an L5 radiculopathy is a disc herniation. Recall from Chapter 4 that the spinal cord sits between bones called vertebrae, and that between the vertebra are discs that act as shock absorbers. The spinal cord terminates at the conus medullaris at L1. Nerve roots beyond L1 travel together as the cauda equina and eventually leave the vertebral column via the neural foramen. Disc herniation can occur in two ways; posterolateral herniation and central herniation. In order to understand the symptoms produced, two facts must be remembered: 1. The further down the cord, the more central and posterior the nerve roots lie in the vertebral coloumn; i.e., the L2 roots lie more anteriolaterally than the S5 roots which lie very central and posterior. 2. Nerve roots exit very high in their foramen. Let’s consider a posterolateral herniation at the L5/S1 disc space. The L5 nerve root exits very high in its foramen, and thus is unlikely to be damaged by the herniation. However, the S1 root is quite vulnerable. Thus, posterolateral herniation tends to damage the root below them. Now consider a central herniation, again at the L5/S1 interspace. The central herniation damages the roots that are most central; in our example this is the S2 root. Then as the herniation worsens, the earlier nerve roots become involved in the reverse order, finally ending with the L5 nerve roots. Thus, a central herniation often damages the most distal nerve roots first, but if it is extensive enough it can involve all the nerve roots up to the level of the herniation. These two very different clinical presentations can be understood easily by remembering the two rules above.

K25928_C007.indd 268

12/01/2017 12:19

Case 16: The 42-year-old kickboxing woman

269

Case 16: The 42-year-old kickboxing woman You’re seeing a distressed 42-year-old woman who presents 3  days after taking up kickboxing. She came to the Emergency Department because of intractable nausea. She states that she’s been very dizzy and “off balance” recently, resulting in a fall. She wonders if she has the flu. The Emergency Doctor treats her nausea, but also notices her pupils are unequal and so pages you to come see her immediately. When you see her she repeats her history for you and you notice she has mild dysarthria; however, her formal language examination shows normal fluency, repetition and comprehension. When you examine her, you notice that she has a subtle ptosis of her right eyelid. In bright light her pupils measure 2  mm on the right and 3  mm on the left. This difference increases in the dark; her right pupil increases to 3 mm but the left increases to 5 mm. She has full visual fields and eye movements. She does not have a facial droop. However, when you ask her to open her mouth, her palate does not raise as much on the right side. Motor examination, including tone, reflexes and power were all normal. She has downgoing toes. Pinprick testing revealed, curiously, that she has decreased pinprick sensation on the right side of her face and on the left side of her body. You repeat this several times to make sure. Vibration testing with tuning fork is normal. Coordination testing revealed she is ataxic on her right side. Distressed, she asks you to place a cold compress on her forehead. As you walk away she mentions to you it doesn’t feel cold on the right side. Where is the lesion? What is going on here?

K25928_C007.indd 269

12/01/2017 12:19

270

Case 16: Findings

Case 16: Findings

K25928_C007.indd 270

12/01/2017 12:19

Case 16: Solution

271

Case 16: Solution 1. Does the patient have UMN weakness, LMN weakness or both? Unfortunately there are no motor symptoms in this case to guide us. Hopefully we’ll have more luck with her sensory exam. 2. Does the pattern of long tract symptoms suggest a localization? Our patient does not have any impairment of vibration, so we can conclude her DC are normal. Thankfully, her pinprick examination reveals an Immediate Localization! Our patient complains of pinprick loss on the right side of the face, but the left side of the body; this is an example of crossed signs, and can only be produced by a brainstem lesion (see Chapter 3). Thus, our initial localization is the brainstem. 3. What is the highest level of dysfunction? We need to see which CN are affected to help us localize to the midbrain, pons or medulla. Since her eye movements are not affected (CN III, CN IV and CN VI) we can conclude that her midbrain and pons are both normal. Looking through the rest of her symptoms, we see that her palate rose asymmetrically; both CN IX and CN X innervate the palate. We can thus refine our initial localization to be the medulla. 4. What is affected? Let’s work through our patient’s complaints. Our patient has an anisocoria that is greater in the dark than in light, which means she has Horner’s syndrome, and that the smaller pupil (right) is the abnormal one. Since the sympathetics do not decussate this tells us that the right sympathetic pathway is involved. Her dysarthria is likely caused by her palate rising asymmetrically. These functions are innervated by CN IX and X and controlled by a single nucleus, the nucleus ambiguus. Thus, the right nucleus ambiguus is involved. She is ataxic on the right side; the spinocerebellar tract doesn’t decussate so we know the right spinocerebellar tract is affected. She is dizzy and off balance, likely caused by dysfunction of the right vestibular nucleus of CN VIII. The decreased pinprick and temperature sensation in the right face means the right CN V nucleus must be affected. The decreased pinprick on the left side of her body must be due to a lesion of the right ST. It would seem that the entire lateral aspect of the right medulla is affected; indeed this is a case of lateral medullary syndrome or Wallenberg syndrome (named after the physician whom first described it). In this case it was caused by a traumatic dissection of the vertebral artery, likely due to the kickboxing, which then embolized causing a stroke.

K25928_C007.indd 271

12/01/2017 12:19

272

Case 16: Clinical Pearl

Figure 7.16 Vertebral Artery Dissection

K25928_C007.indd 272

12/01/2017 12:19

Case 16: Clinical Pearl

273

Clinical Pearl: Vertebral Artery Dissection Artery dissection is an important mechanism of stroke. The walls of arteries are composed of three layers: the adventitia (outermost layer), media (middle layer) and intima (innermost layer). Kickboxing, chiropractic manipulation, intubation, even sneezing and coughing are all types of trauma that have been known to “kink” arteries and cause microtears of their walls. The tear slowly expands and ‘dissects’ the artery wall layers apart until there is a true lumen, where normal blood flows, and a false lumen, where blood gets stuck and subsequently clots. Eventually a piece of clot can break off and travel up to the brain and interrupt blood flow, resulting in a stroke. For mechanical reasons, the vertebral arteries seem more likely to dissect than the carotid arteries. Typical treatment is with an anticoagulant or antiplatelet for 3–6 months until the false lumen has resolved on repeat imaging.

K25928_C007.indd 273

12/01/2017 12:19

Case 17: The 31-year-old woman who saw double

275

Case 17: The 31-year-old woman who saw double You’re asked to see a 31-year-old woman who comes to the Emergency Department with new type of headache. She states that she used to be headache free, but over the last 18 months she has had headaches nearly every day for which she was seeing her family doctor. She described these headaches as a dull ache, lasting all day, and about 6/10 in intensity. Today, however, she was typing at her keyboard when she developed a 9/10 in intensity headache that came on so quickly it was “like someone hit me over the head with a baseball bat.” She stumbled across the room and decided to call an ambulance because she was seeing double everywhere. When you examine her, you feel a sinking feeling in your stomach as you see that her blood pressure is 86/42 and dropping. You regain your composure enough to examine her formally. Her language examination shows normal fluency, repetition and comprehension. You immediately notice that her pupils are unequal; her left pupil is 2 mm and her right is 4 mm. The right pupil barely constricts to light and has a partial ptosis of the eyelid. Curiously, you find that she has lost her right visual field in her right eye, but she has also lost her left visual field in her left eye. She complains of diplopia in all directions of gaze; her left eye appears to move well, but the right eye barely moves at all. Pinprick examination of the face was normal on the left side. On the right side she reports the pin feels dull in the forehead and cheek, but the area just above the chin is normal. She does not have a facial droop, and her tongue and palate are midline. Formal motor examination shows normal tone, 2+ reflexes and normal power in the arms and legs. Sensory examination of the body is normal. Coordination testing is also normal. Where is the lesion? What is going on here?

K25928_C007.indd 275

12/01/2017 12:19

276

Case 17: Findings

Case 17: Findings

CN: cranial nerve.

K25928_C007.indd 276

12/01/2017 12:19

Case 17: Solution

277

Case 17: Solution 1. Does the patient have UMN weakness, LMN weakness or both? In this case, we have no motor symptoms to help guide us. 2. Does the pattern of long tract symptoms suggest a localization? We are not having much luck with this case; our patient’s sensory examination of the arms and legs were normal. We can only go on the fact that all of her symptoms are confined to the head, so our lesion must lie above the cervical spine. 3. What is the highest level of dysfunction? Let’s go through our patient’s complaints. Our patient’s right pupil is large and barely constricts. She also has a ptosis of the right eyelid; both of these support a right sided CN III palsy. She has diplopia in all directions of gaze, and the right eye barely moves, which suggests that CN III, CN IV and CN VI must be involved. Finally, she states that the pin is dull on the right forehead (CN V1) and right cheek (CN V2). Putting this together, you could argue for a large lesion that involves the midbrain (CN III and CN IV), and the pons (CN V1, CN V2 and CN VI). However, that would be a lesion of considerable size; remember the Rule of 4. CN III, CN IV and CN VI are all medial structures and CN V is a lateral structure. For such a large lesion to not involve ANY of the long tracts would be very unlikely. Can anything else help us? Well our patient has a visual field defect, which can only be produced by a cortical lesion. She has lost right sided vision in her right eye and left sided vision of her left eye. Another way of saying this is that she has lost the temporal aspect of her vision in both eyes; she has a bitemporal hemianopsia. This can only be caused by a lesion of the optic chiasm, as it is there that the temporal fields decussate (Fig. 3.5). The optic chiasm is our initial localization. 4. What is affected? How can we reconcile an optic chiasm lesion with dysfunction of the right CN III, CN IV, CN VI, and CN V1 and CN V2? Recall Fig. 3.11. All of these CN pass through the cavernous sinus, which lies directly below the optic chiasm. In this case the woman had a pituitary gland tumor, which caused her dull headaches over the last 18 months. The tumor then ruptured on the right side, causing bleeding into the cavernous sinus. The bleeding caused compression of the optic chiasm and CN. This

K25928_C007.indd 277

12/01/2017 12:19

278

Case 17: Solution

is a life threatening condition because the pituitary itself is soon compressed, causing a defect in hormone secretion, including those hormones that support blood pressure, as evidenced by our patient’s very low readings.

K25928_C007.indd 278

12/01/2017 12:19

Case 17: Clinical Pearl

279

Figure 7.17 Relative Afferent Pupillary Defect

K25928_C007.indd 279

12/01/2017 12:19

280

Case 17: Clinical Pearl

Clinical Pearl: Relative Afferent Pupillary Defect A relative afferent pupillary defect (RAPD) is a related condition to the case just presented and often serves as a frequent source of frustration for students. A RAPD, also known as a Marcus Gunn pupil (after the ophthalmologist who first described it), occurs whenever there is an incomplete lesion to one optic nerve. This incomplete lesion still allows the neuron to send signals to the midbrain, however the strength of the signal is reduced compared to the unaffected side. The best way to understand what happens here is to work through an example. Let’s say that the right optic nerve suffers an insult and only has one half of the working neurons compared to the left side. Stated another way, what if the right eye could only detect one half of the light that the left eye does? Let’s work through this example. You decide to shine light into the right eye (“Case A” in Fig. 7.17). Some light gets through, and the pupillary constriction pathway is completely intact, so the right and left pupil constrict equally due to the dual innervation of the pupils. You then decide to shine light into the left eye (“Case B”). This eye detects double the light than in “Case A.” The pupils both constrict equally, but as there is more stimulating light, they constrict to a smaller size than in “Case A.” Note that there is never an anisocoria of the pupils! In both cases the pupils constricted equally. We now understand why it is called a relative afferent pupillary defect. It’s relative because both eyes detect light, but one detects relatively less than the other. It’s afferent because it has to do with signal input; the lesion occurs to the afferent pathway (the optic nerve). Wait a moment, since both pupils constrict, how are you going to detect an RAPD on examination? If you had shone light in the right eye, noticed the weak constriction, and then waited 5 minutes to shine light in the left eye, to notice the stronger constriction, you may have missed the difference in size between the two responses. The solution is to do the swinging light test. In this case you swing the light quickly between the pupils. Initially you shine light into the right eye (“Case A”), note the weak constriction of the pupils, and then shine light into the left eye (“Case B”) and note the strong constriction of the pupils. You then swing the light back to the right eye (“Case C”). As you do the pupils will appear to paradoxically dilate; they are still constricted compared to their baseline state, but less so than when you shone light into the left eye. This paradoxical dilation is the key feature on examination to confirm a RAPD.

K25928_C007.indd 280

12/01/2017 12:19

Case 18: The 32-year-old man with the large ego

281

Case 18: The 32-year-old man with the large ego A 32-year-old man presented to the Emergency Department with a 3-day history of leg pain, left worse than right. He reports that he hurt himself at the gym when someone said to him “do you even squat bro?”; after attempting to disprove his opponent by overloading the bar, he immediately felt his feet go numb. Disturbed by the experience, he went home, hoping that his ego was the only thing that was damaged. However, 12 hours later he began to notice a sharp pain in the buttocks bilaterally. He says that this pain gradually spread over the entirety of both legs. He also states he hasn’t had a bowel movement since the event at the gym. However, he only decided to seek medical attention when, after 24 hours of not urinating, he spontaneously passed urine and did not notice it until his wife pointed out his pants were wet. He begs you for medication, stating the pain is excruciating. From conversing with him about his history you determine he has normal fluency and comprehension. His ability to repeat a sentence is also intact. He has full visual fields and eye movements, and his pupils are equal and reactive to light. He does not have a facial droop and his palate and tongue are midline. Motor and sensory examination in the arms is normal. When you examine his legs, you do not find any atrophy or fasciculations, but they do have decreased tone. His reflexes are completely absent at both the ankle and knee and his toes are downgoing. He is quite weak in the legs, as shown below. Sensory examination shows decreased vibration at the ankles and knees. Pinprick testing showed lack of sensation along the anterior part of the leg to just above the knee. He had decreased sensation over the penis, scrotum and sacral area. Coordination testing was normal in the arms, and not tested in the legs, given the weakness.

Power Examination Right

Left

HF

5

5

HE

2

2

KE

4-

3

KF

2

2

ADF

3

3

APF

2

2

EHL

3

3

Where is the lesion? What is going on here?

K25928_C007.indd 281

12/01/2017 12:19

282

Case 18: Findings

Case 18: Findings

Some individual figures adapted from Waxman SG. Clinical Neuroanatomy. Lange, New York, 2013.

K25928_C007.indd 282

12/01/2017 12:19

Case 18: Solution

283

Case 18: Solution 1. Does the patient have UMN weakness, LMN weakness or both? Our patient has decreased tone in both legs, as well as absent ankle and knee reflexes, pointing to a LMN type problem. Note that downgoing toes are a normal finding, and should not be used to conclude that the patient has LMN type weakness. However, the constellation of findings above points to a lesion in the PNS. Possible localizations include a radiculopathy, plexopathy or neuropathy. 2. Do the symptoms correspond to a single myotome and dermatome? Let’s review our table of dominant myotomes (Fig. 5.5). In this case only the hip flexors (L2) are of normal power. The knee extensors (KE) (L3), knee flexors (KF) (S2), ADF (L4/L5), ankle plantar flexion (APF) (S1), EHL (L5), and hip extensors (HE) (S1) seem to all be affected. Certainly, this doesn’t correspond to a single myotome. 3. Does the pattern of symptoms correspond to a single nerve? Our patient has weakness of nearly every muscle group, so a lesion in a single nerve is impossible. Indeed, the femoral nerve (KE), sciatic nerve (KF), common peroneal nerve [ADF, extensor hallucis (EH)], tibial nerve (APF), and inferior gluteal nerve (HE) are all involved. It would seem that to produce this pattern of loss, all of the above nerves would need to be affected, on both sides! Where do we go from here? According to our localization algorithm, if we have concluded that the dysfunction isn’t due to a lesion or a single nerve root or single peripheral nerve, we have to either conclude a) the lesion is in the plexus or b) there isn’t a focal lesion, and a generalized disease process is causing dysfunction of multiple parts of the nervous system. The argument that this is due to a lumbosacral plexus lesion is made difficult because both legs are affected, implying that you would need two (large, in this case) lesions to affect both plexuses. Before we conclude that there is a diffuse disease at work, recall Fig.  7.15; all the roots of the lumbosacral area travel in very close proximity. A central herniation of a disc could affect all the roots from the level of the herniation down. We now need to figure out the level of this herniation. The ankle (S1, S2) and knee (L3, L4) reflexes are both absent, so we know that the lesion is at least at the L3 level. Going

K25928_C007.indd 283

12/01/2017 12:19

284

Case 18: Solution

back to the table of dominant myotomes (Fig. 5.5) the hip flexors, which are full power, are innervated by L2. We can thus conclude that our lesion is at L3. The above is called a cauda equina syndrome, because it doesn’t only involve the nerve roots as they travel in the cauda equina, before they join the lumbosacral plexus. In this case the patient’s overzealous squat caused a disc to rupture and herniate centrally, causing impingement of nerve roots.

K25928_C007.indd 284

12/01/2017 12:19

Case 18: Clinical Pearl

285

Figure 7.18 Lumbar Puncture

CSF: cerebrospinal fluid; GBS: Guillain Barré syndrome.

K25928_C007.indd 285

12/01/2017 12:19

286

Case 18: Clinical Pearl

Clinical Pearl: Lumbar Puncture If the mass responsible for the compression in the above case had been anything other than a ruptured disc, our patient would have likely undergone a lumbar puncture. A lumbar puncture is a useful diagnostic test, and can also be a therapeutic intervention. The goal of a lumbar puncture is, in most cases, to obtain cerebrospinal fluid (CSF) for analysis; CSF is most often used to confirm an infectious meningitis (see bottom half of Fig.  7.18) but can also be used to diagnosis Guillain Barré syndrome (GBS), subarachnoid hemorrhage, multiple sclerosis, and malignancies. Therapeutically, it can be used to instill chemotherapy, as well as decrease elevated ICP by removing CSF. In a lumbar puncture, a needle is inserted in the L3–L4 interspace, as shown in the top half of Fig. 7.18. As we saw in the case above, the spinal cord ends at L1, so the needle is being inserted into the space occupied by the cauda equina. Damage to the spinal cord itself is not possible, and a lumbar puncture is viewed as a very safe procedure. As the needle is inserted it passes through several layers, the first of which is the skin and then the subcutaneous tissue. Next, it will pass through the ligaments that hold the vertebrae together. Finally, it pierces the dura mater and arachnoid mater to enter the subarachnoid space, where CSF is located. The ability to quickly interpret lab values obtained from a lumbar puncture is important. Typical CSF measures include protein concentration, glucose concentration and the concentration of white blood cells (usually just referred to as “cells”). A normal lumbar puncture should be clear in colour, with very little protein, and a maximum of 5 white blood cells, as the CSF is normally sterile. In a bacterial infection, the protein is very elevated, which indicates that the blood brain barrier has been broken. The glucose is decreased because it is being consumed by bacteria, and the cell count is high with a differential indicating neutrophil predominance. A viral infection is more subtle in presentation; the protein is not as high, the glucose is normal, and there are typically less cells, though they are more lymphocytic in nature. GBS will have a markedly elevated protein, but NO increase in cells, as there is no infection, and the glucose will be normal.

K25928_C007.indd 286

12/01/2017 12:19

Case 19: The 59-year-old woman who was like stone

287

Case 19: The 59-year-old woman who was like stone You walk into the Emergency Department to see your next consult and find a despondent husband who brought in his 59-year-old wife. He states that over the last 18 months she has become “like stone;” she has become very withdrawn and moves very slowly, or not at all. Over this time, she refused to see a physician. He brought her to the Emergency Department because over the last week she developed severe urinary incontinence, though this didn’t seem to particularly bother her. When you examine her, you agree that she has very little spontaneous verbal output, and seems to just sit there; however, her ability to repeat and comprehend are intact. You find her CN to be normal; pupils are equal and reactive, she has full eye movements and visual fields, she does not have a facial droop and her tongue and palate are midline. Motor testing is normal in the arms. However, her legs demonstrate increased tone, 3+ reflexes and bilateral upgoing toes. She is quite weak in the legs; her formal power examination is recorded below. Her pinprick and vibration testing is normal. You decide to challenge her with some more strenuous sensory tests. You use your pen to trace out a number on both her legs, but she is quite unable to tell you what that number is; she cannot even identify that it was a number. However, when you repeat this on her hands, she (slowly) gets the correct answer. When you ask her to rub her ankle down her shin as part of the coordination examination, she just looks at you blankly. Similarly, when you ask her to walk she just looks at you confused. You demonstrate walking, but she struggles to lift her feet. She continues to look at you confused. Power Examination Right

Left

HF

4-

4-

HE

4-

4-

KE

4-

4-

KF

4-

4-

ADF

3

3

APF

4-

4-

EHL

4

4

Where is the lesion? What is going on here?

K25928_C007.indd 287

12/01/2017 12:19

288

Case 19: Findings

Case 19: Findings

K25928_C007.indd 288

12/01/2017 12:19

Case 19: Solution

289

Case 19: Solution 1. Does the patient have UMN weakness, LMN weakness or both? Our patient has weakness accompanied by increased reflexes, increased tone and upgoing toes; these are all signs of an UMN problem. Thus, we know that the lesion lies in the CNS. 2. Does the pattern of long tract symptoms suggest a localization? In this case, our patient has UMN weakness of both legs, suggesting both corticospinal tracts are involved. As we saw in Case 2, bilateral UMN leg weakness localizes to either the thoracic/lumbar cord or to the medial aspect of both motor cortices. In this case, our patient had normal pinprick and vibration testing. We must hope that her other symptoms help us to differentiate between these two localizations. 3. What is the highest level of dysfunction? While her primary sensory testing was normal, her further sensory testing reveals an Immediate Localization to the cortex! She cannot discern the number traced on her leg; this is an example of agraphesthesia. Similarly, the confused look the patient gives when you ask her to perform any task with the leg, such as walking or the heel to shin testing, is an example of apraxia. Our patient has intact proprioception and sufficient power to walk, but cannot figure out how to execute this complex motor task. Agraphesthesia and apraxia are examples of higher functions that can only be produced by a cortical lesion. Thus, our localization must involve the medial aspect of both motor cortices. 4. What is affected? Now that we know that both cortices are involved we can draw them in cross section. Our patient also complains of incontinence, which we would expect since bowel and bladder function is also controlled by the medial aspect of the motor cortex. In general, both cortices need to be involved before incontinence occurs. The localization of the apraxia and agraphesthesia is nonspecific, but may involve the frontal lobe. In summary, we see that the medial side of both motor cortices is affected, and our patient has evidence of marked frontal lobe dysfunction. While this might seem a bizarre combination of symptoms, it is actually quite common and can be produced by bilateral ACA strokes, or a disease known as normal pressure hydrocephalus, which we will explore shortly. In this case, however, the patient had a parasagittal meningioma, a benign tumor, compressing all of the above structures. Removal of the tumor improved the weakness and incontinence greatly, but cognitive function only partially recovered.

K25928_C007.indd 289

12/01/2017 12:19

290

Case 19: Clinical Pearl

Figure 7.19 Hydrocephalus and Monro–Kellie Doctrine

Redrawn with permission from Waxman SG. Clinical Neuroanatomy. Lange, New York, 2013.

K25928_C007.indd 290

12/01/2017 12:19

Case 19: Clinical Pearl

291

Clinical Pearl: Hydrocephalus and Monro–Kellie Doctrine Certain tumors can obstruct the drainage of CSF and result in hydrocephalus (Greek for, literally, “waterhead”), which is defined as an abnormally high amount of CSF. Hydrocephalus can be classified into two types; unfortunately two equally common naming conventions exist, so one must be familiar with both. • Obstructive (also called noncommunicating) hydrocephalus. An extrinsic mass (usually a tumor, or clotted blood) blocks the flow of CSF, usually at the level of the cerebral aqueduct or the 4th ventricle. The choroid plexus continues to produce CSF however, and hydrocephalus results. As the blockage occurs in the inferior part of the ventricular system, the lateral ventricles swell in size significantly, which can be seen on imaging. It is called noncommunicating hydrocephalus as the two CSF compartments, head and spine, are cut off from each other by the compression. The majority of cases of hydrocephalus are of the obstructive type. • Nonobstructive (also called communicating) hydrocephalus. This results either from an overproduction of CSF by the choroid plexus (due to rare choroid plexus tumors), or blockage of the resoprtion of CSF by the arachnoid granulations. The arachnoid granulations can be prone to scarring which renders them ineffective, usually after an infection such as meningitis, or after a subarachnoid hemorrhage. On imaging, all the ventricles will be dilated, not just the lateral ones. Hydrocephalus ties in closely with the concept of ICP and the Monro–Kellie Doctrine (MK Doctrine). The MK Doctrine simply says that because the skull is a fixed volume, the volume (V) of the all contents of the skull must be a constant, specifically: VSkull = VBrain + VCSF + VArterial + VVenous Stated in words, the total volume of the skull is equal to the volume of the brain plus the volume of the CSF, arterial blood supply and venous blood supply. What if there is an expanding mass, such as a tumor? Then the MK Doctrine states that: VSkull = VBrain + VCSF + VArterial + VVenous +Vmass The brain will attempt to compensate for the increasing volume of the mass. Unfortunately, the brain can’t change its size, so VBrain is fixed. The first compensatory measure will be to decrease the output of CSF, thus decreasing VCSF. The brain will then increase venous drainage, decreasing VVenous. Finally, under truly dire circumstances the brain will decrease VArterial, but this greatly increases the risk of significant ischemia and stroke. When this fails, the relatively weak cortical veins will begin to be crushed under the enormous pressure, which blocks venous drainage, leading to massive spikes in ICP. Herniation of brain tissue through the foramen magnum, and death, is the final result.

K25928_C007.indd 291

12/01/2017 12:19

292

Case 19: Clinical Pearl

But what if the volume of the skull wasn’t fixed? Surgically, this can be achieved by placing an emergency drain, or by removing part of the skull. Usually a drain is the preferred method. This lowers the ICP by draining the CSF, and decreasing VCSF. This life saving measure can often be carried out in a manner of minutes at the bedside.

K25928_C007.indd 292

12/01/2017 12:19

Case 20: The 19-year-old woman with a single complaint

293

Case 20: The 19-year-old woman with a single complaint The next patient in your busy clinic is a 19-year-old woman referred to you by her ophthalmologist. She states that over the last 3 weeks she has had double vision. When you ask her if there is any pattern she mentions that she thinks it’s whenever she looks to the right. As you converse with her you find her fluency, repetition and understanding of language to be normal. You examine her eyes closely; you cannot find any visual field defects, pupillary abnormalities or any eyelid ptosis. Her eyes seem to be in perfect alignment with each other as she looks forward. She does not have any diplopia on upgaze, downgaze or leftward gaze, and her eyes move conjugately through these directions. However, when she looks to the right, her right eye abducts but her left eye does not move. She does not have a facial droop and her tongue and palate are midline. Motor examination reveals normal tone, 2+ reflexes and no weakness. Sensory testing to pinprick and vibration is normal. Coordination is normal in both the arms and the legs. It would seem that the inability of the left eye to adduct whenever she looks to the right is her sole abnormality on exam. Where is the lesion? What is going on here?

K25928_C007.indd 293

12/01/2017 12:19

294

Case 20: Findings

Case 20: Findings

PRRF: paramedian pontine reticular formation. Redrawn with permission from Greenberg D, et al. Clinical Neurology. McGraw–Hill Education, New York, 2015.

K25928_C007.indd 294

12/01/2017 12:19

Case 20: Solution

295

Case 20: Solution As our patient has absolutely no long tract signs whatsoever, we’ll make a brief departure from our localizing algorithm. Indeed our entire neurological examination is normal except for failure of the left eye to adduct. That’s it. Since this is an eye movement problem, lets return to the pathway for eye movements we outlined in Fig. 3.9, which is reproduced on the previous page. We’ll examine several potential localizations in turn and we will hopefully find our answer. 1. FEF lesion: As we know the FEF drive both eyes to the contralateral side. A lesion of the left FEF would mean that both eyes are strongly driven to the left side and unable to cross midline. Clearly this doesn’t account for our presentation. 2. CN III lesion: The medial rectus muscle controls eye adduction, so our patient could have a palsy of the left CN III. However, this would also cause problems with the superior rectus, inferior rectus and inferior oblique muscles resulting in the classic “down and out” eye. Let’s soldier on. 3. CN VI lesion: If we had a lesion to the right CN VI, the right eye would fail to abduct, but the left eye would still adduct. This is the inverse of the problem we are faced with. 4. Abducens Nucleus lesion: Remember that the abducens nucleus of CN VI initiates horizontal gaze for both eyes. The right abducens nucleus initiates rightward gaze for both eyes via the MLF. Thus, a lesion here would mean that both eyes would not move when attempting to look right. 5. MLF lesion: The left MLF connects the right abducens nucleus to the left CN III nucleus. A lesion here would not interfere with the abducens nucleus; the right abducens nucleus fires, and the right eye abducts. However, the signal to the left CN III nucleus is blocked, and the left eye fails to adduct. Eureka, we have found it! A lesion to the MLF results in an internuclear ophthalmoplegia, or INO, which is named because the lesion lies in between two nuclei. A lesion to any nucleus i.e., (CN VI or CN III) is termed a nuclear lesion. Similarly, a lesion to the FEF or anywhere along the path connecting the FEF to the brainstem is called a supranuclear lesion. An INO in the absence of any other neurological symptoms is a classical finding for multiple sclerosis (MS), and should be the first thing that springs to mind in an otherwise healthy young woman.

K25928_C007.indd 295

12/01/2017 12:19

296

Case 20: Clinical Pearl

Figure 7.20 Multiple Sclerosis

MS: multiple sclerosis.

K25928_C007.indd 296

12/01/2017 12:19

Case 20: Clinical Pearl

297

Clinical Pearl: Multiple Sclerosis MS is an autoimmune disease of the CNS. The immune system inappropriately attacks the myelin of neurons (Fig. 1.2) in a process called demyelination. This results in greatly slowed conduction of action potentials because they can no longer jump from Node of Ranvier to Node of Ranvier. However, because the axon is undamaged the neuron still lives, and oligodendrocytes can remyelinate the neuron, restoring at least part of its function. A key feature of MS is that it’s “multiple;” the patient suffers multiple attacks over time, and in multiple locations, producing different symptoms. In order to be diagnosed with MS the lesions need to satisfy McDonald Criteria and demonstrate dissemination in time and dissemination in space. This can either be fulfilled by evidence of clinical attacks or on MRI scan, or a combination of both. There are three typical locations that MS affects. The first is the optic nerve, leading to optic neuritis, in which the patient loses vision and has significant pain on moving the eye. The second is the spinal cord, which usually results in total dysfunction of the cord at the level which is attacked. The last is the brainstem, which can cause a variety of symptoms such as dysarthria, dysphagia, diplopia (as seen in this case, due to an INO), and weakness. MS can follow certain patterns over time. The most common type of MS is Relapsing Remitting MS, in which the patient experiences an attack of demyelination with subsequent symptoms, which then remit once the oligodendrocytes repair the damage. The patient returns to baseline until another attack, or relapse, occurs. However, long into the course of the disease, oligodendrocytes begin to fatigue and can no longer fully repair the demyelination. Clinically, the patient no longer returns to baseline between attacks and they begin to accumulate disability. Eventually they may switch to a secondarily progressive course in which they don’t suffer attacks at all and simply accumulate disability with time. Some patients have an atypical and rare version of MS in which they do not experience relapses at all; they immediately begin to accumulate disability. These patients have primary progressive multiple sclerosis.

K25928_C007.indd 297

12/01/2017 12:19

Case 21: The 62-year-old woman who kept burning her right hand

299

Case 21: The 62-year-old woman who kept burning her right hand It has been a long night, and you are about to go home, when you are paged to the Emergency Department to see a 62-year-old woman with a remote history of breast cancer. When you arrive, the Emergency Doctor apologizes for the consult, and tells you he thinks the patient is hysterical because her symptoms are “all left sided, except lack of pain, which is right sided.” When you question her, you find an odd constellation of complaints. She tells you she has had weakness in her left arm for the last 3 months, but her right hand feels “strange,” and she has burnt it several times without noticing. She’s had difficulty walking for the last 2  weeks. She finally came to the Emergency Department because she’s recently noticed that she has to void urine up to 10 times per day and she finds that when she gets the urge to void it is so strong that on several embarrassing occasions she has not made it to the bathroom in time. When you examine her, you find her language to have normal fluency, repetition and comprehension. On CN examination you see that she has full visual fields and eye movements, but her left pupil is 3  mm and her right pupil is 4  mm, which you find unsettling. You shut off the light and find the left increases to 4 mm, and the right increases to 6 mm. She does not have a facial droop and her tongue and palate are midline. She is weak on her left side, and her formal power examination is recorded below. She has spasticity in her left leg, but you wonder if her left arm has decreased tone. Her left bicep reflex is 1+, but her tricep, knee and ankle are all 3+ on the left side, and she has an upgoing left toe. She can not feel any vibration on the left side of her body below the level of the shoulder, but the right side is entirely normal. However, on pinprick testing the opposite was true, just as the Emergency Doctor reported; she cannot feel the pin on the right side below the level of the shoulder, but the left side is entirely normal. Coordination testing was deferred as she was quite weak.

Power Examination

Where is the lesion? What is going on here?

K25928_C007.indd 299

Right

Left

Deltoid

5

3

Bicep

5

3

Tricep

5

4

WE

5

4

FE

5

4

HF

5

4-

KE

5

4-

KF

5

4-

ADF

5

4-

APF

5

4-

12/01/2017 12:19

300

Case 21: Findings

Case 21: Findings

K25928_C007.indd 300

12/01/2017 12:19

Case 21: Solution

301

Case 21: Solution 1. Does the patient have UMN weakness, LMN weakness or both? Our patient has increased tone in the left leg, increased reflexes at the left ankle, knee and triceps and a left upgoing toe, which are all strongly indicative of an UMN type lesion. However, her left arm has decreased tone and her left biceps reflex is decreased, which are signs of a LMN type lesion. Thus, our patient has LMN type weakness at the bicep and UMN type symptoms for the muscles below this level. 2. Does the pattern of long tract symptoms suggest a localization? The combination of LMN type symptoms in one area of the body and UMN type symptoms below that area can only be produced by a spinal cord lesion. In addition, her sensory examination also suggests an Immediate Localization. Our patient has pinprick loss on the right, but vibration loss on the left. This is an example of crossed sensory signs, and can only be produced by a lesion to the spinal cord, which is our initial localization. 3. What is the highest level of dysfunction? Our patient complains of complete sensory loss below the level of the mid shoulder; this is a sensory level and is another Immediate Localization to the spinal cord. Consulting the dermatome map (Fig. 5.15) we see that this area is roughly innervated by C5 or C6. Our patient has LMN weakness (decreased tone and 1+ reflexes) in the bicep, which after checking our table of dominant myotomes (Fig. 5.5), we see is also innervated by C6. We can refine our localization to be the spinal cord at C6. 4. What is affected? Since we now know we’re in the cervical spinal cord, we can draw it in cross section. Let’s start with motor symptoms. We have LMN signs in the left bicep, so we know that the left AHC cell is involved. Below C6, we have UMN type weakness on the left so the left corticospinal tract must be involved. The patient complains of vibration loss below C6, so the left DC are involved. The patient also complains of loss of pinprick on the right. Since the ST decussates immediately, that means the left ST is involved. Adding it all up, we see virtually the entire left side of the spinal cord is affected; our overall localization is the left hemicord at C6. The total loss of one side of the cord is known as Brown–Sequard syndrome. In this case, a metastasis from our patient’s previous breast cancer was compressing the cord on the left side.

K25928_C007.indd 301

12/01/2017 12:19

302

Case 21: Solution

You think we’ve forgotten about the pupils don’t you? We see that the anisocoria is greater in dark than in light, so we have a Horner’s syndrome and the smaller pupil (left) is the affected one. Remember the sympathetics travel into the spinal cord until about C8/T1, as shown in Fig. 3.6. The sympathetics to the left pupil were affected by our lesion.

K25928_C007.indd 302

12/01/2017 12:19

Case 21: Clinical Pearl

303

Figure 7.21 Classification of Spinal Cord Lesions

K25928_C007.indd 303

12/01/2017 12:19

304

Case 21: Clinical Pearl

Clinical Pearl: Classification of Spinal Cord Lesions Whether the causative lesion of spinal cord dysfunction is located inside or outside of the spinal cord itself is of great clinical importance. For example, we know that multiple sclerosis is a disease that targets the spinal cord tissue itself, and needs to be treated medically. However, a slipped disc, or metastatic cancer, such as in our case, is a lesion that actually lies outside the spinal cord; they cause dysfunction by extrinsically compressing the cord and require surgical treatment. Overall, spinal cord lesions can arise from three different areas: 1. Intramedullary lesions (‘medulla’ is Latin for ‘middle’) These are lesions of the spinal cord tissue itself (either the gray or white matter, or both), such as the multiple sclerosis lesions mentioned above. Other examples include primary tumors of the CNS, and lesions caused by nutritional deficiencies. 2. Extradural lesions These lesions lie outside the dura mater of the meninges and hence lie completely outside the CNS. They cause dysfunction by compressing the cord. Examples include the slipped disc mentioned above, other trauma, and metastatic tumors. 3. Intradural extramedullary lesions These are lesions that arise from the meninges themselves. They are intradural because they are part of the CNS, but lie outside the cord, and so are extramedullary. They also cause symptoms by compressing the cord. The majority of these lesions are due to tumors of the meninges.

K25928_C007.indd 304

12/01/2017 12:19

Case 22: The 21-year-old man transferred from a remote hospital

305

Case 22: The 21-year-old man transferred from a remote hospital Your next patient is a 21-year-old man arriving to you from a small hospital up north via helicopter airlift. The patient was brought to the smaller hospital’s Emergency Department by his girlfriend after he fell several times in the last 24  hours. Flipping through the transfer notes, you see he was seen twice in the last 2 weeks. Originally he was brought in with dehydration after severe diarrhea and needed IV fluids. He was then seen 5 days later with a complaint of painful tingling in his feet up to his mid shin. He had blood-work drawn, which was normal and was then sent home. Now, he states that the painful tingling is present up to the hip level, and is also in the fingertips of both hands. He is unsure as to why he is falling and why he has transferred hospitals. As you converse with him you find his fluency, repetition and understanding of language to be normal. His CN are normal; he has full visual fields and eye movements, pupils that are equal and reactive, no facial droop and his tongue and palate are midline. His motor examination in the arms is normal. However, in his legs he has remarkably decreased tone. Despite several attempts, you cannot elicit reflexes at either the knee or the ankle. Power examination in the arms are normal; however, he has significant weakness of both legs. His toes are downgoing bilaterally. Sensory examination shows a complete loss to vibration in the legs and in the hands in the upper extremity. He can feel vibration in his elbows on both sides. He also shows a severe loss of proprioception in the legs. His pinprick examination shows some mild loss in the same areas. He is quite unsteady on his feet and he needs to be held up by his girlfriend in order to walk. Power Examination Right

Left

HF

4+

4+

HE

4

4

KE

4

4

KF

4-

4-

ADF

4-

4-

APF

4-

4-

EHL

3

3

Where is the lesion? What is going on here?

K25928_C007.indd 305

12/01/2017 12:19

306

Case 22: Findings

Case 22: Findings

Some individual figures adapted from Waxman SG. Clinical Neuroanatomy. Lange, New York, 2013.

K25928_C007.indd 306

12/01/2017 12:19

Case 22: Solution

307

Case 22: Solution 1. Does the patient have UMN weakness, LMN weakness or both? Our patient complains of bilateral leg weakness in the context of decreased tone and reflexes that cannot be elicited. This strongly points to a LMN problem and we can conclude our lesion lies in the PNS. Possible localizations include a radiculopathy, plexopathy or neuropathy. 2. Do the symptoms correspond to a single myotome and dermatome? Our patient has weakness of every muscle group of his legs; the HF (L2), KE (L3), KF (S2), ADF (L4/L5), APF (S1), EHL (L5), KF (S2), and HE (S1) seem to all be affected. He also has a complete loss of vibration and pinprick not only in the lower legs, but also in his hands! Clearly this massive amount of dysfunction cannot be due to a lesion of a single nerve root. 3. Does the pattern of symptoms correspond to a single nerve? Unfortunately we run into the same problem here; our patient’s pattern of weakness would involve the femoral nerve, inferior gluteal nerve, sciatic nerve, common peroneal nerve and the tibial nerve. This does not even take into account his sensory complaints in the hand. If our patient’s symptoms were confined to a single leg, we could argue that perhaps there was a lesion affecting the entire lumbosacral plexus. However, even this is quite unlikely as a lesion would have to be of quite impressive size to do so; in addition our patient also has symptoms in his hands. This makes a lumbosacral lesion impossible. What are we left with? The symptoms in this case cannot be caused by a focal lesion. A nonfocal disease process (such as diabetes, thyroid dysfunction and other genetic conditions) can attack the nerves as a whole resulting in multiple diffuse lesions. In this case, our patient’s symptoms are best classified as a length dependent polyneuropathy, in which the longest nerves are affected first. Dysfunction begins in the most distal muscles and sensory areas. As the disease progresses, it moves up the body and affects all the nerves it comes across. Once the disease reaches just above the knees, the nerves of the arm will begin to be affected. What could the etiology of this length dependent polyneuropathy be? There are several clues in the case, the most important of which is the loss of vibration/proprioception and the complete absence of reflexes. In fact, this represents a prototypical case of GuillainBarré syndrome (GBS), which we’ll turn our attention to now.

K25928_C007.indd 307

12/01/2017 12:19

308

Case 22: Clinical Pearl

Figure 7.22 Guillain–Barré Syndrome

K25928_C007.indd 308

12/01/2017 12:19

Case 22: Clinical Pearl

309

Clinical Pearl: Guillain–Barré Syndrome GBS, named after the French physicians who first described it in 1916, is more accurately known as acute inflammatory demyelinating polyneuropathy (AIDP). Classically, GBS occurs after an exposure to a trigger, such as a recent infection, or in rare cases, a vaccination. The most common cause is Campylobacter jejuni infection; this bacteria is often present in rivers and other bodies of water in a forest. The infection is believed to trigger an autoimmune attack in which the immune system mistakes the myelin sheaths of the PNS for the offending agent; demyelination subsequently occurs. For reasons that are not completely understood, the longest nerves are typically affected first. The patient’s symptoms typically begin in their feet and then slowly ascend up the body. Symptoms are symmetric and bilateral. Motor, sensory and autonomic nerves can all be involved. Compromise of the autonomic nerves can lead to dangerous levels of dysfunction; by the time that the level of the thorax is involved, patients can have wild fluctuations in heart rate and blood pressure. Most concerning is involvement of the nerves leading to weakness of the diaphragm and eventual respiratory failure. Due to involvement of both sensory and motor systems, patients quickly lose their reflexes completely. This so called areflexic paralysis is a highly specific sign of GBS. Another highly specific finding is albuminocytologic dissociation found on lumbar puncture. CSF analysis shows the protein is highly elevated indicating breakdown of the blood brain barrier, but there is no increase in the amount of cells present, indicating lack of an infection. The speed of the attack in GBS can be extremely varied. Some patients decline and need ICU admission within a matter of hours, whereas some will only suffer modest involvement and recover within several weeks. In addition, there are no predictors as to how far GBS will ascend up the body before beginning to remit. The natural history of GBS is such that it spontaneously remits, and does not recur, though there are exceptions. Medical treatment is largely supportive. Typically, remyelination occurs and the patient recovers neurological function. Despite the advances in intensive care, GBS carries a 5% mortality rate, and 20% of patients will have permanent neurological deficits.

K25928_C007.indd 309

12/01/2017 12:19

Case 23: The 17-year-old quarterback who passed out

311

Case 23: The 17-year-old quarterback who passed out You are enjoying a beautiful Sunday afternoon in September watching your brother compete in a high school football game. Much to your dismay, your brother’s team’s star quarterback takes a brutal tackle, and appears knocked out. You slowly get up from the stands, and walk down onto the field, but, to your surprise, he gets up and appears fine. You breathe a sigh of relief and sit back down as you have a score to settle with a delicious smelling hotdog. Several hours later, you begin your shift in the Emergency Department. You go to see your first patient and recognize the quarterback from before. His football team mates tell you he was not very lively during the after party and only had one beer. They found him passed out in the bathroom, and they were unable to awaken him, so they brought him to the hospital. When you examine him, he keeps dozing off unless you yell his name in his ear. He is very slow to respond and when he does talk you find him to be very confused. Given his somnolence you defer from formally evaluating his language, but he does, with prompting, obey your commands. His right pupil is large and does not constrict as much to light as the left pupil. When you shut off the lights, the left pupil dilates, but the right remains the same size. However, both eyes appear midline and he has full eye movements. He has mild ptosis of the eyelid on the right side. He has a mild facial droop on the right side, but he can move his eyebrows. The rest of the CN can’t reliably be assessed. A full motor examination is impossible. When you look at him, you see that he has assumed a rather odd posture. His right arm is extended at the elbow, but flexed at the wrist. His right leg is extended at the knee, and the foot is plantar flexed and internally rotated. He has tremendously increased tone on the right, so much so that you can barely move the limbs. The left side is normal. He has 3+ reflexes in the arm, and his knee and ankle reflexes elicit clonus. His right toe is strongly upgoing. Once again, the left side is normal. The only part of the sensory examination that can be performed is reaction to pain; he does not seem to wince when you poke him in the right arm or leg. When you rub on his sternum he tries to swat you away with this left arm, but not his right. Where is the lesion? What is going on here?

K25928_C007.indd 311

12/01/2017 12:19

312

Case 23: Findings

Case 23: Findings

K25928_C007.indd 312

12/01/2017 12:19

Case 23: Solution

313

Case 23: Solution 1. Does the patient have UMN weakness, LMN weakness or both? Our patient has tremendously increased tone and increased reflexes on the right (the knee and ankle being graded at 4+ due to the clonus). He also has a right upgoing toe. These are all signs of UMN pathology, so we can conclude that the lesion lies in the CNS. 2. Does the pattern of long tract symptoms suggest a localization? Rubbing on someone’s sternum inflicts significant pain. We can conclude that our patient is weak on the right since he only used his left arm to try to get us to stop. He also doesn’t feel pain in the right arm or leg. Unfortunately this doesn’t help us too much, and the lesion could be in cortex, internal capsule, brainstem or cervical spine. 3. What is the highest level of dysfunction? Our patient has an anisocoria; the right pupil is larger than the left one. The right does not constrict well to light, so we suspect a parasympathetic problem. This is confirmed by noting the anisocoria is greater in light than dark. This means that there is a parasympathetic problem, and that the larger pupil (right) is the abnormal one. This is supported by our finding of mild ptosis on the right. The parasympathetics are carried by CN III; either the CN III nuclei in the midbrain or CN III itself is affected by the lesion. 4. What is affected? The lesion involves either the right CN III or its nucleus in the midbrain. Since our patient also has weakness and numbness, we can safely say our lesion must be in the midbrain. Our patient has UMN symptoms in the right arm and leg. Since the corticospinal tract decussates at the level of the medulla, this means that the left corticospinal tract is affected. But wait a moment, this would imply two separate lesions! Examining the cross section of the midbrain we see that the right CN III and the left corticospinal tract are very far apart from each other. How are our patient’s symptoms possible? In order to answer the question, you will have to be told that the quarterback suffered a large bleed on the right side of the brain, when he was initially hit. This began to expand, and eventually put local pressure on the right side of the midbrain. This local pressure caused the right sided CN III findings. However, the pressure then became so high that the entire midbrain shifted to the left side of the skull and it began to press against the

K25928_C007.indd 313

12/01/2017 12:19

314

Case 23: Solution

dura at the level of the tentorum cerebelli. The compression of the left midbrain caused dysfunction of the left corticospinal tract. The indentation of the contralateral midbrain by the tentorum cerebelli is also known as compression due to Kernohan’s Notch, after the physician who first described it. It is one of the few false localizing signs in neurology. As you can see, if you had localized based on the more obvious pattern of weakness, and not examined the eyes (but you would never do that would you?), you would have concluded the lesion was on the left. This was of more clinical importance 50 years ago before the advent of computed tomography (CT) scanners; during this time physicians had to use the bedside examination to determine which side of the head to drill in order to relieve the increased ICP. However, it remains true that in cases of decreased LOC, localization should be decided based on eye findings, rather than the pattern of weakness!

K25928_C007.indd 314

12/01/2017 12:19

Case 23: Clinical Pearl

315

Figure 7.23 Cerebral Bleeds

K25928_C007.indd 315

12/01/2017 12:19

316

Case 23: Clinical Pearl

Clinical Pearl: Cerebral Bleeds The brain is susceptible to four different types of hemorrhages. They are named according to where the collection of blood, called a hematoma, forms. 1. Epidural Hematoma: This hemorrhage is typically caused by a skull fracture that tears the middle meningeal artery (MMA), and is what happened to our unfortunate quarterback in the previous case. The MMA runs between the skull bone and the dura mater, so blood accumulates in the epidural space. Typically the initial trauma that ruptures the MMA also transiently stuns the brain resulting in a brief loss of consciousness. The brain recovers and consciousness is regained, but blood begins to slowly pool in the epidural space. Eventually the hematoma grows so large it begins to cause the brain to herniate, resulting in another, permanent (if untreated) loss of consciousness. Importantly, there is a “lucid interval” between the episodes of loss of consciousness, as we saw with our football player. On imaging, the hematoma often appears “lens shaped” as it is confined by the cerebral sutures. If it is detected before herniation occurs it is highly treatable with a favorable prognosis. 2. Subdural Hematoma: This hemorrhage is caused by rupture of the bridging veins, which run between the subarachnoid space and the dural sinuses. Blood accumulates in the subdural space between the dura mater and the arachnoid mater. It usually occurs in the elderly as a result of falls. There is no lucid interval and a decreased level of consciousness can occur soon after the initial trauma. On imaging the hematoma takes on a more crescent appearance as it is not confined by the cerebral sutures. Prognosis is highly dependent on how early it is treated. 3. Subarachnoid hemorrhage: This hemorrhage typically occurs as the result of aneurysmal rupture of an artery in the Circle of Willis. Since these arteries lie in the subarachnoid space, blood quickly accumulates there and mixes with CSF; this allows for diagnosis by lumbar puncture. Patients typically present with a “thunderclap headache,” which comes on fulminantly. On imaging blood is found within the various sulci and fissures of the brain. The prognosis is poor even with prompt treatment. 4. Intracerebral hemorrhage: This hemorrhage is caused by a rupture of a small artery deep within the brain tissue itself. The hematoma occurs in whatever part of the brain the rupture occurred. It is common in the elderly and in those with uncontrolled hypertension. Its symptoms depend on its location but it is usually associated with a headache. These bleeds have a highly variable prognosis, dependent on their size and location.

K25928_C007.indd 316

12/01/2017 12:19

Case 24: The 51-year-old man with tingly fingers

317

Case 24: The 51-year-old man with tingly fingers You are asked to see a 51-year-old truck driver. He states that about a year ago he began to notice numbness and tingling in the 4th and 5th digits of his left hand. He thought little of it, but over the last several months he has noticed that he keeps dropping things out of his left hand. He thinks it is worst right after a long day of driving. He decided to seek medical help when he could no longer hold onto his cell phone reliably. His language examination shows normal fluency, repetition and comprehension. He has pupils that are equal and full visual fields and eye movements. He does not have a facial droop and his tongue and palate are midline. You examine his left hand and find significant atrophy in nearly the entire hand, but it is especially pronounced near the 4th and 5th digits. His formal power examination is recorded below but he has full strength in his legs. His reflexes are normal and his toes are downgoing. His sensory examination is normal in the legs, however, in the arms, he has a profound loss to pinprick in the 5th digit of the left hand. Digits 1, 2, and 3 have normal sensation, and he could feel the pin on the lateral aspect of the 4th digit, but not the medial. His coordination examination is normal.

Power Examination Right

Left

Deltoid

5

5

Bicep

5

5

Tricep

5

5

WE

5

5

WF

5

4-

FE

5

5

FF*

5

4

DI

5

4-

EP

5

5

FP

5

5

ADP

5

4-

ABP

5

5

OP

5

5

*4th and 5th digits only; 2nd and 3rd digits normal power.

Where is the lesion? What is going on here?

K25928_C007.indd 317

12/01/2017 12:20

318

Case 24: Findings

Case 24: Findings

Some individual figures adapted from Waxman SG. Clinical Neuroanatomy. Lange, New York, 2013.

K25928_C007.indd 318

12/01/2017 12:20

Case 24: Solution

319

Case 24: Solution 1. Does the patient have UMN weakness, LMN weakness or both? Our patient presents with isolated hand weakness. An UMN type problem is certainly possible, but it would have to be a very small lesion in order to only affect the hand; a LMN type problem is much more likely. In addition, the patient has normal reflexes and downgoing toes which are not in keeping with an UMN type lesion. Finally, we have found significant atrophy of the affected hand, which is characteristic of a LMN type weakness; we can thus conclude that our lesion lies in the PNS. Potential localizations include a radiculopathy, plexopathy, or neuropathy. 2. Do the symptoms correspond to a single myotome and dermatome? Checking our table of dominant myotomes (Fig.  5.5) we see that the dorsal interossei are innervated by T1 and the flexor digitorum is innervated by C8. Furthermore, it is only our patients’ 4th and 5th flexor digitorum that are weak, which cannot be caused by a radiculopathy. Checking our table of dermatomes (Fig.  5.15) you might be tempted to say that our patient’s sensory loss corresponds to the C8 dermatome. But if you look closely, you’ll see that the C8 dermatome involves both the 4th and 5th digits in their entirety. Our patient has intact sensation of the medial side of the 4th digit but not on the lateral. We can thus safely conclude our patient’s symptoms are not caused by a radiculopathy. 3. Does the pattern of symptoms correspond to a single nerve? In Chapter 5 we mentioned that the ulnar nerve supplies the majority of the hand muscles, with a few notable exceptions, so let’s consider whether our patient has an ulnar neuropathy. We’ll begin with the sensory exam; as you can see the ulnar nerve “splits” the 4th digit, which is what our patient complains of. Splitting of sensation across the 4th digit is completely unique to an ulnar neuropathy. In addition, the ulnar nerve innervates the flexor digitorum of the 4th and 5th digit only! This again is characteristic of an ulnar neuropathy. We can safely conclude that is what we are dealing with. Let’s draw out the path for the ulnar nerve and see where exactly our lesion is located. It is easiest to begin distally; we know that the hand muscles are involved so we are at least at the level of the wrist/Guyon’s canal. Moving up, we see that wrist flexion, provided by flexor carpi, is also affected, so we are at least at the level of the elbow. The ulnar nerve

K25928_C007.indd 319

12/01/2017 12:20

320

Case 24: Solution

does not have any branches in between the elbow and axilla, so that is as far as we can tell clinically. This case is an example of an ulnar neuropathy at the elbow, which is the second most common peripheral neuropathy after carpal tunnel syndrome. In this neuropathy, the ulnar nerve gets compressed by the distal humerus in an area of the cubital tunnel. This happens whenever people rest their forearms on objects, such as, in this case, the driver’s window of a truck.

K25928_C007.indd 320

12/01/2017 12:20

Case 24: Clinical Pearl

321

Figure 7.24 Nerve Injuries

K25928_C007.indd 321

12/01/2017 12:20

322

Case 24: Clinical Pearl

Clinical Pearl: Nerve Injuries There are several types of injury that can occur to peripheral nerves, but before we can consider them we need to review the internal architecture of a peripheral nerve. A nerve is a bundle of axons. There are several important layers of connective tissue in a nerve. Each individual axon is covered in a layer of protective connective tissue called an endoneurium. Axons travel together and are surrounded by a layer of protective tissue called the perineurium. The perineurium and the axons that it contains is referred to as a fascicle. The outermost layer of tissue of the nerve is the epineurium. Between the fascicles is adipose tissue and arteries, called the vasa nervorum, as well as veins. Infarction of the vasa nervorum can lead to death of that part of the nerve. In 1943, HJ Seddon came up with a classification of peripheral nerve injury still used today. He separated nerve injuries into three types: 1. Neurapraxia – in this case, there has been an attack of focal demyelination, but the axon of the nerve remains completely intact. The nerve does not conduct action potentials because of the myelin disruption. However, the Schwann cells will soon replace the myelin and the nerve is fully restored within hours to days. 2. Axonotmesis – in this case the axon itself has been disrupted, BUT the endoneural sheaths remain intact. The nerve undergoes a process called Wallerian degeneration. Distal to the injury, the axon and its components are degraded by macrophages. The axon will regrow, and the intact endoneural sheath serves to guide it to the correct location in the muscle. The axon grows at a rate of about 1.5 mm per day. Thus the nerve may take months or even years to recover. Recovery will likely not be full. 3. Neurotmesis – in this case there is a disruption of the connective tissue. Less severe injuries have a disruption of the axon and the endoneurium, but more significant injuries will include the perineurium and even the epineurium, leading to a total transection of the nerve. Wallerian degeneration occurs, but because there is no endoneurium to guide it, growth of the axon is chaotic and occurs in all directions leading to the formation of a useless neuroma. Nearly all cases require surgical intervention in order to reconnect connective tissue. If successful, then the axon will regrow at the 1.5 mm/day as above. If the axon cannot recover, the muscle that it innervates can be re-innervated by a neighbouring axon in a process called sprouting.

K25928_C007.indd 322

12/01/2017 12:20

Case 25: The 59-year-old man who couldn’t read

323

Case 25: The 59-year-old man who couldn’t read Your friend, a psychiatrist, has asked you to see a patient that was initially referred to her. The patient is a 59-year-old right handed man who has the odd complaint of not being able to read. He previously worked as an accountant and tells you that he just woke up this way one day. You text your soon to be ex–friend and ask her why on earth she referred the patient to you. However, before you dismiss the patient’s complaint you decide to dutifully do a neurological exam. His language examination is normal; he speaks fluently, understands complex instructions and can repeat. You hand him a book, and ask him to read. He tries, but he says he’s unsure if there are even words on the page. You give him a pen, and ask him to write something, assuming that he will return to you some nonsensical scribbles. You’re taken a back when he hands you back the sheet with “I am in the Doctor’s office, because I cannot read.” You ask him to write several more sentences, and they all return in perfect order. You shuffle the papers, and hand him a random sentence he wrote, asking him to read it. He states he cannot. You continue your examination; you are quite surprised to find that he has lost the right aspect of his visual field in each eye. His pupils are equal and he has full eye movements. He does not have a facial droop and his tongue and palate are midline. His motor examination including tone, reflexes, and power is completely normal. He has downgoing toes bilaterally. Sensory examination to pinprick and vibration is normal. Coordination testing is also normal. Where is the lesion? What is going on here? How expensive a dinner will you buy your psychiatrist friend in apology?

K25928_C007.indd 323

12/01/2017 12:20

324

Case 25: Findings

Case 25: Findings

K25928_C007.indd 324

12/01/2017 12:20

Case 25: Solution

325

Case 25: Solution In this case, we’ll make a quick departure from our regular localizing algorithm as there are no long tract findings to guide us. We should begin with our patient’s visual field complaints. He has lost the right side of his visual field in both eyes. This is consistent with a RHH. Recalling the visual pathway, Fig. 3.5, we know that this is only possible for lesions that are posterior to the optic chiasm. Since our patient has a RHH, we suspect that there is a lesion of the left occipital lobe. Remember, aphasia is a disorder of language, not just of speech, even though speech problems are the most common presentation. Thus, our patient is aphasic. In this case the patient cannot read, which is called alexia, but can write perfectly. In other words, he has alexia without agraphia (agraphia is the inability to write). Perhaps if we consider what brain processes are involved in reading we will be able to further classify our patient’s lesion. Firstly, he has a RHH; he is completely dependent on his left visual field for all visual information. Thus, all visual information, including words on the page of a book, is being received by the right occipital lobe. Language is controlled by the dominant hemisphere. Since our patient is right handed, this means that the visual information needs to be transmitted to Wernicke’s area in the left hemisphere, for interpretation. This is facilitated by the splenium of the corpus callosum. We know that our patient must have a lesion to the left occipital lobe in order to cause the RHH. A particularly large lesion would extend into the corpus callosum, cutting off this decussation of visual information to Wernicke’s area. This means that the patient can see the words, but not be able to send them to Wernicke’s area for interpretation into language. This would explain the alexia. Wernicke’s area itself, however, is intact, which is why he does not have any problems with understanding speech. In addition, Broca’s area is also intact, meaning the production of language, speech or written, is normal; he does not have agraphia. It seems that we can explain our patient’s symptoms with a large lesion to the left occipital cortex, extending into the corpus callosum! Alexia without agraphia is an example of a disconnection syndrome, where the symptoms are caused by a lesion in the white matter. This causes a critical interruption in the transmission of information from one part of the brain to another. While quite rare in real practice, it is particularly known for coming up in exams.

K25928_C007.indd 325

12/01/2017 12:20

326

Case 25: Clinical Pearl

Figure 7.25 White Matter Tracts of the Brain

K25928_C007.indd 326

12/01/2017 12:20

Case 25: Clinical Pearl

327

Clinical Pearl: White Matter Tracts of the Brain White matter tracts pervade the brain. The white matter just below the level of the cortex is called the centrum semiovale. Just inferior to the centrum semiovale lays the corona radiata. White matter can be divided into three different types of tract: 1. Projection tracts: these tracts connect the cortex to various other parts of the CNS, including the basal ganglia, cerebellum and spinal cord. We have already met many examples of these including the long tracts, optic radiations, auditory radiations and thalamocortical fibers. 2. Association tracts: these tracts connect parts of the same hemisphere to each other; an example is the arcuate fasciculus that connects Wernicke’s area to Broca’s area. U-fibers connect adjacent areas of cortex. The uncinate fasciculus connects the inferior frontal lobe to the temporal lobe. The inferior longitudinal fasciculus connects the occipital lobe to the temporal lobe and allows for the recognition of objects. The superior longitudinal fasciculus connects the frontal lobe to the parietal and occipital lobes. The cingulum is part of the cingulate gyrus and helps connect the basal ganglia. 3. Commissural tracts: these tracts connect the individual hemispheres to each other. The corpus callosum is by far the largest and most important tract in this category. The much smaller anterior commissure connects the two temporal lobes to each other and the posterior commissure connects the pretectal nuclei of the midbrain. The hippocampal commissure connects the hippocampi and plays an important role in the circuits of the brain responsible for memory. Knowledge of the white matter tracts is very important because they are often hijacked in patients with epilepsy and used to spread seizure discharges from one area of the brain to another. For example, a corpus callosotomy is a procedure in which the corpus callosum is surgically transected; in patients with certain types of epilepsy this procedure can reduce seizure frequency by upwards of 90%.

K25928_C007.indd 327

12/01/2017 12:20

329

Index Note: page numbers in italics refer to figures and tables.

1st, 2nd and 3rd order neurons 12, 13 abducens nerve (CN VI) 56, 57, 69, 71 exit from the brainstem 58 exit through the base of the skull 86 pathway through the cavernous sinus 76, 77 abducens nucleus 68, 73 abductor pollicis 136, 137 aberrant firing 111 acetylcholine (Ach) 194–5 action potentials 7 acute inflammatory demyelinating polyneuropathy see Guillain–Barré syndrome adductor pollicis 134, 135 adrenal glands 195 adrenergic neurons 194–5 agnosia 23, 40, 41 agraphesthesia 40, 41 case study 287–9 alexia, case study 323–5 amyotrophic lateral sclerosis (ALS) 236–7 case study 233–5 aneurysms common sites 219 MCA 209–12 PCOM 215–18 anhidrosis, Horner’s syndrome 223 anisocoria CN III palsy 215–18 medical vs. surgical 219–20 epidural hematoma 311–14 Horner’s syndrome 192, 193, 221–4, 225–6, 299–302 lateral medullary syndrome 269–71

K25928_C008.indd 329

ankle common peroneal nerve 146–7 tibial nerve 144–5 ankle reflex, innervation 126, 127 anosognosia 41 anterior cerebral artery (ACA) 16, 17, 46, 47 ACA stroke 164 territory 50, 51 anterior choroidal artery 46, 47 territory 50, 51 anterior circulation 16, 17, 46–7 anterior commissure 327 anterior communicating artery (ACOM) 16, 17, 46, 47 aneurysms 219, 220 anterior cord lesions 171 case study 185–7 anterior horn cells (AHCs) 10, 11, 33, 100, 101 lesions in poliomyelitis 188, 189 anterior inferior cerebellar artery (AICA) 16, 17, 48, 49 territory 91, 92, 93 anterior limb, internal capsule 30, 31 anterior nerve roots 10, 11 anterior radicular arteries 114, 115 anterior root, spinal nerves 100, 101 anterior spinal artery 48, 49, 114, 115 anterior spinal artery stroke 187 aphasia 23, 37, 38–9 case studies MCA stroke 179–81 superior branch of the MCA stroke 239–41

10/01/2017 10:50

330

Index

apraxia 23, 43 case studies parasagittal meningioma 287–9 subarachnoid hemorrhage 209–12 arachnoid granulations 18, 19 arachnoid mater 18, 19 arcuate fasciculus (AF) 36, 37 areflexic paralysis 309 arterial supply central nervous system 16, 17 anterior circulation 46–7 medulla 90–1 midbrain 94–5 pons 92–3 posterior circulation 48–9 vascular territories of the brain 50–1 spinal cord 114–15 artery dissection 259, 272–3 association cortex 40, 41 association tracts 327 astereognosis 40, 41 ataxia 45 heel to shin test 44 lateral medullary syndrome 269–71 atrial fibrillation 183 atrophy 111 auditory agnosia 41 auditory association cortex 40 auditory cortex 40 autonomic nervous system 4, 5 cranial nerves 56, 57 overview 20, 21 pupillary size control 66–7 see also parasympathetic nervous system; sympathetic nervous system autonomic neurotransmitters 194–5 axillary nerve 125, 130, 131 formation in the brachial plexus 122 sensory distribution 148, 149 axonotmesis 321, 322 axons 6, 7 Babinski sign (up-going toe) 112, 113 basal ganglia 26, 27 arterial supply 50–1 basilar artery 16, 17, 46, 48, 49, 114 aneurysms 219 stroke 245–7

K25928_C008.indd 330

territory 92, 93 top of the basilar syndrome 257–60 basilar perforators 48, 49, 254, 255 “Bellman’s tip” pose 192, 193 berry (saccular) aneurysms 220 see also aneurysms bicep reflex, innervation 126, 127 biceps 131 bitemporal hemianopsia 64, 65 pituitary gland tumor 275–8 bleeds see cerebral bleeds; subarachnoid hemorrhage brachial plexus 10, 11, 121, 122–3 how to draw it 124–5 lower trunk (Klumpke’s) palsy 173 upper trunk (Erb–Duchenne) palsy 173 case study 191–3 brain herniation syndromes 261–2 brainstem 8, 9 arterial supply 91 exit of cranial nerves 58–9 peduncles 58, 59 “Rule of 4” 88–9 brainstem lesions crossed signs 55 in multiple sclerosis 296, 297 bridging veins, rupture of 316 Broca’s aphasia 38, 39 case study 239–41 Broca’s area 24, 25, 36, 37 Brown–Sequard syndrome 171 case study 299–302 buccal branch, CN VII 78, 79 buccofacial apraxia 43 callosomarginal artery 46, 47 Campylobacter jejuni, Guillain–Barré syndrome 308, 309 “cape” sensory loss 230, 231 cardiac embolism 183 carotid endarterectomy 242, 243 carotid stenosis 242–3 carotid stenting 242, 243 carpal tunnel 136 carpal tunnel syndrome 207–8 cauda equina 98, 99, 267–8 cauda equina syndrome 170 case study 281–4

10/01/2017 10:50

Index

caudate 26, 27 arterial supply 50 cavernous sinus 47, 52, 53, 76–7 bleeding into 277–8 central cord syndrome 172 case study 227–9, 230–1 central disc herniation 267, 268 central herniation 261, 262 central nervous system (CNS) 4, 5 arterial supply 16, 17 anterior circulation 46–7 medulla 90–1 midbrain 94–5 pons 92–3 posterior circulation 48–9 vascular territories of the brain 50–1 long tracts 12, 13 overview 8, 9 somatotopy 14, 15 central sulcus 9 centrum semiovale 327 cerebellar peduncle 58, 59 cerebellum 8, 9, 44–5 arterial supply 48–9 cerebral aqueduct 18 cerebral bleeds 315–16 epidural hematoma, case study 311–14 see also subarachnoid hemorrhage cerebral infarction see stroke cerebral peduncle 58, 59 cerebrospinal fluid (CSF) 18, 19 analysis of 285–6 Guillain–Barré syndrome 309 hydrocephalus 290–2 cerebrum 8, 9 cervical branch, CN VII 78, 79 cervical cord 102, 103 cervical cord lesions 116, 117 high transection 168 low transection 169 cervical nerves 10, 11 cervical vertebrae 98, 99 chief sensory nucleus 75 cholinergic neurons 194–5 chorda tympani 78 choroid plexus 18, 19 ciliary ganglion 60, 66, 67 cingulate gyrus 24, 25, 26

K25928_C008.indd 331

331

cingulum 326, 327 Circle of Willis 16, 17, 19 clonus 112, 113 cocaine eye drops, diagnosis of Horner’s syndrome 225, 226 cochlear nerve 80, 81 cochlear nucleus 80, 81 collateral vasculature Circle of Willis 16, 17, 19 of the spinal cord 115 coma 214 Glasgow Coma Scale 213 see also loss of consciousness commissural tracts 327 common carotid artery 16, 17 common peroneal nerve 142, 143, 145, 146–7 formation in the lumbosacral plexus 138, 139 sensory distribution 148, 149 common peroneal neuropathy 176 case study 263–6 communicating arteries 16, 17 see also anterior communicating artery; posterior communicating artery conception tract, language circuit 37 conduction aphasia 38, 39 confluence of sinuses 52, 53 ‘coning’ (tonsillar herniation) 262 conjugate eye movement 73 conscious sensation 5 consciousness 29 levels of 213–14 see also loss of consciousness constructional apraxia 43 conus medullaris 98, 99 coordination, screening examination 161 corpus callosum 24, 26, 27, 327 arterial supply 50, 51 cortex arterial supply 50–1 cerebral 9 specialized areas 24, 25 corticobulbar tract 31, 32–3 see also motor pathways corticospinal tract 12, 13, 32–3 crossed signs 55 in the internal capsule 31

10/01/2017 10:50

332

Index

lesions 117 somatotopy 106, 107 see also motor pathways corticothalamic fibers 31 cranial nerves 4, 5 CN I (olfactory nerve) 86 CN II (optic nerve) 56, 57, 62, 63 CN III (oculomotor nerve) 49 CN III palsy case study 215–18 medical vs. surgical 219–20 CN IV (trochlear nerve) 69, 71, 76, 77 CN V (trigeminal nerve) 74–5, 76, 77 CN VI (abducens nerve) 69, 71, 76, 77 CN VII (facial nerve) 78–9 CN VIII (vestibulocochlear nerve) 80–1 CN IX (glossopharyngeal nerve) 80–1 CN X (vagus nerve) 82–3 CN XI (spinal accessory nerve) 84, 85 CN XII (hypoglossal nerve 84, 85 exit from the brainstem 58–9 exit through the base of the skull 86–7 extraocular muscle innervation 68–9 eye movements 70–1 conjugate gaze 72–3 facial sensation 74–5 motor innervation to the neck and the tongue 84–5 muscles of mastication 74, 75 overview 56–7 parasympathetic component 20, 21, 60–1 CN IX 80 CN VII 78 CN X 82, 83 pathway through the cavernous sinus 76–7 screening examination 159 CREST (Carotid Revascularization Endarterectomy Stenting Trial) 243 cribriform plate 86 crossed signs 55 basilar artery stroke 245–7 lateral medullary syndrome 269–71 medial medullary syndrome 197–9 sensory 97, 104, 105 cryptogenic stroke 183 CSF see cerebrospinal fluid

K25928_C008.indd 332

decerebrate posturing 213, 214 decorticate posturing 213, 214 decussation sites corticospinal tract 33 dorsal columns 35 long tracts 12, 13 optic chiasm 62, 63 spinothalamic tract 35 deep peroneal nerve 146, 147 deltoid muscle 130, 131 demyelination 297 dendrites 6, 7 dermatomes 126, 127, 148, 149 diabetes, lipohyalinosis 254 diplopia multiple sclerosis 293–5 pituitary gland tumor 275–7 top of the basilar syndrome 257–60 disconnection syndromes, case study 323–5 discs see intervertebral discs dissection, arterial 259, 271, 272–3 divisions of the nervous system 4, 5 dominant myotomes 128–9 dorsal columns (DC) 12, 13, 29, 34–5, 104, 105 crossed signs 55 lesions 117 tabes dorsalis 188, 189 screening examination 161 somatotopy 106, 107 dorsal interossei 134, 135 dorsal motor nucleus 60, 61, 82, 83 location in the medulla 90 dorsal root ganglion (DRG) 10, 11, 13, 100, 101 lesions in tabes dorsalis 188, 189 “down and out” eye 215–18 drowsiness 214 dura mater 18, 19 dysarthria 37 case study 197–9 dysconjugate gaze 73 Edinger–Westphal nucleus 60, 61, 66, 67, 94, 95 El Escorial criteria, amyotrophic lateral sclerosis 236, 237 elbow, ulnar neuropathy 317–20

10/01/2017 10:50

Index

endarterectomy 242, 243 endoneurium 321, 322 epidural hematoma 315, 316 case study 311–14 epidural space 18, 19 epineurium 321, 322 Erb–Duchenne palsy 173 case study 191–3 extradural spinal cord lesions 303, 304 extensor carpi 132, 133 extensor digitorum 132, 133 extensor hallucis longus 146, 147 extensor pollicis 132, 133 external carotid artery 16, 17 extraocular muscles 68–9 eye movements 70–1 conjugate gaze 72–3 “down and out” eye 215–18 internuclear ophthalmoplegia 293–5 “wrong way” eyes 248–9 case study 245–7 facial droop 78, 79 LMN 245–7 UMN lacunar stroke 251–3 MCA stroke 179–81 subarachnoid hemorrhage 209–12 superior branch of the MCA stroke 239–41 top of the basilar syndrome 257–60 facial nerve (CN VII) 56, 57, 78–9 exit from the brainstem 58 exit through the base of the skull 86 parasympathetic component 60, 61 facial nucleus 78, 92, 93 facial sensation 74–5 false localizing signs 314 falx cerebri 18 fasciculations 111 femoral nerve 142, 143 formation in the lumbosacral plexus 138, 139 sensory distribution 148, 149 flexor carpi 134, 135 flexor digitorum 134, 135, 136, 137 flexor pollicis 136, 137 fluency of speech 39

K25928_C008.indd 333

333

foramen magnum 86 foramen ovale 86 foramen rotundum 86 fourth ventricle 18 frontal eye field (FEF) 24, 25, 73 lesions 181, 241, 248–9 frontal lobe 8, 9 arterial supply 50 gait screening examination 161 wide based 45 ganglia 20, 21 definition of term 7 gastrocnemius 144, 145 geniculate ganglion 78, 79 geniculate nuclei 28 genu corpus callosum 26 internal capsule 30, 31 geographic apraxia 43 Glasgow Coma Scale 213–14 global aphasia 38, 39 globus pallidus 26, 27 glossopharyngeal nerve (CN IX) 56, 57, 80–1 exit from the brainstem 58 exit through the base of the skull 86 parasympathetic component 60, 61 gluteus maximus 140, 141 gluteus minimus and medius muscles 140, 141 gray matter cerebral 9 spinal cord 100, 101 Great vein of Galen 52, 53 greater petrosal nerve 78 Guillain–Barré syndrome 308–9 case study 305–7 CSF analysis 285, 286 Guyon’s canal 134, 135 gyri 8, 9 hallucinations, top of the basilar syndrome 257–60 hamstrings 142, 143 Hand of Benediction (Preacher’s Hand) 207, 208

10/01/2017 10:50

334

Index

headache, subarachnoid hemorrhage 209, 212, 316 hearing 81 heel to shin test 44, 45 hemianopsia 64, 65 hemi-neglect 43 case study 209–12 hemispheres, cerebral 9 hemorrhage see cerebral bleeds; subarachnoid hemorrhage hemorrhagic stroke 182, 183 herniation brain 261–2 epidural hematoma 316 intervertebral discs 267–8 cauda equina syndrome 281–4 higher functions 23 hip, nerves and muscles of 140–1 hippocampal commissure 327 homonymous hemianopsia 64, 65, 323–5 case studies MCA stroke 179–81 top of the basilar syndrome 257–60 homonymous quadrantanopsia 64, 65 homunculi 14, 15, 24 horizontal gaze center (CN VI nucleus) 73 Horner’s syndrome 225–6 case studies 221–4 brachial plexus lesion 191–3 Brown–Sequard syndrome 299–302 lateral medullary syndrome 269–71 hydrocephalus 290–2 hydroxyamphetamine, diagnosis of Horner’s syndrome 225, 226 hypertension, lipohyalinosis 254 hypoglossal canal 86 hypoglossal nerve (CN XII) 56, 57, 84, 85 exit from the brainstem 58 exit through the base of the skull 86 hypoglossal nucleus 84, 85 location in the medulla 90 ideomotor apraxia 43 iliopsoas muscle 140, 141 inferior ganglion 80, 81 inferior gluteal nerve 140, 141 formation in the lumbosacral plexus 138, 139

K25928_C008.indd 334

inferior longitudinal fasciculus 326, 327 inferior oblique muscle 68, 69 inferior olive 90, 91 inferior rectus muscle 68, 69 inferior sagittal sinus 52, 53 inferior salivatory nucleus 60, 61, 80, 81 inferior vagal ganglion 82 insula 26, 27 intercostal nerves, sensory distribution 148, 149 internal acoustic meatus 78, 79, 86 internal capsule 26, 27, 30–1 arterial supply 50, 51 lesions associated features 165 lacunar stroke 251–3, 254–5 somatotopy 14 internal carotid artery (ICA) 16, 17, 46, 47 pathway through the cavernous sinus 76, 77 internal cerebral veins 52 internal jugular vein 52, 53 internal medullary lamina 28, 29 internuclear ophthalmoplegia (INO) 295 interspinous ligament 98, 99 intervertebral discs 98, 99 herniation 267–8 cauda equina syndrome 281–4 intracerebral hemorrhage 315, 316 intracranial pressure (ICP), Monro–Kellie (MK) Doctrine 291–2 intradural extramedullary spinal cord lesions 303, 304 intralaminar nuclei, thalamus 29 intramedullary spinal cord lesions 303, 304 ischemic stroke 182, 183 jugular foramen 86 Kernohan’s notch 261, 262, 314 Klumpke’s palsy 173 knee, nerves and muscles of 142–3 knee reflex, innervation 126, 127 L5 radiculopathy 268 differentiation from peroneal neuropathy 265–6

10/01/2017 10:50

Index

lacunar stroke 183, 254–5 case study 251–3 language screening examination 159 see also alexia; aphasia language areas 25 language circuit 36–7 and aphasias 38–9 lateral cutaneous nerve of the forearm 130, 131, 148, 149 lateral cutaneous nerve of the thigh 148, 149 lateral geniculate nucleus (LGN) 28, 62, 63 lateral medullary syndrome 168 case study 269–71 lateral pons, lesions of 167 lateral rectus muscle 68, 69 lateral ventricle 18 lenticulostriate arteries 46, 47, 255 lesser petrosal nerve 80 levator palpebrae superioris 217 level of consciousness (LOC) 213–14 line bisection task 42, 43 lipohyalinosis 254, 255 localization, false signs 314 localization algorithm 154–8 locked-in syndrome 200–1 long tract lesions 117 crossed signs 55 long tracts 12, 13 somatotopy 106–7 see also corticospinal tract; dorsal columns; medial lemniscus; spinothalamic tract loss of consciousness brain herniation syndromes 261-2 epidural hematoma 311–14, 316 Glasgow Coma Scale 213–14 PCOM aneurysm rupture 215–17 top of the basilar syndrome 257–60 Lou Gehrig’s disease see amyotrophic lateral sclerosis lower motor neuron lesions 110–11 localization algorithm 154, 157–8 see also facial droop lower motor neurons (LMNs) 12, 13, 32, 33 lumbar cord 102, 103 lumbar cord lesions 116 transection 170

K25928_C008.indd 335

335

lumbar nerves 10, 11 lumbar puncture 285–6 in Guillain–Barré syndrome 309 lumbar vertebrae 98, 99 lumbosacral anatomy 267–8 lumbosacral plexus 10, 11, 121 how to draw it 138–9 mandibular branch, CN V 74, 75 mandibular branch, CN VII 78, 79 Marcus Gunn pupil (RAPD) 279–80 masseter muscle 75 mastication 57, 74, 75 maxillary branch of CN V 74, 75 McDonald’s Criteria, multiple sclerosis 296, 297 Meckel’s cave 75 medial cutaneous nerve of the arm 148, 149 medial geniculate nucleus (MGN) 28 medial lemniscus (ML) 12, 13, 35, 293–5 location in the medulla 90 location in the midbrain 94, 95 location in the pons 92, 93 medial longitudinal fasciculus (MLF) 72–3 lesions 293–5 location in the medulla 90, 91 location in the midbrain 94 location in the pons 92, 93 medial medullary syndrome 167 case study 197–9 medial pons, lesions 166 medial rectus muscle 68, 69 median nerve 125, 136–7 formation in the brachial plexus 122 sensory distribution 148, 149 median nerve lesions 174 carpal tunnel syndrome 207–8 medical CN III palsy 219, 220 medulla 8, 9 exit of cranial nerves 58, 59 lateral medullary syndrome 168, 269–71 location of structures 90, 91 long tracts 12 medial medullary syndrome 167, 197–9 vascular territories 90–1 meningeal dura mater 18, 19 meninges 18, 19 meningioma, case study 287–9

10/01/2017 10:50

336

Index

meningitis, CSF analysis 285, 286 mesencephalic trigeminal nucleus 75 midbrain 8, 9 exit of cranial nerves 58, 59 long tracts 12 vascular territories 94–5 midbrain syndrome 166 middle cerebral artery (MCA) 16, 17, 46, 47 aneurysms 219, 220 case study 209–12 MCA stroke 164 Case study 179–81 superior branch 239–41 territory 50, 51 middle meningeal artery (MMA) 316 Millard Gubler syndrome, case study 245–7 monocular vision loss 64, 65 mononeuropathy 120, 121 common peroneal nerve 176, 263–6 median nerve 174 carpal tunnel syndrome 207–8 radial nerve 174, 203–6 sciatic nerve 176 tibial nerve 175 ulnar nerve 175, 317–20 mononeuropathy multiplex 121 Monro–Kellie (MK) Doctrine 291–2 motor cortex 8, 24, 25 arterial supply 50, 51 somatotopy 14 motor homunculus 24, 25 motor pathways 12, 13, 32–3 crossed signs 55 lesions 117 amyotrophic lateral sclerosis 233–5, 236–7 internal capsule stroke 251–3 location in the medulla 90, 91 location in the midbrain 94, 95 location in the pons 92, 93 somatotopy 106, 107 motor system muscle table 162–3 neurotransmitters 194–5 screening examination 159 muscle power assessment 160–1 MRC Grading of Muscle Power Scale 160–1 Muller’s muscle 217

K25928_C008.indd 336

multiple sclerosis 296–7 case study 293–5 muscarinic acetylcholine receptors 195 muscle table 162–3 musculocutaneous nerve 125, 126, 127, 130, 131 formation in the brachial plexus 122 myelin 6, 7 myotomes 127, 128–9 NASCET (North American Symptomatic Carotid Endarterectomy Trial) 243 neck, motor innervation 84, 85 neglect 43 case study 209–12 nerve injuries 321–2 nerve roots 121 neural foramen 98, 99 neurapraxia 321, 322 neuromas 322 neurons, anatomy of 6–7 neuropathy 121 neurotmesis 321, 322 neurotransmitters, autonomic 194–5 nicotinic acetylcholine receptors 195 Nodes of Ranvier 6, 7 nondominant hemisphere lesions 43 nonobstructive (communicating) hydrocephalus 290, 291 nonspecific nuclei, thalamus 29 norepinephrine (NE) 194–5 nuclear lesions 295 nuclei for cranial nerves 57, 60–1 nuclei of the thalamus 28–9 ‘nucleus’, use of the term 7 nucleus ambiguus 80, 81, 82, 83, 85 location in the medulla 90 nucleus solitarius 57, 78, 79, 80, 81, 82, 83 obstructive (noncommunicating) hydrocephalus 290, 291 obtunded patients 214 case study 245–7 obturator nerve 140, 141 formation in the lumbosacral plexus 138, 139 sensory distribution 148, 149

10/01/2017 10:50

Index

occipital lobe 8, 9, 62, 63 arterial supply 48, 49, 50, 51 oculomotor nerve (CN III) 56, 57, 69, 71 CN III palsy case study 215–18 medical vs. surgical 219–20 exit from the brainstem 58 exit through the base of the skull 86 parasympathetic component 60, 61 pathway through the cavernous sinus 76, 77 pupillary size control 67 oculomotor nucleus 68, 94, 95 olfaction 57 olfactory nerve (CN I) 86 oligodendrocytes 7 ophthalmic artery 46, 47 ophthalmic branch of CN V 74, 75 opponens pollicis 136, 137 optic canal 86 optic chiasm 62, 63 relationship to the cavernous sinus 77 optic nerve (CN II) 56, 57, 62, 63 exit from the brainstem 58–9 exit through the base of the skull 86 optic neuritis 296, 297 optic radiations 62, 63 optic tract 62, 63 otic ganglion 60, 80 pain sensation 13 see also spinothalamic tract paramedian pontine reticular formation (PPRF) 72, 73 parasagittal meningioma, case study 287–9 parasympathetic nervous system 4, 5 cranial nerves 56, 57, 60–1 CN IX (glossopharyngeal nerve) 80 CN VII (facial nerve) 78, 79 CN X (vagus nerve) 82, 83 neurotransmitters 194–5 overview 20, 21 pupillary size control 66–7 parietal lobe 8, 9 arterial supply 50 Parkinson’s disease 27, 95 pericallosal artery 46, 47

K25928_C008.indd 337

337

perineurium 321, 322 periosteal dura mater 18, 19 peripheral nerves 11 peripheral nervous system (PNS) 4, 5, 119 dominant myotomes 128–9 lesion types 120–1 overview 10, 11 reflexes 126, 127 sensory distribution 126–7, 148–9 summary 150 peroneal neuropathy 176 case study 263–6 peroneus muscle 146, 147 pia mater 18, 19 pituitary gland, relationship to the cavernous sinus 76, 77 pituitary gland tumor, case study 275–8 plexopathy 120, 121 poliomyelitis 188, 189 polyneuropathy 121 length dependent 307 see also Guillain–Barré syndrome pons 8, 9 exit of cranial nerves 58, 59 lacunar stroke 254 lateral lesions 167 long tracts 12 medial lesions 166, 245–7 “wrong way” eyes 248–9 vascular territories 92–3 postcentral gyrus 24, 25 posterior cerebral artery (PCA) 16, 17, 46, 48, 49 PCA stroke 165 territory 50, 51, 94 posterior circulation 16, 17, 48–9 posterior commissure 327 posterior communicating artery (PCOM) 16, 17, 46, 49 aneurysms 219, 220 case study 215–18 posterior cord lesions 172 posterior cutaneous nerve of the thigh 149 posterior horn 104, 105 posterior inferior cerebellar artery (PICA) 16, 17, 48, 49, 114, 115 territory 90, 91 posterior limb, internal capsule 30, 31

10/01/2017 10:50

338

Index

posterior nerve roots 10, 11 posterior radicular arteries 114, 115 posterior root, spinal nerves 100, 101 posterior spinal arteries 114, 115 posterolateral disc herniation 267, 268 postganglionic neurons 21 power assessment 160–1 Preacher’s Hand (Hand of Benediction) 207, 208 precentral gyrus 24, 25 preganglionic neurons 21 premotor cortex 24, 25 pretectal nucleus 67 primary auditory area 24, 25 primary motor cortex 9 primary progressive MS 297 primary sensory cortex 9 projection tracts 327 pronator teres 136, 137 proprioception 13 see also dorsal columns prosopagnosia 41 pterygoid muscles 75 pterygopalatine ganglion 60, 78 ptosis CN III palsy 215–17, 215–18, 219–20 epidural hematoma 311–14 Horner’s syndrome 222, 223, 225–6 pituitary gland tumor 275–8 top of the basilar syndrome 257–60 pulvinar nucleus 28 pupillary light reflex 67 pupillary size control 66–7 relative afferent pupillary defect (RAPD) 279–80 see also anisocoria pure word deafness 41 putamen 26, 27 arterial supply 50 quadrantanopsia 64, 65 quadriceps 142, 143 radial nerve 125, 132–3 formation in the brachial plexus 122 sensory distribution 148, 149 radial nerve lesions 174 case study 203–6

K25928_C008.indd 338

radiculopathy 120, 121 L5 268 differentiation from peroneal neuropathy 265–6 Ranvier, Nodes of see Nodes of Ranvier recurrent laryngeal nerve 82 red nucleus 94, 95 reflex arcs 108–9 effect of lower motor neuron lesions 111 effect of upper motor neuron lesions 111, 112 innervation 126, 127 relapsing remitting MS 297 relative afferent pupillary defect (RAPD) 279–80 resting tone 111 reticular activating system (RAS)? 214 retina 62, 63 rigidity 112, 113 “Rule of 4” 88–9 saccular (berry) aneurysms 220 see also aneurysms sacral cord 102, 103 sacral cord lesions 116 sacral nerves 10, 11 sacral sparing, central cord syndrome 230, 231 sacral vertebrae 98, 99 Saturday Night Palsy 203–6 Schwann cells 6, 7 sciatic nerve 142, 143 formation in the lumbosacral plexus 138, 139 sciatic nerve lesions 176 screening examination coordination and gait 161 cranial nerves 159 language 159 motor system 159 muscle power assessment 160–1 muscle table 162–3 sensory system 161 segmental arteries 114, 115 seizures 249 semicircular canals 80, 81 sensation, conscious and unconscious 5 sensory association cortex 24, 25, 40

10/01/2017 10:50

Index

sensory bands 97, 104, 105 central cord syndrome 230–1 case study 227–9 sensory cortex 8, 24, 25 arterial supply 50, 51 sensory homunculus 24, 25 sensory innervation 126, 127, 148 axillary nerve 130, 131 facial nerve (CN VII) 78–9 femoral nerve 142 glossopharyngeal nerve (CN VIII) 80–1 median nerve 136, 137 musculocutaneous nerve 130, 131 obturator nerve 140, 141 radial nerve 132, 133 sciatic nerve 142 tibial nerve 144 trigeminal nerve (CN V) 74–5 ulnar nerve 134, 135 vagus nerve (CN X) 82–3 sensory levels 97, 105 case study 185–7 sensory system, screening examination 161 sensory tracts 34–5 spinal cord 104–5 see also dorsal columns; spinothalamic tract “shawl” sensory loss 230, 231 sinuses, venous 18, 19, 52–3 cavernous sinus 76–7 sleep–wake cycle 29, 214 small vessel disease see lacunar stroke soleus muscle 144, 145 solitary nucleus 90 somatic nervous system 4, 5 somatotopy 14, 15, 24 cerebellum 44, 45 internal capsule 31 motor pathways 32–3 sensory pathways 34–5 spinal cord 101, 106–7 spinal nerves 101 spasticity 112, 113 specific nuclei, thalamus 29 speech see aphasia; language areas; language circuit spinal accessory nerve (CN XI) 56, 57, 84, 85 exit from the brainstem 58 exit through the base of the skull 86

K25928_C008.indd 339

339

spinal cord 4, 8, 9, 99, 100–1 areas of body innervated by 102, 103 arterial supply 114–15 cross sections 102, 103 long tracts 12 sensory 104–5 somatotopy 14, 106–7 stretch reflex 108–9 upper and lower motor neuron lesions 110–11 spinal cord disease 188–9 spinal cord lesions 116–17 anterior 171 case study 185–7 Brown–Sequard syndrome 171 case study 299–302 cauda equina 170 central cord syndrome 172, 230–1 case study 227–9 cervical spine high transection 168 low transection 169 classification of 303–4 lumbar transection 170 posterior cord 172 thoracic transection 169 spinal nerves 4, 5, 10, 11, 100, 101 spinal nucleus, location in the medulla 90 spinal trigeminal nucleus 75 spine 98–9 spinocerebellar tract location in the medulla 90, 91 location in the pons 92, 93 spinothalamic tract (ST) 12, 13, 29, 34–5, 104, 105 crossed signs 55 lesions 117 location in the medulla 90, 91 location in the midbrain 94, 95 location in the pons 92, 93 screening examination 161 somatotopy 106, 107 spinous processes 98, 99 spiral groove, humerus 206 splenium, corpus callosum 26 sprouting 322 stenting 242, 243 sternocleidomastoid (SCM) muscle 84, 85

10/01/2017 10:50

340

Index

straight sinus 52, 53 strawberry picker’s palsy see common peroneal neuropathy stretch reflexes 108–9 effect of lower motor neuron lesions 111 effect of upper motor neuron lesions 111, 112 grading of 109 innervation 126, 127 stroke anterior spinal artery 187 basilar artery 245–7 causes of 182–3 artery dissection 272–3 lacunar 254–5 case study 251–3 lateral medullary syndrome 269–71 medial medullary syndrome 197–9 middle cerebral artery occlusion 179–81 superior branch of the MCA case study 239–41 top of the basilar syndrome case study 257–60 stroke localization ACA 164 MCA 164 PCA 165 stupor 214 stylopharygeus muscle 80, 81 subacute combined degeneration (SACD), spinal cord 188, 189 subarachnoid hemorrhage 315, 316 case study 209–12 subarachnoid space 18, 19 subdural hematoma 315, 316 subdural space 18, 19 subfalcine herniation 261, 262 submandibular ganglion 60, 78 substantia nigra 26, 27, 94, 95 arterial supply 50 sulci 8, 9 superficial peroneal nerve 146, 147 superior cerebellar artery (SCA) 16, 17, 48, 49 territory 91, 94 superior gluteal nerve 140, 141 formation in the lumbosacral plexus 138, 139

K25928_C008.indd 340

superior longitudinal fasciculus 326, 327 superior oblique muscle 68, 69 superior orbital fissure 86 superior rectus muscle 68, 69 superior sagittal sinus 52, 53 superior salivatory nucleus 60, 61, 78, 79 supinator muscle 132, 133 supplementary motor area 24, 25 supraclavicular nerves, sensory distribution 148 supranuclear lesions 295 surgical CN III palsy 219, 220 swinging light test 279, 280 Sylvian fissure 8, 9 sympathetic chain 20 sympathetic nervous system 4, 5 Horner’s syndrome 225–6 case study 221–4 neurotransmitters 194–5 overview 20, 21 pupillary size control 66–7 sympathetics location in the medulla 90, 91, 92 location in the midbrain 94, 95 location in the pons 92, 93 syringomyelia 229, 231 tabes dorsalis 188, 189 tactile agnosia 41 temperature sensation 13 see also spinothalamic tract temporal branch, CN VII 78, 79 temporal lobe 8, 9 arterial supply 48, 49, 50, 51 temporalis muscle 75 tentorium cerebelli 18, 261 thalamocortical fibers 31 thalamus 26, 27 arterial supply 50 lacunar stroke 254 lateral geniculate nucleus 62, 63 long tracts 13 nuclei of 28–9 third ventricle 18 thoracic cord 102, 103 thoracic cord lesions 116 transection 169 thoracic nerves 10, 11

10/01/2017 10:50

Index

thoracic vertebrae 98, 99 thunderclap headache 209, 212, 316 tibial nerve 142, 143, 144–5 formation in the lumbosacral plexus 138, 139 sensory distribution 148, 149 tibial nerve lesions 175 tibialis anterior 146, 147 tibialis posterior 144, 145, 266 tone 111 effect of upper motor neuron lesions 112, 113 tongue deviation of 197–9 motor innervation 84, 85 tonsillar herniation 261, 262 top of the basilar syndrome 257–60 transcortical motor (TCM) aphasia 38, 39 transcortical sensory (TCS) aphasia 38, 39 transient ischemic attacks (TIAs) 243 case histories 221, 239 transverse myelitis 296 transverse processes 98, 99 transverse sinus 52, 53 trapezius muscle 84, 85 tremor, heel to shin test 44, 45 tricep reflex, innervation 126, 127 triceps 132, 133 trigeminal ganglion 74, 75 trigeminal motor nucleus 74, 75 trigeminal nerve (CN V) 56, 57, 74–5 exit from the brainstem 58 exit through the base of the skull 86 pathway through the cavernous sinus 76, 77 trigeminal nucleus 92, 93 trigeminal sensory nuclei 75 trochlear nerve (CN IV) 56, 69, 71 exit from the brainstem 57, 58 exit through the base of the skull 86 pathway through the cavernous sinus 76, 77 trochlear nucleus 68 truncal ataxia 45 U-fibers 326, 327 ulnar nerve 125, 134–5 formation in the brachial plexus 122 sensory distribution 148, 149

K25928_C008.indd 341

341

ulnar nerve lesions 175 case study 317–20 uncal herniation 261, 262 uncinate fasciculus 326, 327 unconscious sensation 5 unconsciousness, Glasgow Coma Scale 213–14 up-going toe (Babinski sign) 112, 113 upper motor neuron lesions 110–11 clinical features 112–13 localization algorithm 154–6, 158 see also facial droop upper motor neurons (UMNs) 12, 13, 32, 33 upward herniation 261, 262 urinary incontinence, case study 287–9 vagus nerve (CN X) 56, 57, 82–3 exit from the brainstem 58 exit through the base of the skull 86 parasympathetic component 60, 61 vasa nervorum 322 vascular territories of the brain 50–1 medulla 90–1 midbrain 94–5 pons 92–3 vegetative state 214 venous drainage of the head 52–3 venous sinuses 18, 19 ventral anterior (VA) nucleus 28 ventral lateral nucleus (VLN) 28 ventral posterior lateral (VPL) nucleus 28, 29, 35 ventral posteromedial (VPM) nucleus 28, 29, 35 ventricles 18, 19 hydrocephalus 290–2 vermis 44, 45 vertebrae 98, 99 vertebral artery 16, 17, 46, 48, 49, 114, 115 territory 90, 91 vertebral artery dissection 259, 271, 272–3 vertebral foramen 99 vestibular nerve 80, 81 vestibular nucleus 80, 81 location in the medulla 90 location in the pons 92

10/01/2017 10:50

342

Index

vestibulocochlear nerve (CN VIII) 56, 57, 80–1 exit from the brainstem 58 exit through the base of the skull 86 vibration sensation 13 see also dorsal columns visual agnosia 41 visual association cortex 40 visual cortex 24 visual field defects 64–5 bitemporal hemianopsia 275–8 homonymous hemianopsia 323–5 visual fields 62, 63 visual pathway 62–3 vitamin B12 deficiency 189 voluntary-automatic dissociation 43

K25928_C008.indd 342

Wallenberg syndrome (lateral medullary syndrome) 269–71 Wallerian degeneration 322 Wernicke’s aphasia 38, 39 Wernicke’s area 24, 25, 36 “whiplash” injury 231 white matter cerebral 9 tracts of the brain 326–7 in the spinal cord 100, 101 wide based gait 45 Willis, Circle of see Circle of Willis “wrong way” eyes 245–7, 248–9 zygomatic branch, CN VII 78, 79

10/01/2017 10:50